2003年09月 の投稿ログ


10334.階差数列  
名前:Miki(高1です)    日付:9月30日(火) 16時9分
(問題)
2,3,5,15,42,123,・・・の一般項を求めなさい。

解答見ますと
an=2+3^0+3^1+3^2+・・・+3^n-2
後は省略します。

※項数
0,1,2,・・・(n−2)←なぜn−1にならないのですか?
階差数列の一般項って原数列の初項と階差数列の初項から第(n-1)項までの
和を足して求めるんですよね?解説よろしくお願いします。


  



10335.Re: 階差数列
名前:Bob    日付:9月30日(火) 17時10分
まず「2,3,6,15,42,123,・・・の」の間違えですよね?

本題ですが、
※項数
0,1,2,・・・(n−2) 良く見てみましょう。
もし
0,1,2,・・・(n−1)にすると項はいくつありますか?
1から(n−1)項まではn−1項ですが、
はじめに0があるのでこれを足すとn−1+1でn項になってしまいます。

だからn−1項にするには、
0,1,2,・・・(n−2) にしなければいけないのです。

http://homepage3.nifty.com/sumida-3/


10338.Re: 階差数列
名前:Miki(高1です)    日付:9月30日(火) 17時28分
Bobさん♪説明ありがとうございました。理解できました。

>まず「2,3,6,15,42,123,・・・の」の間違えですよね
すみません、書き間違いました。


10354.Re: 階差数列
名前:ヨッシー    日付:10月1日(水) 13時43分
>原数列の初項と階差数列の初項から第(n-1)項までの和を足して求める
これは、正しいです。

ただ、第(n-1)項が 3^n-2 であることに気付かないといけません。
 
http://yosshy.sansu.org/

10333.教えて下さい。  
名前:メダカ(中1年)    日付:9月30日(火) 15時1分
@1/6(2n+4)=1/3(n+2)
A1/3{1/6n+1/2}=1/18(n+3)

@とAのカッコの外側に出す考え方がよく分かりません。
@はカッコの中を2で割って、その2をカッコの外側に出すと1/6に2を掛けてますよね?Aの方はカッコの中を最小公倍数の6を掛けて、その6をカッコの外側に出すと1/3を6で割ってますよね?う〜んよく分かりません。考え方を教えて下さい。よろしくお願いします。



10336.こう考えて見ては…
名前:Bob    日付:9月30日(火) 17時17分
(1/6)(2n+4)=(1/6)・2・(n+2)   2を()外へ
          =1/3(n+2)



1/3{1/6n+1/2}のほうは
=(1/3)・(1/6n+3/6)   通分
=(1/3)・{(n+3)/6}    {}内計算
=(1/3)・(1/6)・(n+3)  1/6をかっこ外へ
=1/18(n+3)

http://homepage3.nifty.com/sumida-3/


10337.Re: 教えて下さい。
名前:ヨッシー    日付:9月30日(火) 17時27分

(1) は、分子分母を2で割っているだけです。
(2) は、1/3 と 1/6 の掛け算です。

なお、この問題は、題意から、式を推測できますが、
上のような書き方では、

のように見えます。
正確に伝えるには、(1)は、
 (1/6)(2n+4)=(1/3)(n+2) とか、いっそ式が変わっても、
 (2n+4)/6=(n+2)/3 と書くべきですし、
(2)は、
 (1/3){(1/6)n+1/2}=(1/18)(n+3) とか、
 (n/6+1/2)/3=(n+3)/18
のように書くべきです。こうすれば、どんな見方をしても、2通り以上の
読まれ方はされません。
 
http://yosshy.sansu.org/


10369.Re: 教えて下さい。
名前:メダカ(中1年)    日付:10月2日(木) 14時56分
ヨッシーさん、Bobさん、説明ありがとうございました。

10325.問題わかりません  
名前:ヤッス(高3)    日付:9月29日(月) 22時24分
y=x^2上にP(a-1,(a-1)^2),Q(a+1,(a+1)^2)をとる。
aが-1から1まで動くとき、線分PQが描く部分の面積を求めよ。

どうも、よくわかりません、ご教授お願いします、



10328.Re: 問題わかりません
名前:ケロ    日付:9月30日(火) 0時0分
直線PQの方程式を求める。
aについての二次方程式が出来る。判別式≧0。ここで、グラフを描いてみる。
y=x^2のグラフと、判別式=0のグラフと、a=-1とa=1のときの直線PQのグラフ。
じっと眺めていると、どこを積分すればいいかがわかる。
と思います。


10329.Re: 問題わかりません
名前:ケロ    日付:9月30日(火) 0時31分
ちょっと付け加えます。
直線PQは判別式=0のグラフの接線になります。
重根になることで解ります。


10347.Re: 問題わかりません
名前:ヨッシー    日付:9月30日(火) 23時8分
とりあえず、こんなの

等積変形すれば、多少は楽に?
 
http://yosshy.sansu.org/


10361.Re: 問題わかりません
名前:ケロ    日付:10月1日(水) 22時9分
直線PQの方程式は、y=2ax-a^2+1。これをaに関して平方完成すると、
(a-x)^2=x^2-y+1≧0 となるので、y≦x^2+1 。これが図の上の曲線。
一方、直線PQは、x^2+1=2ax-a^2+1 から、(x-a)^2=0 だから、
y=x^2+1に常に接している。とくに、
a=-1のとき、y=-2x、 a=1のとき、 y=2x だから、
y=x^2との接点はそれぞれ、(-1,2)と(1,2)。
あとは積分するだけ。
(真ん中はヨッシー師匠の等積変形なら長方形の面積)。
だと思います。

10318.すいませんが・・暇があったらのでいいので・・  
名前:IGA    日付:9月29日(月) 17時6分
内接円の定理について教えてくれませんか?
実は・・
学校で削除されちゃってならわないんです。
しかしちょっとその定理を使わないと解けない問題があるので・・
一応参考書には載っているのですが・・削除されたということで余り詳しくのってないのです。
すいません。暇があればお願いします。

※削除されたかどうかは定かではありませんが・・おそらくされたと・・

こんなむちゃくちゃな質問ですいません。こんな変な質問なんでひまがあったらでよろしいのでお願いします。

『内接円の定理』は円に内接している三角形四角形についての
決まり事だと思います。



10321.Re: すいませんが・・暇があったらのでいいので・・
名前:zyam54    日付:9月29日(月) 19時31分
ひょっとして、「接弦定理」のことではありませんか。「内接円の定理」というのは聞いたことがないのですが。接弦定理は中学校3年生の内容から高校に移りましたので、削除されたというのもうなずけるのですが・・・


10323.Re: すいませんが・・暇があったらのでいいので・・
名前:hippo    日付:9月29日(月) 20時45分
>内接円の定理
三角形の三つの角の二等分線は一点で交わり、その点から三角形の三つの辺までの距離は等しい。

ついでに
>外接円の定理
三角形の三つの辺の垂直二等分線は一点で交わり、その点からの三角形の三つの頂点までの距離は等しい。

数学関係の有名なサイトに「数学の部屋」があります。そこに内接円と外接円の性質が図で説明してありますので、直リンクを貼っておきます。
http://web2.incl.ne.jp/yaoki/gainai.htm
図を実際に動かして色々試してみてください。
http://www5d.biglobe.ne.jp/~tokkii/


10324.Re: すいませんが・・暇があったらのでいいので・・
名前:IGA    日付:9月29日(月) 21時27分
ありがとうございます。ご丁寧に・・<(_ _*)> アリガトォ
ちょっと疑問なんですが、スレッドにある番号の左にかいてある□はなんですか?
なにかいみがあるのですか?
すいません変な質問で・・


10326.Re: すいませんが・・暇があったらのでいいので・・
名前:田村 正和    日付:9月29日(月) 23時18分
IGAさん・・・いつも思うんだけどアリガトォじゃないでしょ?ありがとうございましたでしょ?
俺もチャットでよく顔文字使うけどこういうまじめなところでは顔文字極力使わないように。^^;くらいはありだけど。
で肝心な質問ですがナンバーの左の口は削除するためなどに設けられているものです。


10327.Re: すいませんが・・暇があったらのでいいので・・
名前:IGA    日付:9月29日(月) 23時45分
すいません。つい・・
単語登録してあるのを忘れてました。
今後気をつけます・・
すいませんm(_ _)m
削除のためだったのですか!
ありがとうございます。

あとまた質問なんですが「削除KEY」と「□」をどのように組み合わせて削除するのでしょうか?

変な質問なんで無視してもかまいません。
できたら教えてくださるとありがたいです。


10331.Re: すいませんが・・暇があったらのでいいので・・
名前:ヨッシー    日付:9月30日(火) 0時33分
記事を削除するには、書き込みのときに削除KEY を入力しておくことが、
前提となります。

消したい記事の□ にチェックを入れて、一番下の 削除KEY のところに、
削除キーを入力して、[削除]ボタンを押すと、消すことが出来ます。
 
http://yosshy.sansu.org/


10344.Re: すいませんが・・暇があったらのでいいので・・
名前:IGA    日付:9月30日(火) 21時32分
ありがとうございます!!!
すいませんでした。お忙しい中有り難うございます。!

10312.大数8月号について  
名前:docomo    日付:9月29日(月) 0時10分
こんばんわ。大数8月号の数列ステップアップ講座を読んでいて、わからないことがあったのでお願いします。(a^kのkは添え字です)
                    n  n
a^k=A^(k+1)-A^kの形に変形できたとすると煤∞(A^(k+1)-A^k)
k=1 k=1
=A^(n+1)-A^1
「本題」
nは自然数、a^kは上の通りとする。
恒等式 k=1/2(k+1)k-1/2k(k-1)が成り立つことより、a^k=kのとき、A^k=1/2k(k-1)とできて、
n
    婆=A^(n+1)-A^1=1/2n(n+1)となる
k=1
ここでA^k=1/2k(k-1)とかくことの意味は何ですか?また、この式は(n+2)(n-1)/2になってしまうのですが???ガウスの考えでこの公式は知っているんですが。どうかよろしくお願いします。高校一年です。



10313.Re: 大数8月号について
名前:docomo    日付:9月29日(月) 0時12分
すみません、すごく見づらくなってしまいました。


10314.Re: 大数8月号について
名前:ケロ    日付:9月29日(月) 1時32分
A^k=1/2k(k-1)とかくことの意味は何ですか?>
確認しただけだと思います。
(n+2)(n-1)/2になってしまうのですが?>
-1/2k(k-1)はk=1のとき0です。k=2と勘違いしたのではありませんか。


10330.Re: 大数8月号について
名前:docomo    日付:9月30日(火) 0時32分
前者はA^kだけを書くことに違和感を感じたので。後者は文字がいっぱい出てきたために式をよく理解していませんでした。どうもすみませんでした。

10306.(untitled)  
名前:atusin    日付:9月28日(日) 13時37分
数学の公式や定理などと発見した年や発見した人などが載っている本が何かありましたら教えてください。



10309.Re: (untitled)
名前:Sar    日付:9月28日(日) 17時23分
これなんてどうでしょう。以前図書館で斜め読みしただけなので、お望みの物かどうかは分かりませんが……

このシリーズの微分方程式と一次変換と空間図形は買いました(^^;

#なお、クリックするとeS!Booksによる紹介を開きますが、これはURLが短かったのでこのサイトを選んだだけであって、他意はありません。


10339.Re: (untitled)
名前:atusin    日付:9月30日(火) 17時41分
ありがとうございました。この本探してみます。

10288.相加・相乗平均?  
名前:yuu    日付:9月27日(土) 22時48分

3a+a/3の最小の値はどうなるのでしょうか?
よろしくお願いします



10290.訂正
名前:yuu    日付:9月27日(土) 22時49分
 

3a+3/aの最小の値はどうなるのでしょうか?
よろしくお願いします


10291.Re: 相加・相乗平均?
名前:ヨッシー    日付:9月27日(土) 22時52分
タイトルからして、(相加)≧(相乗) を使うのでしょう。
ということは、a>0ですね。
相加平均・相乗平均の関係
 a+b≧2√(ab)
より、
 3a+3/a≧2√(3a・3/a)
とすればいいでしょう。
等号は3a=3/aのときなので、これが、最小のときです。
 
http://yosshy.sansu.org/


10292.Re: 相加・相乗平均?
名前:yuu    日付:9月27日(土) 23時17分
ということは
a=3となるのでしょうか?


10294.Re: 相加・相乗平均?
名前:ヨッシー    日付:9月27日(土) 23時20分
a=3 を
 3a=3/a
に当てはめると、
 9=1
になりますね。
 3a=3/a
の両辺にaを掛けて
 3a2=3
 a2=1
です。
 
http://yosshy.sansu.org/


10295.Re: 相加・相乗平均?
名前:yuu    日付:9月27日(土) 23時48分
ありがとうございました


10296.Re: 相加・相乗平均?
名前:とも(高3)    日付:9月28日(日) 1時31分
この問題は 今日のセンター模試の問題だと思うんですが、「相加・相乗平均」を使う問題だったんですかぁ・・・・
a>0 で1桁の自然数が入るようになっていたので、「最小値はa=1のときで6だろこれ」とレベルの低い考え方でやってしまいました。勉強になりました<m(__)m>

正直、数学UBは難しかった・・


10305.Re: 相加・相乗平均?
名前:ジャグラ 高3    日付:9月28日(日) 13時18分
3a+3/aと言うのを見ただけで、あぁセンター型の模試やったんだねって
分かりました(笑)俺もなんとかそこは相加、相乗使って導けましたが、
時間に追われる形で解いてく事になるのでなかなか余裕ないっすよね;
U、Bは難しかったよね〜^^;

10287.不等式  
名前:otoma高1    日付:9月27日(土) 22時35分
aは正の定数とする。x,yが正の数のとき、つねに不等式
 √(x^2+ay^2)≧(3x+4y)/√17・・・@
が成り立つためのaのとりうる値の範囲はa≧□であり、
a=□で@の等号が成り立つとき、x/y=△である。

□と△を求めたいです。
@は両辺とも正なので、2乗すればいいと思うのですが、
それから先が分かりません。
よろしくお願いします。



10297.Re: 不等式
名前:ケロ    日付:9月28日(日) 1時35分
‘aの項だけを左辺へ。両辺をy^2で割る。
x/y=tとおき、右辺でf(t)のグラフを考える(t>0)。平方完成する。
軸がマイナスになる。ので、f(t)>f(0)。だから、f(0)≧右辺。
だと思います。


10307.Re: 不等式
名前:ケロ    日付:9月28日(日) 14時43分
失礼。
軸がマイナスになる。→ならない。t=3/2で最小。f(t)≧f(3/2)。f(3/2) ≧右辺。かな?


10332.Re: 不等式
名前:ケロ    日付:9月30日(火) 1時8分
左辺と右辺がごっっーチャになってる。
最後の右辺を左辺にすればどうにかなる?


10362.Re: 不等式
名前:ケロ    日付:10月1日(水) 22時42分
上にかいた右辺と左辺を反対にしてみます。
両辺を二乗して整理すると
8(x^2-3xy-2y^2)≧-17ay^2。 両辺をy^2で割ると、
8((x/y)^2-3x/y-2)≧-17a 。ここで、x/y=t (t>0)とおくと、
8(t^2-3t-2) ≧-17a 。
f(t)= t^2-3t-2 = (t^2-3/2)^2-17/4 だから、
f(t)の最小値はt=3/2で-17/4 。従って、
8*(-17/4)≧-17a 。よって、a≧2。
等号が成り立つのは、x/y=3/2 のとき。
だと思います。


10372.Re: 不等式
名前:otoma高1    日付:10月2日(木) 18時10分
返事の書き込みが遅れてゴメンナサイ。
何回もレスしてくださってありがとうございます。
理解することができました。
ありがとうございました!

10285.確率  
名前:高校2年です    日付:9月27日(土) 22時21分
黒の碁石がM個 白の碁石がN個はいっている袋から石を取り出していく
{このとき取った石は戻さない}全て取り出すまで繰り返しその途中
常に「取り出した黒」>「取り出した白」となっている確率はいくらか?
          =

すいませんがこの問題を教えてください



10308.訂正
名前:高校2年です    日付:9月28日(日) 17時8分

=の誤りです。まちがっていてすいませんでした


10316.Re: 確率
名前:キューダ    日付:9月29日(月) 16時52分
例えば、黒4個、白2個だと、
●●●●○○  ●●●○●○  ●●○●●○  ●○●●●○
●●●○○●  ●●○●○●  ●○●●○●
●●○○●●  ●○●○●●
の9通りあるが、第1及び第7のように取り出される確率は
(4/6) * (3/5) * (2/4) * (1/3) * (2/2) * (1/1) = 4! * 2! / 6!
(4/6) * (2/5) * (3/4) * (2/3) * (1/2) * (1/1) = 4! * 2! / 6!
で、他の場合も同じく 4! * 2! / 6!である。
一般の黒M個、白N個の場合も同様で、ある特定の取り出し方が起こる
確率は全て、M! * N! / (M+N)! = 1/C(M+N,M)
従って、求める確率は、条件「常に(黒の数)≧(白の数)」を満たす
ような取り出し方が何通りあるかを数え、それと上の積を取ればよい。


条件を満たす取り出し方は、下図に於いてSからGへのルートのうち、
「ベースライン」を下回らないルートに対応させることができる。
(黒を引くと“/”、白を引くと“\”をたどる事とする。)

もちろん、SからGへの全ルート数は、(M+N)! / (M! * N!) だが、これは、
「ベースライン」の下を通過するルート「違反ルート」も数えている。

・・・・・・・・・・/\
・・・・・・・・・/\/\
・・・・・・・・/\/\/\
・・・・・・・/\/\/\/\
・・・・・・/\/\/\/\/\
・・・・・/\/\/\/\/\/\G
・・・・/\/\/\/\/\/\/
・・・/\/\/\/\/\/\/
_S/\/\/\/\/\/\/______ベースライン
__\/\/\/\/\/\/_______反転軸
・・/\/\/\/\/\/
・S'\/\/\/\/\/
・・・\/\/\/\/
・・・・\/\/\/
・・・・・\/\/
・・・・・・\/

だがその「違反ルート」の数は案外簡単に数えられる。
方法は、各「違反ルート」を、「最初にベースラインを下回った地点」
(=最初に反転軸に達した地点)の前後で分断し、前半部分、つまりSから
反転軸までのルートだけを、反転軸を対称軸にして反転させた「違反ルート†」
と言うものを考えると、全「違反ルート†」は、S’から、Gまでの全ルート
と一致することに注目するというものである。
つまり、((M+1)+(N-1))! / ((M+1)! * (N-1)!) 通りあることが解る。

従って、条件を満たすルート数は、 C(M+N , M) - C(M+N , M+1)
求めるべき確率は、
( C(M+N , M) - C(M+N , M+1) ) / C(M+N , M) = (M+1-N) / (M+1)
となる。


10317.Re: 確率
名前:キューダ    日付:9月29日(月) 16時55分
図は、お手持ちのエディッタなどへペーストするなどして、等幅フォントで
みてください。


10319.Re: 確率
名前:高校2年です    日付:9月29日(月) 18時56分
ありがとうございますキューダさん
こんなに難しかったとは…

ほんとにありがとうございました

10280.ほんとに今日は  
名前:呆け人    日付:9月27日(土) 21時37分
じぶんで考えてないわけではないのですが…ほんとに教えてもらってばかりですみません。いつかきっと答える側にまわりたいとおもいます
で、今回の問題なのですが、中学校のもんだいです。
妹に聞かれてしまったのですがよく説明できませんでした。

y=4/3x-3をx軸方向に−2移動させたグラフをもとめよ
という問題で、-2移動するから変化するyは(-2)×4/3ではないのかという質問に、グラフを書いてyは上にあがることを説明したのですが
グラフを書かなきゃわからないの?ときかれてしまい
+まだよくわからないという返答が帰ってきました。
僕もちゃんと説明できるようになりたいので、
よりわかりやすく教えていただけたらとおもいます。



10284.Re: ほんとに今日は
名前:田村 正和    日付:9月27日(土) 22時14分
>y=4/3x-3をx軸方向に−2移動させたグラフをもとめよ
これはグラフを書かないと理解させにくいでしょう。逆にグラフを書かないとしたらそれは問題を解きなれてるからです。
で肝心な答えですがy=4/3(x+2)−3です。
私は簡単なグラフを考えてたとえばx2
これをx軸方向に−2移動させたら(x+2)2となり
ちゃんと−2を代入したらx2のf(0)の値と一致しているなと確認して理解しました。


10289.Re: ほんとに今日は
名前:あいこ(高1)    日付:9月27日(土) 22時49分
こんばんは★いいお兄さんですね!!わたしは高校で習ったy=a(x−p)+qの公式で理解しましたよ!!
この公式では頂点が(p,q)。y=3/4x−3っていうのは、つまり公式で言うとp=0かつq=-3のとき、ようは頂点が(0,−3)になるんですね。しかしこれは2次関数の話でこの(0,−3)は一次関数においてはy軸との切片にあたります。
なので二次関数のときと同様に、(よく二次関数にもx軸方向に○、y軸方向に□平行移動するっていう問題とかありますよね?)一次関数も頂点!いや切片!に移動する数をたすんですね。(0+2,3)みたいな...
しかし中学生はきっと今の説明は分かりませんね;にしても、あの一次関数の問題は紙にグラフを書かなくとも頭の中にでも描かなくては解けませんね;
簡単なえがき方は、まずy軸との切片を考える!
切片が(0,3)だから...これをx軸方向に−2平行移動 だからy軸との切片は (−2,3) y=3/4x−3を平行移動だから xの係数は変わらない xの係数が3/4で切片が(−2,3)のグラフを求めればいいんだ!!!!

って具合ですかね;わかりにくくてごめんなさい;


10299.(untitled)
名前:呆け人    日付:9月28日(日) 8時21分
僕自身も田村さんのように考えていました。高校分野での求め方
(数1の2次関数の平行移動、数2の図形と方程式あたり?)
を中学校の教科書に載せていたらなーと時々思っていました。
あいこさんの方法では最初のグラフの切片となっている点を固定したまま
動かして考えるということですね。なるほど…

お二人ともありがとうございます。今度はうまく説明できるとおもいます。もう一度説明してみて、わかってくれたら高校でのグラフの話しを織り交ぜてみたいなあ(大技?)

10268.頼みます  
名前:IGA    日付:9月27日(土) 19時35分
Original Size: 925 x 443, 17KB

四角形ABCDは平行四辺形である。
AD//EF

の条件なんですけど・・・
AH:HC=AB:FC
になるわけがわかりません。

すいません。教えてください。



10273.Re: 頼みます
名前:中川 幸一    日付:9月27日(土) 20時27分
△ABH ∽ △FCH
となることは分かりますか?
そのことが証明できれば明らかになります。

http://www3.ezbbs.net/01/k-nakagawa/


10276.Re: 頼みます
名前:IGA    日付:9月27日(土) 20時54分
ああ!
有り難うございます。
定理ばかりならってたので、相似な図形を見つけるということを忘れてました〜<(_ _*)> アリガトォ

10265.教えてください  
名前:高校1年生    日付:9月27日(土) 17時40分
2個のさいころを同時に投げる試行を考える。少なくとも1個は
1の目が出るか、または出る目の和が奇数であるような事象の確率
を求めよ。

この問題を教えてください!お願いします



10266.Re: 教えてください
名前:ヨッシー    日付:9月27日(土) 18時23分

こういう図を描くか、思い浮かべて数えればいいです。
さいころ2個の場合、この36マスの図が基本になります。
答えは、23/36 です。

また、奇数と偶数は同数ずつなので、
奇数の数(18通り)+偶数で1を含むもの(5通り)=23通り
としても出来ます。
 
http://yosshy.sansu.org/


10303.教えてください
名前:高校1年生    日付:9月28日(日) 10時42分
わあー。びっくりしました。どうもありがとうございます!!
凄く分かりやすかったです。

10258.重複順列と重複組み合わせ  
名前:    日付:9月27日(土) 12時3分
こんにちは。重複順列と重複組み合わせの違いを教えて下さい。どういう時に「重複順列」で、どういう時に「重複組み合わせ」を使うのですか?宜しくお願いします…。



10320.Re: 重複順列と重複組み合わせ
名前:帰ってきた赤猫(旧 : Red cat)    日付:9月29日(月) 19時6分
その前に「順列」と「組み合わせ」の違いは理解していらっしゃるでしょうか?

10255.証明教えてください  
名前:図形大嫌い(>_<)    日付:9月27日(土) 9時51分
三角形ABCの辺BCの中点をDとする時、
       AD<1/2(AB+AC)
である事を証明せよ。
という問題なんですけど・・・証明がかなり苦手なので教えてください(涙)
余弦定理とか使えばいいのかな。。。
ちなみに大学生です・・・・(=_=;



10256.Re: 証明教えてください
名前:花パジャ    日付:9月27日(土) 10時35分
線分ADを点D側に延長して、ED=DAとなる点を考えます
△EBAは点Dを中心として△ABCを180°回転した図形、点対称です
で、
 AE=2*AD
 AE>AB+AC
です


10275.Re: 証明教えてください
名前:ケロ    日付:9月27日(土) 20時29分
花パジャ師匠は、
三角形の一辺の長さは他の二辺の長さの和より短い。
を使っています。△EBA→△ECBかな。
解答解説係。


10279.Re: 証明教えてください
名前:花パジャ    日付:9月27日(土) 21時36分
あ...その通りです < △ECB


10310.ありがとうございます!!
名前:図形大嫌い(>_<)    日付:9月28日(日) 19時26分
大学生なのに教えていただいてホントありがとうございます!!
ほんとに証明大嫌いなんです・・・(涙)
もいっぺん作図してヒントを参考に考えてみます!!

10254.(untitled)  
名前:呆け人    日付:9月27日(土) 9時40分
まいどまいどすみませんが
P(3t/2(t2+1),3t2/2(t2+1))
でtが変化するときPの描く軌跡を求めよ。
というもんだいで、答えは
x2+(y-3/4)2=9/16の円、点(0,3/2)を除く
でした。除いた点の求め方がわかりません。



10257.Re: (untitled)
名前:我疑う故に存在する我    日付:9月27日(土) 11時28分
ヒント。
>P (3t/2(t2 + 1), 3t2/2( t2+ 1)) の x 座標が 0 なら、 t = 0 だから y 座標も 0, 従って 点 (0 ,3/2) は除かれる・・・と言うだけでは不完全です。


10260.Re: (untitled)
名前:呆け人    日付:9月27日(土) 12時56分
あ、やっぱり…そう考えて違うよなーと思い直して…
うーん


10261.Re: (untitled)
名前:我疑う故に存在する我    日付:9月27日(土) 14時5分
先ず x^2 + (y - 3/4)^2 = 9/16 を満たす事は直接計算で分る。
次に f (t) = 3t/2(t^2 + 1) の増減を調べ、 f (t) の値の取りうる範囲を調べる。
最後に t が 0 -> ∞, 0 -> - ∞ のときの 3t^2/2(t^2 + 1)) 挙動を調べれば、除く点が (0, 3/2) だけでよい事が分る。


10271.Re: (untitled)
名前:ケロ    日付:9月27日(土) 19時57分
別解。
‘x=3t/2(t2+1),y=3t2/2(t2+1) から、tを消去します。
‘x=0のとき、(0,0) 。
‘x≠0のとき、y/x=tだから、これを代入して、円の式。
‘x=0のときは(0,0)だけだから、点(0,3/2)は除く。


10272.Re: (untitled)
名前:呆け人    日付:9月27日(土) 20時16分
ケロさん、我疑う故に存在する我さん、レスありがとうございます。

我疑う故に存在する我さん
答えの方はそこまで踏み込んではいなかったようなのですが
やはり書くべきでしょうか(ものぐさですみません)。


10278.Re: (untitled)
名前:ケロ    日付:9月27日(土) 21時32分
あれっ?それ自体が問題だったのか。
でも、y/xを考えると、と入れればいいのかな?


10281.tの変化に伴う点の動きを追う為の蛇足
名前:花パジャ    日付:9月27日(土) 21時44分
t=tanθ と置くと -π/2 < θ < π/2 で
x=(3/4)*sin2θ , y=(3/4)(1-cos2θ) となる
ここで -π < 2θ < π であって
件の点は、2θ=2π(または -π) に対応する点


10286.Re: (untitled)
名前:ケロ    日付:9月27日(土) 22時29分
なるほど。勉強になります。2π→πかな。
パジャ師匠入力訂正係。


10300.Re: 知りたいっ
名前:呆け人    日付:9月28日(日) 8時38分
花パジャさんの最後の部分だけわかりません
2θ=π、−πに対応する点…はてな
tanでおくなんてステキですね!
不定積分でもa2+x<sup>2 がでてきたときtanでおいてましたし…
ぜひ理解したいのでもう少し教えてください。


10301.Re: (untitled)
名前:呆け人    日付:9月28日(日) 8時38分
あららa<sup>2+x<sup>2です


10302.Re: (untitled)
名前:呆け人    日付:9月28日(日) 8時40分
…すみません。タグa2+x2これでどうだっ


10304.蛇足の蛇足で、tについての検討
名前:花パジャ    日付:9月28日(日) 10時43分
解から点Pを出すような感じで...

点C(0,3/4)を中心とし点A(0,3/2)を通る円を考える(解で出てくる円です)
x軸上の点B((3/2)t,0)と点Aとを結んだ線と件の円との交点をPとする
∠BAC=θと置くと、t=tanθであり
原点をOとすると、∠OCP=2θ

...で、上記の点Pをtで表すと、問題の式になったりなんかしちゃったりします

円上の点を線上の点と1対1対応させようとしたけど、1点だけ対応がつかない、て感じ
もしくは、無限遠というのを1点と捉えれば対応OK?


10311.Re: (untitled)
名前:呆け人    日付:9月28日(日) 21時21分
すごいなー、わかりました。たぶん自分だけでは到底こういう考え方に
行きつかないので(苦笑)改めて数学おもしろいとおもいました。
みなさま本当ありがとうございます!!!

10244.組み合わせと確率  
名前:ayaka    日付:9月26日(金) 23時50分
確率が苦手で、これは解説も答えもあるのですが、解き方がよく理解できませんでした…。
赤球4個と白球5個入っている袋の中から3個の球を取り出すとき、次の確立を求めよ。
(1)2個が白球で1個が赤球である確率。
(2)3個とも同じ色である確率。
(3)少なくとも1個は赤球である確率。
すみませんが詳しく教えて下さい…。宜しくお願いします。



10251.Re: 組み合わせと確率
名前:IF    日付:9月27日(土) 0時18分
とりあえず、すべての球が区別できるとして、すべての取り出し方を求めます。  9C3通り
白から2個、赤から1個取り出す場合、取り出し方は、5C2×4C1通り
あとは割るだけです。 
残りの問題も同じ要領でやってみてください。

10239.「場合の数」  
名前:ミカン    日付:9月26日(金) 21時45分
こんばんは。次の問題の解き方が分かりません…。解説を見てもどうしてそうなるのか分かりませんでした。教えて下さい(>ー<)宜しくお願いします。
(1)108の正の約数の個数を求めよ。
(2)108の正の約数の和を求めよ。



10241.Re: 「場合の数」
名前:IF    日付:9月26日(金) 23時6分
108=2^2・3^3であり、108の約数は
 2^X・3^Y  (X=0,1,2 Y=0,1,2,3)
の形で表されます。例えば、約数の1つである36は2^2・3^2
だから約数の個数はこのXとYの組み合わせの個数で決まります。


10242.Re: 「場合の数」
名前:帰ってきた赤猫(旧 : Red cat)    日付:9月26日(金) 23時9分
(1) まずは 108 を素因数分解してみましょう。
ここでは例を挙げて説明します。

ex) 18 の正の約数の個数
18 = 2 * 32 = 21 * 32
なので、18 の正の約数の個数は
20 * 30 = 1
20 * 31 = 3
20 * 32 = 9
21 * 30 = 2
21 * 31 = 6
21 * 32 = 18
の 6 個。

(2) 有名な公式もありますが、直に足しても大丈夫でしょう。


10243.Re: 「場合の数」
名前:ミカン    日付:9月26日(金) 23時41分
IF 先生、帰ってきた赤猫先生、どうもありがとうございます。
良く分かりました(^▽^)答えも出せましたが、
(1)は解説は(2+1)(3+1)=12という計算をしているのですが、(2+1)(3+1)という計算式で、なぜ答えが出るのですか?宜しくお願いします。


10249.Re: 「場合の数」
名前:中川 幸一    日付:9月27日(土) 0時9分
18=2×32

(20+21)(30+31+32)
=2030+2031+2032+2130+2131+2132
=1+3+9+2+6+18

これを見て何か気付きませんか?

PS IF 先生、帰ってきた赤猫(旧 : Red cat)先生 出しゃばってスミマセン。

http://www3.ezbbs.net/01/k-nakagawa/


10252.Re: 「場合の数」
名前:帰ってきた赤猫(旧 : Red cat)    日付:9月27日(土) 0時18分
>(2+1)(3+1)=12
108 = 22 * 33
なので、
・「2」が「22」の右肩の「2」
・「3」が「33」の右肩の「3」
のことである、ということは、IF さんの説明からお分かりになると思います。

「+ 1」については、108 の約数が
2x * 3y
の形をしていて、それぞれ x = 0 のとき、y = 0 のときも考えなくてはいけないので、その分の + 1 個、ということです。

(2) については、中川さんのヒントをもとに「有名な公式」を考えてみてください。


10259.Re: もう少しお願いします。
名前:ミカン    日付:9月27日(土) 12時42分
「2」が「2^2」の右肩の「2」
「3」が「3^3」の右肩の「3」を用いてなぜ答えが出るのですか???
何度もすみません…。宜しくお願いします。


10269.Re: 「場合の数」
名前:ケロ    日付:9月27日(土) 19時35分
帰ってきた赤猫師匠と中川幸一師匠の最初のレスは「18の約数の数を求めよ」という問題の解き方です。
簡単な例を挙げて、ヒントとしたのです。また、
IF師匠と二回目の帰ってきた赤猫師匠のレスは「108の正の約数の個数を求めよ。」の解法です。

108 = 22 * 33>
108=2×2×3×3×3なので、約数は2が2個、3が3個を組み合わせたもの。
2が0個、1個、2個で3通り。これが2+1=3。
3が0個、1個、2個、3個で4通り。3+1=4。
だから、3×4=12。12通り。


10274.Re: もう少しお願いします。
名前:ミカン    日付:9月27日(土) 20時28分
ケロ 先生、どうもありがとうございます。(^▽^)「2」が「2^2」の右肩の「2」「3」が「3^3」の右肩の「3」を用いてなぜ答えが出るのかは分かりましたが、計算式の3×4=12。12通りの「×」が分かりません。なぜ掛け算ですか?何度もすみません。宜しくお願いします。


10277.Re: 「場合の数」
名前:ケロ    日付:9月27日(土) 21時13分
    2の個数 0   1   2
3の個数
   0    00  01   02
   1    10  11   12
   2    20  21   22
   3    30  31   32

たて×横。で、どうですか。


10282.Re: 「場合の数」
名前:ミカン    日付:9月27日(土) 22時0分
こんばんは。ケロ 先生、納得できましたっ(^▽^)とっても分かりやすく教えてくださってありがとうございました。


10283.お願いします…。(><;)
名前:ミカン    日付:9月27日(土) 22時13分
こんばんは。(2)108の正の約数の和を求めよ。の問題で、

>「有名な公式」を考えてみてください。

考えてみたのですが思いつきません…。すみませんが、教えていただけないでしょうか。


10293.Re: 「場合の数」
名前:ケロ    日付:9月27日(土) 23時19分
有名な公式は、
中川幸一師匠のレスそのものです。(18の正の約数の和を求めよ。)
ノートに写して考えてください。書き直すと、
(1+2)(1+3+3×3)
=(1×1)+(1×3)+(1×3×3)+(2×1)+(2×3)+(2×3×3)
=1+3+9+2+6+18   
そのあと、108の正の約数の和を求めよ。を自分で考えてみましょう。
もしすぐにわからなくても、毎日考えていれば、
いつかわかる日が来ると思います。


10298.Re: 「場合の数」
名前:ケロ    日付:9月28日(日) 2時1分
等比数列。習ったのかな?もし習ってないなら、
こういう問題があるとします。
1+3+3×3+3×3×3+3×3×3×3+3×3×3×3×3=答 …(1)
両辺に3を掛けます。
(1+3+3×3+3×3×3+3×3×3×3+3×3×3×3×3) ×3=答×3
1×3+3×3+3×3×3+3×3×3×3+3×3×3×3×3+3×3×3×3×3×3=答×3  …(2)
(2)−(1)
3×3×3×3×3×3−1=答×2
両辺を2で割ります。
(3×3×3×3×3×3−1)÷2=答。
この考えで公式を作ります。

10237.(untitled)  
名前:呆け人    日付:9月26日(金) 20時52分
f(x)=x3-6kx2+12kxがx≧4で単調増加であるとき
kの取り得る範囲を求めよ。
というもんだいで
x≧4のときf'(x)≧0であることが条件(ここまではわかります)。
2k<4すなわちk<2のとき求める条件はf'(4)≧0
k≧2のとき求める条件はf'(2k)≧0

f(x)を基本形にして、軸の値と4で場合分けをしてるのも、
f'( )のカッコの中の値は各範囲での最小値であるというのもわかるのですが
しかしながらなぜ(最小値の微分係数)≧0としているのかがわかりません。
僕はf(2k)≧0、f(4)≧0と思っていたのですが…



10238.Re: (untitled)
名前:呆け人    日付:9月26日(金) 20時54分
ダッシュがみにくかったかも


10240.Re: (untitled)
名前:帰ってきた赤猫(旧 : Red cat)    日付:9月26日(金) 23時4分
>なぜ (最小値の微分係数) ≧ 0 としているのかがわかりません
うーん、正確には
(微分係数の最小値) ≧ 0
ですが…。

要は、単調増加という性質は微分係数のみによって決まるもので、そのときの関数の値は関係ないので、
>f(2k) ≧ 0、f(4) ≧ 0
といったものは、単調増加の条件にはならない、ということです。


10253.Re: (untitled)
名前:呆け人    日付:9月27日(土) 9時35分
(最小値の微分係数)ではなくて(微分係数の最小値)
ということだったんですね。それならわかりました。
微分係数の最小値≧0なら、ほかのとこも単調増加でOKだからですね。
ありがとうございました!

10230.お願いします  
名前:高校1年生    日付:9月26日(金) 17時17分
最小値・最大値の問題です。

1 2X+Y=1のとき、X^2+Y^2の最小値を求めよ。

2 X+2Y+3=0のとき、XYの最大値を求めよ。

やり方もわからないので教えてください。



10231.Re: お願いします
名前:ヨッシー    日付:9月26日(金) 18時11分
グラフを使う方法などもありますが、これらの問題は、円とか双曲線で、
座標を読むのも、たぶん難しいので、結局は式だけで解くことになるでしょう。

さて、基本は、どちらかを x= または y= の形に変形して、もう一方に代入する、です。
2次式になるので、あとは、2次関数の最大、最小の考え方を使います。

例えば 1 は、
 Y=1−2X を X^2+Y^2 に代入して、
X^2+(1−2X)^2=5X^2−4X+1
  =5(X−2/5)^2+1/5
といった具合です。
 
http://yosshy.sansu.org/


10235.Re: お願いします
名前:帰ってきた赤猫(旧 : Red cat)    日付:9月26日(金) 20時22分
Original Size: 611 x 611, 10KB Original Size: 611 x 611, 14KB

解法についてはヨッシーさんの解説で尽くされていますが、酔った勢い(マテ)でこんなものを作ってみました。目でお楽しみください。

#GRAPES、いい感じです。



10245.お願いします
名前:高校1年生    日付:9月26日(金) 23時51分
すごく分かりやすかったです。どうもありがとうございまーす☆


10262.Re: お願いします
名前:我疑う故に存在する我    日付:9月27日(土) 14時23分
>Red cat さん。
酔った勢いであまり書き込まないように。


10315.Re: お願いします
名前:帰ってきた赤猫(旧 : Red cat)    日付:9月29日(月) 10時54分
>酔った勢い
失礼いたしましたm(_ _)m

#理解の一助になれば、と思い、幾何的なイメージをグラフ化してみた
#のですが、本問では必要なかったかも知れませんね。


10322.Re: お願いします
名前:我疑う故に存在する我    日付:9月29日(月) 20時4分
グラフはあった方がよいと思いますが、それは兎も角、
私は(私も?)以前酔った勢いでメールを書いて失敗した経験がありますので。

10227.すいません。が質問です。  
名前:IGA    日付:9月26日(金) 16時56分
すいません。質問なんですけど(たいした質問ではないのにスレッドをたててもうしわけございません)m(_ _)m
過去の記事をあさってたら・・理科についての質問があったのですが・・
やはり数学と理科は同じようなものですから・・
理科についても質問してもいいのですか?
すいません。変な質問で・・・



10232.Re: すいません。が質問です。
名前:ヨッシー    日付:9月26日(金) 18時14分
まぁ、質問次第ですね。
あまり、かけ離れると、ほったらかしにされたり、
理科の質問掲示板を紹介されて終わりだったりします。
(過去の例から見て)

話が膨らむような話題が良いですね。
 
http://yosshy.sansu.org/


10233.Re: すいません。が質問です。
名前:IGA    日付:9月26日(金) 18時17分
うむ・・・
そうですか・・・
ではどなたか理科の質問掲示板を知っていらっしゃる方がいるのなら、
教えていただけるならありがたいです。


10246.Re: すいません。が質問です。
名前:味噌汁    日付:9月26日(金) 23時58分
ただgooで検索をかけただけで申し訳ありませんが…

http://bbs5.cgiboy.com/p/78/00719/
http://e-net.kir.jp/hokago/ahh/math/
http://hpcgi2.nifty.com/stand/mkakikomitai9/mkakikomitai9.cgi
http://www.html.ne.jp/south/bbs/prep/yybbs.cgi
http://www.iwakuni.org/fchem/index2.htm

すでにご自分で探し出されていたならばすみません…。
他にも探せば出てきそうですね…


10250.Re: すいません。が質問です。
名前:IGA    日付:9月27日(土) 0時17分
味噌汁さん〜♪ありがとうございます。バンザーイ!!∩(´∀`∩) (∩´∀`)∩バンザーイ!! お忙しいところありがとうございます!

今後この掲示板でも質問させていただきま〜す。

10221.わり算  
名前:こうすけ(小5年)    日付:9月26日(金) 15時21分
少数のわり算ができません。解き方を教えてしてください。おねがいします。
1÷55.85



10225.Re: わり算
名前:ヨッシー    日付:9月26日(金) 16時29分
私のページの「全国のお父さん向け ヨッシーの数学テキスト」の
第8回をご覧下さい。

0.8÷0.2 と 8÷2 が同じことであることを理解することから始めましょう。


でもこの問題、割り切れませんね。
なら、55.85=5585/100=1117/20 のように、分数にして、割り算するってことかな?
 
http://yosshy.sansu.org/


10263.Re: わり算
名前:こうすけ(小5年)    日付:9月27日(土) 14時32分
ヨッシー先生、「全国のお父さん向け ヨッシーの数学テキスト」をみて理解する事ができました。ありがとうございました。

10218.円に内接する四角形  
名前:otoma高1    日付:9月26日(金) 1時48分
円に内接する四角形ABCDがあり、AB=2,BC=3,CA=4である。
(1)cos∠ABC=?,sin∠ABC=?
(2)CD=2のとき、AD=?
(3)線分CDの長さの最大値は?
(4)四角形ABCDの面積は、CD=?のとき、最大値?をとる。


(1)cos∠ABC=-1/4,sin∠ABC=√15/4
(2)AD=4
とここまではできたのですが、この後が分かりません。
どなたかお願いします。



10220.Re: 円に内接する四角形
名前:ヨッシー    日付:9月26日(金) 12時51分
(3) CD は、この外接円の弦であり、弦の最大は直径であるときですね。
(4) △ABCは固定なので、△ACDが最大の時を探します。
 ACは長さ固定なので、これを底辺として、高さが最大となるようなDの位置を
 決めます。
 
http://yosshy.sansu.org/


10229.Re: 円に内接する四角形
名前:otoma高1    日付:9月26日(金) 17時11分
(4)なのですが、高さが最大になるときというのは、
CD=ADのときでいいのでしょうか?


10234.Re: 円に内接する四角形
名前:ヨッシー    日付:9月26日(金) 20時8分
結果としてそうですね。
べつにCD=ADを、目指して解くわけではありませんが。
 
http://yosshy.sansu.org/


10236.Re: 円に内接する四角形
名前:otoma高1    日付:9月26日(金) 20時45分
何回もスミマセン。

CD=AD=xとおいて、△ACD=(1/2)*x*x*sin∠ADCとしました。
これで△ADCの面積の最大値を求めようと思ったのですが、うまくいきません。
どこが間違っているんでしょうか?


10267.Re: 円に内接する四角形
名前:ヨッシー    日付:9月27日(土) 18時40分
その式だと x につれて△ACDも変化して、
最大も何もありません。
第一、∠ADCは保証されていますが、AC=4が保証できていません。


正弦定理から、外接円の半径はわかるので、上のような図から、
三平方だけで、面積もCDも出すことが出来ます。
 
http://yosshy.sansu.org/


10270.Re: 円に内接する四角形
名前:otoma高1    日付:9月27日(土) 19時47分
やっと解くことができました。
図まで描いてくださって、ありがとうございました。

10209.高1です 絶対値  
名前:期末中    日付:9月25日(木) 23時37分
問1
2lX+1l−3lX−1l=−2を解け
また不等式2lX+1l−3lX−1l>−2を解け
問2
3lX+1l−2lX−1l−2X=2Xを解け
また不等式3lX+1l−2lX−1l−2X=Xを解け
 どうかお願いします。絶対値が複数個出てくるとわからないんです



10211.Re: 高1です 絶対値
名前:期末中    日付:9月25日(木) 23時45分
問1の答えがX=−五分の1、7
      −五分の1<X<7
問2の答えがX=−1、三分の5
      X<−1 −1<X<三分の5
です
どうかお願いします


10212.Re: 高1です 絶対値
名前:IF    日付:9月26日(金) 0時12分
絶対値が複数個出てきた場合でも、1つずつ取り出して丁寧に場合分けすればわかると思います。

問1
  x≧1のとき    x+1>0、x−1≧0
  −1≦x<1のとき x+1≧0、x−1<0
x<−1のとき   x+1<0、x−1<0


10213.Re: 高1です 絶対値
名前:期末中    日付:9月26日(金) 0時24分
返信ありがとうございます!一度参考にしてやってみます!!

10203.点と直線  
名前:T.T.C.高2    日付:9月25日(木) 22時52分
1)kを任意の実数とする。方程式(2k−1)x+(k−2)y−3k+3=0を表す直線について
A)この直線はkが変化しても定点を通る。その座標は?
B)その定点と直線2y−ax+1=0(a>0)との距離が√2の時のaの値を求めよ。
2)平面の3点O(0、0)、A(63、0)、B(15、20)に対して三角形OABの次の座標をそれぞれ求めよ。外心、内心、垂心の出し方がわかりません。
どうかお願いします。



10205.Re: 点と直線
名前:ジャグラ 高3    日付:9月25日(木) 23時14分
A)まず展開して任意のすべてのKの値に関係なく・・・つまり
Kについての関数とそれに関係ないもう一つの関数に区別する必要が
あると思います。
K(2x+y-3)-x-2y+3=0
あとは連立させてx=y=1が導かれるかと思います。
B)これは点と直線の距離の関係により導かれますね。

2)三角形の外心は3点から等距離にある点Pを中心とし、
内心は3つの角のうち2つの2等分線の交点で求まり、
垂心はいまいち分かりません^^;


10216.Re: 点と直線
名前:Sar    日付:9月26日(金) 1時18分
Original Size: 401 x 401, 7KB

2)について。
とりあえずこんなものを作ってみました。求め方の一例を記しております。



10219.Re: 点と直線
名前:Sar    日付:9月26日(金) 2時34分
ああ、よく考えたら一辺はx軸上にあるから図のような事をしなくてももうちょっと簡単に内心は求まりますね(^^;

内接円の半径と面積、辺長の関係について、教科書などを参照下さい。

10200.積分法  
名前:ジャグラ 高3    日付:9月25日(木) 22時47分
こんばんわ、明日テストなので教科書の問題等を参考に
勉強していたのですが、疑問が生まれたので教えて頂きたいです;;

aを正の定数とする.定積分∫(a→0)√(a^2-x^2)dxを求めよと言う
問題なのですが、
x=asinθとおくと dx=acosθdθ   0≦θ≦π/2においてはと
その後続いて答えπa^2/4となるのですが、
何故x=asinθと置いてθを媒介変数とし置換積分を行ったのが今いち
分からないのですが・・・^−^;
お願いします><



10202.Re: 積分法
名前:帰ってきた赤猫(旧 : Red cat)    日付:9月25日(木) 22時51分
√(a2 - x2) を x = a sin θ とおいて変形すると無茶苦茶簡単になると思います。実は、それこそが x = a sin θ とおく理由です。


10204.Re: 積分法
名前:ジャグラ 高3    日付:9月25日(木) 22時58分
Red Catさん>
返答ありがとうございます!
なるほど、確かに簡単になることは分かりました。
ですが、いきなりasinθと置けば簡単になりそうと分かるものなの
でしょうか?^^;
コツが必要なのかな・・・それとも慣れですか;;


10207.Re: 積分法
名前:IF    日付:9月25日(木) 23時26分
y=√(a^2-x^2)とおくと、
y^2=a^2-x^2
x^2+y^2=a^2
これは円を表すので、媒介変数表示
 x=asinθ、y=acosθ
を利用しているのです。


10210.Re: 積分法
名前:ジャグラ 高3    日付:9月25日(木) 23時42分
IFさん>
返答ありがとう!!
それでasinθと置いたのですね^〜^
なるほど、納得しました。
ありがとー><


10264.Re: 積分法
名前:我疑う故に存在する我    日付:9月27日(土) 15時47分
>答えπa^2/4となるのですが、
なりません。 -πa^2/4 です。

10193.ちょっと難しい問題FOR ME  
名前:IGA    日付:9月25日(木) 17時30分
Oを原点とするxy平面上に、放物線y=1/4x^2と直線y=2x+5があり、これらの2交点をx座標の小さい順にA,Bとする。また∠OABの二等分線をl(エル)とする。

次の問いに答えなさい。

lの方程式を求めなさい。

まずAとBの座標を求めると・・
A(−2.1)B(10.25)
OAの長さは√5です(前の設問にありました)
こっからなにをしたらいいか・・・・
とりあえず二等辺三角形を作るような気がするのですが・・・
答えは良いのでヒントをくださるとうれしいです。



10194.Re: ちょっと難しい問題FOR ME
名前:帰ってきた赤猫(旧 : Red cat)    日付:9月25日(木) 17時44分
直線 AB の方程式はいうまでもなく y = 2x + 5 です。
直線 OA の方程式は y = - (1/2)x です。これはすぐにわかりますね。
この二本の直線の傾きにちゅうも〜く!!(勉強小僧風に(古))
すると ∠OAB = !?


10195.Re: ちょっと難しい問題FOR ME
名前:帰ってきた赤猫(旧 : Red cat)    日付:9月25日(木) 18時1分
Original Size: 652 x 652, 17KB

グラフを作って見たので参考にどうぞ。



10197.Re: ちょっと難しい問題FOR ME
名前:ヨッシー    日付:9月25日(木) 18時13分
角の二等分線の定理でも良いですが、ここはひとつ私の作ったこちらの問題が、
役に立ちそうです。
 
http://yosshy.sansu.org/


10198.Re: ちょっと難しい問題FOR ME
名前:IGA    日付:9月25日(木) 18時14分
なるへそ!
傾き同士をかけてー1になればその直線同士は直角であるという公式がありましたね。
うむ・・・(悲しいことに今の学校では習わないんですよね・・・削減されちゃって・・)
じゃーとりあえず角度は90度ですね・・・

ここからどうすればいいのか・・・三平方の定理を使うのですか?
ヒントだけで良いです。
PS問題文中のlの方程式を求めなさいというのがひっかかりのですが・・
方程式とは直線の式という意味なのですか?


10201.Re: ちょっと難しい問題FOR ME
名前:帰ってきた赤猫(旧 : Red cat)    日付:9月25日(木) 22時47分
∠OAB = 90°を見つけてちょっと喜んでしまいましたが、あまり解法とは関係なかったようです(-_-;)

>角の二等分線の定理
を使うと、直線 l と直線 OB の交点が求まります。2 点が決まれば、直線も決まりますよね?

>方程式とは直線の式という意味なのですか?
その通りです。


10214.Re: ちょっと難しい問題FOR ME
名前:IGA    日付:9月26日(金) 0時38分
考えてみました・・
いろいろ垂線を引いてやってみました。
そうすると・・
まず求める座標をTとします。TはAと連立させるときに使う座標。
Tの座標を(t、5/2t)とすると・・
OT=t√29/2
OB=5√29
AO=√5
AB=12√5とでてきます。※三平方の定理でxざひょうに向かって垂線をたくさん引きました。
そうすると角の二等分線の定理を使います。
√5:12√5=t√29/2:5√29-t√29/2
になりますよね?
これを解くと・・
t=10/13になります
これをだいにゅうして
T(10/13,25/13)になります。
これをAの座標と連立すると
y=1/3x+5/3となります。
この式はあり得ないような気がするのですが・・・
指摘お願いします。


10217.Re: ちょっと難しい問題FOR ME
名前:帰ってきた赤猫(旧 : Red cat)    日付:9月26日(金) 1時22分
Original Size: 611 x 611, 108KB

正解ですよ!
グラフ第 2 弾を作ってみたので、ご覧ください。



10226.Re: ちょっと難しい問題FOR ME
名前:IGA    日付:9月26日(金) 16時35分
ああ〜ありがとうございます!感激です・・・
感慨無量です。
うれしいです。
帰ってきた赤猫(旧 : Red cat)さん
ヨッシーさん
ありがとうございます。
あとケロさんも・・・
本当に感激です。
お忙しい中ありがとうございました。

10191.確率  
名前:いちご    日付:9月25日(木) 17時0分
8本のくじの中に当たりくじが3本ある。引いたくじを元に戻さないで
a,b,c3人がこの順に1本づつ引くとき、1人だけが当たる確率を求めよ。

この問題が分からないので教えてください。
期末テストに出るのですがなかなか理解できません。
引いたくじを元に戻す場合はこの問題と式はどう変わるのですか?


高校1年生



10192.Re: 確率
名前:帰ってきた赤猫(旧 : Red cat)    日付:9月25日(木) 17時29分
場合分けして考えてみましょう
(1) a だけが当たる確率
a 当たり → 確率 3/8 (8 本中 3 本当たり、5 本はずれ)
b はずれ → 確率 5/7 (7 本中 2 本当たり、5 本はずれ)
c はずれ → 確率 4/6 (6 本中 2 本当たり、4 本はずれ)
∴ 3/8 * 5/7 * 4/6 = 5/28
同様に
(2) b だけが当たる確率
(3) c だけが当たる確率
を求め、それを全部足せば O.K.

引いたくじを戻すときは、いつでも当たる確率 3/8、はずれる確率 5/8 で計算することになるので、計算式が変わります。

(1) ' a だけが当たる確率
a 当たり → 確率 3/8
b はずれ → 確率 5/8
c はずれ → 確率 5/8
∴ 3/8 * 5/8 * 5/8 = 75/512
等々。


10228. 確率
名前:いちご    日付:9月26日(金) 17時5分
ありがとうございます!来週からテストなので頑張りたいと思います。
またよろしくお願いします!!

10181.不等式の範囲の入試問題で  
名前:T.T.C.高2    日付:9月25日(木) 0時22分
1)ax2+(a2−a+2)x+2(a−1)>0(aは定数とする)の解が、1−a<x<−2/aとなるaの値を定めよ。
2)すべての実数xに対して、1/3<x2-px+p2/x2+x+1<3が成り立つpの値の範囲を求めよ。
3)a、bを正の整数とし、√2はa/bとa+2b/a+bの間にあることを示せ。
4)aを定数とするとき、関数f(x)=(a2+1)x2−4axについて
A)すべての実数xに対して(x)>−1となるためのaの条件を求めよ。
B)すべての整数xに対して(x)>−1となるためのaの条件を求めよ。
これらがわかりません。どうか助けてください。



10185.(1) だけでも
名前:帰ってきた赤猫(旧 : Red cat)    日付:9月25日(木) 9時31分
ax^2 + (a^2 - a + 2)x + 2(a - 1) = (x + a - 1)(ax + 2)
と因数分解できることに気づくと…。


10186.(2) 以降のおはなし
名前:帰ってきた赤猫(旧 : Red cat)    日付:9月25日(木) 10時22分
(2) 真ん中のは (x^2 - px + p^2)/(x^2 + x + 1) でしょうか。
分母になっている x^2 + x + 1 について、ちょっと検証してみましょう。
x^2 + x + 1
= x^2 + 2(1/2)x + (1/2)^2 - (1/2)^2 + 1
= (x + 1/2)^2 + 3/4 > 0
このことから、元の問題は

(1/3)(x^2 + x + 1) < x^2 - px + p^2 < 3(x^2 + x + 1)
が全ての実数 x について成り立つような p の値の範囲を…

と直すことが出来ます。

(3) その様な整数 a , b が存在する、という主旨なのでしょうか?

(4) (A) は簡単です。問題は (B) ですが、(B) における基本的な考え方は (A) と同じです。ただ、x が整数に限られているために、注意を必要とします。


10187.Re: 不等式の範囲の入試問題で
名前:我疑う故に存在する我    日付:9月25日(木) 13時11分
3)の解答。
c が a と b の間にある ⇔ (a - c)(b - c) < 0 (端っこも含めれば、 ≦ 0).

さて、 a, b > 0 なる前提で同値変形する。
>√2はa/bとa+2b/a+bの間にある ⇔
{ (a/b) - √2 }*{ (a + 2b)/(a + b) - √2 } < 0 ⇔
{ a^2 - 2b^2 }*{ (a + 2b)^2 - 2( a + b)^2 } < 0 ⇔
-(a^2 - 2b^2)^2 < 0.

最後の式は、 a, b が自然数なら成立。


10188.Re: 不等式の範囲の入試問題で
名前:帰ってきた赤猫(旧 : Red cat)    日付:9月25日(木) 13時27分
(3) あっ!!そういうことか。
私が妙な勘違いをしていました。


10196.おまけ
名前:我疑う故に存在する我    日付:9月25日(木) 18時7分
全く同様に 「√3 は a/bと (a + 3b)/(a + b) の間にある」事が証明できる。
又、 m, n が、 0 < n < m < 2n なる自然数(有理数)の時 、√2 は、 m/n と (2n - m)/(m - n) の間にある事が示される。

受験問題を解くばかりでなく、たまには息抜きで問題の変形・拡張などしてみたら如何?


10199.Re: 不等式の範囲の入試問題で
名前:T.T.C.高2    日付:9月25日(木) 22時27分
ありがとうございます。できました。

10171.オームの法則について  
名前:飯尾和代    日付:9月24日(水) 18時33分
2オーム3オーム6オームの抵抗を直列に接続したときの合成抵抗は、この抵抗を並列に接続したときの合成抵抗の何倍となるか。
という問題の答えと解き方を教えてください



10176.Re: オームの法則について
名前:ケロ    日付:9月24日(水) 20時23分
ええとなんだっけ。
電圧をEとする。
直列の場合。どの場所を流れる電流も同じだから、それをIとおくと、
E=2I+3I+6I=11I。
E=RIだから、11オーム。
並列の場合。それぞれの抵抗の両端の電圧は同じで、
流れる電流の大きさは違うから、それらを
I[1]、I[2]、I[3]とおくと、
E=2I[1] 、E=3I[2] 、E=6I[3] だから、
I[1]=E/2 、I[2]=E/3、I[3]=E/6。
I[1]+I[2]+I[3]=I゜とおいて、代入して整理すると、
E=I゜。
だから、1オーム。
久しぶりだなあ。どうかな。 


10178.Re: オームの法則について
名前:タイモン    日付:9月24日(水) 21時40分
抵抗については、ほとんど公式化された関係式があったはずですよね。
それを活用すると、以下の結果が導かれます。
抵抗をR1,R2,R3とする。
これらを直列に繋げたときの全抵抗Rは、
R=R1+R2+R3
これらを並列に繋げたときの全抵抗Rは、
1/R=(1/R1)+(1/R2)+(1/R3)
です。
ご自身で確認してみてください。

10162.教えて下さい!  
名前:豚カツ(高一)    日付:9月24日(水) 15時37分
3^n-1+3^n-2+3^n-3+・・・・+3+1←この等比数列の和を求めたいのですが、公比、初項は何になりますか?公比は1/3になりますよね?



10164.Re: 教えて下さい!
名前:帰ってきた赤猫(旧 : Red cat)    日付:9月24日(水) 15時43分
>3^n-1+3^n-2+3^n-3+・・・・+3+1
逆向きに考えてみると…。
1 + 3 + ... + 3^(n - 3) + 3^(n - 2) + 3^(n - 1) = ??


10167.(untitled)
名前:豚カツ(高一)    日付:9月24日(水) 17時9分
あっ!そうでした。帰ってきた赤猫サン、ありがとうございました。

10161.ありがとうございました(^^)  
名前:かずみ(社会人3年)    日付:9月24日(水) 14時48分
図まで書いていただきありがとうございました。(^^)
(イメージどおりです!)
特に名前がないということで承知しました。
またわからないことがあった時にはよろしくお願いします。

10157.(untitled)  
名前:松原妙子    日付:9月24日(水) 13時55分
ルート6と7そして8の覚え方をお願いします。



10158.ルートの憶え方
名前:帰ってきた赤猫(旧 : Red cat)    日付:9月24日(水) 14時1分
√6 ≒ 2.44949(似よ、良く良く)
√7 ≒ 2.64575(菜に虫いない)
√8 = 2√2
√2 ≒ 1.41421356(ひとよひとよにひとみごろ)
#√2 は漢字失念;;

一応、
√8 ≒ 2.8284271(ニヤニヤ、世にない)
という方法も。


10160.Re: (untitled)
名前:ヨッシー    日付:9月24日(水) 14時29分
とりあえず、こちらなどを。
(18禁だそうです^^;)
 
http://yosshy.sansu.org/

10153.図形の種類について教えてください。  
名前:かずみ(社会人3年)    日付:9月24日(水) 11時6分

昔の教科書が見当たらないので教えてください。

たとえば、頭の部分が四角錘、下の部分が四角柱になっている図形は
なんという図形になるのでしょうか。
(四角錘、四角柱、円錐、円柱、三角錐、三角柱とはどういう形のものをいうのかおしえてください。)

うまく表現ができなくて申し訳ないのですが、よろしくお願いします。



10154.Re: 図形の種類について教えてください。
名前:帰ってきた赤猫(旧 : Red cat)    日付:9月24日(水) 12時1分
Original Size: 240 x 254, 3KB

こんな図形でしょうか。名前は別にないと思います。
#もし名前があるようでしたら、遠慮なく突っ込んでくださいまし。

「注」「錘」の定義は忘れてしまいました(滝汗)。
#図学を習ってからもう 9 年も経ってるもので…。
大雑把に言うと、母線の平行移動による軌跡みたいなのが「注」で、母線の一端を固定して移動したものの軌跡みたいなのが「錘」。



10155.赤っ恥!!
名前:帰ってきた赤猫(旧 : Red cat)    日付:9月24日(水) 12時33分
>注
「柱」ですね。失礼いたしましたm(_ _)m


10159.Re: 図形の種類について教えてください。
名前:我疑う故に存在する我    日付:9月24日(水) 14時11分
あなたが単に名前を知りたいのか、それを言葉で人に伝えたいのかで回答も異なります。

注:「円」にも、円周、円板、開円板などいろいろな意味がある。「錘」にも、片側、両側(直線の集合)など、いろいろな意味がある。円錐も両側がなければ双曲線も「単曲線」になる。

10148.教えてくださいm(__)m  
名前:たく(中3)    日付:9月24日(水) 3時45分
いつもお世話になっております。いつも質問ばかりで申し訳ないです。。今まで質問に応答していただいた方、誠にありがとうございます。今回もよろしくお願いしたいと思います。

あなたは今、山登りの途中、山道の分岐点に来ています。道は左右にわかれており、一つの道は正しい登山道で、もう一つの道は「けもの道」に迷い込んでしまいます。そこにはA,Bのふたりの村人がいます。
・一人は正直者で、必ず本当のことを答えます。
・一人はうそつきで、必ず嘘をつくことがわかっています。
 ただしどちらがAでどちらがBなのか、二人の区別をすることはあなたにはできません。あなたはふたりの内どちらかに一回だけ質問をして、正しい道を見つけることができるでしょうか。ただし村人は「はい」か「いいえ」しか答えません。その質問を考えてください



10149.Re: 教えてくださいm(__)m
名前:ヨッシー    日付:9月24日(水) 6時17分
AまたはBに、道について聞いても、それぞれ違う答えが返ってくるだけです。
Aに聞いてもBに聞いても、同じ答えが返ってくる質問が必要です。

ヒントは、「質問にもう一方の人をからめる」です。
「もし、彼に『・・・』と聞いたなら〜」
という感じです。
 
http://yosshy.sansu.org/


10163.Re: 教えてくださいm(__)m
名前:たく(中3)    日付:9月24日(水) 15時38分
う〜ん、考えてみたんですが、やはりダメですね。もう少し教えてもらってもいいでしょうか?


10165.Re: 教えてくださいm(__)m
名前:帰ってきた赤猫(旧 : Red cat)    日付:9月24日(水) 16時26分
仮に「左の道は正しい道ですか?」という質問を A とします。
------------------------------
| | | 正直者 | 嘘つき |
|-----------------------------
| | 本当 | はい | いいえ |
| A |------|------------------
| | 嘘 | いいえ | はい |
------------------------------

上の表のように答えるはずです。

そこで。

B「『左の道は正しい道ですか?』と彼に聞いたら、彼は『いいえ』と答えますか?」

と質問するのです。このとき、正直者の答え方は質問 A のときと同じです。
しかし、嘘つきの答え方は逆になります。

もし左の道が正しければ、彼 = 正直者は「はい」と答えますから、B の答は「いいえ」になるはずです。
しかし、嘘つきはそれと逆のことを言うので、返事は「はい」になってしまう、というわけです。


10166.表の部分だけ書き直しです。
名前:帰ってきた赤猫(旧 : Red cat)    日付:9月24日(水) 16時28分
-------------------------
| |   |正直者|嘘つき|
|------------------------
| |本 当|は い|いいえ|
|A|------|--------------
| | 嘘 |いいえ|は い|
-------------------------


10180.Re: 教えてくださいm(__)m
名前:たく(中3)    日付:9月25日(木) 0時6分
詳しく説明していただき感謝していますm(__)mしかし、あまりぴんとこないのですが、説明していただけないでしょうか。すみません。


10182.Re: 教えてくださいm(__)m
名前:帰ってきた赤猫(旧 : Red cat)    日付:9月25日(木) 0時28分
要点は、普通に質問すると、嘘つきの答えは正直者と逆になってしまうところを、どうやってひっくり返すか、にあります。

有名な論理パズルですが、知らないとなかなか難しいものがあります。
#要は私の力不足ゆえ、これ以上の説明が思いつかない、ということです(滝汗)。

別解もあります。
「『左の道は正しい道ですか』と聞かれたら、貴方は『はい』と答えますか?」
これは、村人が一人しか立っていない(ただし、正直者か嘘つきかわからない)ときにも有効です。


10183.Re: 教えてくださいm(__)m
名前:キューダ    日付:9月25日(木) 2時39分
物事を反転させて見せるメガネと、そのまま見せるメガネがあったとすると、
村人に質問をするということは、そのどちらかのメガネをかけて、物ごとを
観ると言うことに対応させることができます。

ヨッシーさんのヒントは、

「それぞれが、どちらのメガネなのかは分からなくても、その二つのメガネ
 を重ねてかければ、必ず、反転された世界が見える」

という性質を利用するものです。

帰ってきた赤猫(旧 : Red cat) さんが別解として紹介された物は、

「どちらのメガネか分からなくても、同じメガネを二つ重ねてかけると、
 必ず、通常の世界が見える」

という性質を利用する物です。

つまり、前者は、「1×(−1)も(−1)×1も−1である。」
後者は、「1×1も(−1)×(−1)も1である。」
という事を利用すると考えれば、理解しやすいでしょう。


なお、ヨッシーさんの方法は、村人がお互いに、他方が正直者であるか
嘘つき者であるかを知っているという前提の基に成立する「質問」となります。
問題では、この点について、言及がないので、この前提を必要としない、
後者を利用した「質問」の方がベターと言えるでしょう。


10184.Re: 教えてくださいm(__)m
名前:帰ってきた赤猫(旧 : Red cat)    日付:9月25日(木) 9時17分
■キューダさん
お久しぶりです。
#「あの時」の非礼については、ひとえに私の(人間としての)未熟さ
#故のこととお許し下さい。

メガネに例えるところがいい感じです。フォローありがとうございま
す。

>ヨッシーさんの方法は、村人がお互いに、他方が正直者であるか嘘
>つき者であるかを知っているという前提の基

そうなんですよねぇ。私も解説を書きながら思ってました。

10142.順列の問題です。  
名前:あいこ(高1)    日付:9月23日(火) 22時40分
昨日に引き続き順列について質問させていただきます!以下の問題です。
4枚のカードの表と裏にそれぞれ1と2、3と4、5と6、7と8、の数字が書いてある。これらを並べて三桁の数字を作ると何通りできるか。

わたしは樹形図を書いてみました!(はたしてその樹形図の書き方があっているのか不安ですが;)百の位が1のとき十の位は3か4か5か6か7か8、一の位は...っといたような感じにやっていきましたがミスをしたのかわかりませんが答えがちがいました;解答のほうは。4枚のカードの順列の数から始めていますが、そこから始める意味も分かりません!!教えてください!



10143.Re: 順列の問題です。
名前:ジャグラ 高3    日付:9月23日(火) 23時36分
こんばんわ、まず答えは384通りであってますか?
アッテタラ説明します・・・。間違ってる可能性もあるのでーー;


10144.Re: 順列の問題です。
名前:ジャグラ 高3    日付:9月23日(火) 23時41分
P(8、3)でも384通りを越えないので
384通りじゃないですね^^;
どうすればいいんでしょ...
答えを一回教えて頂けないでしょうか?樹形図を用いずに考えている
のですが、いまいち自信がなくて。(;;


10145.Re: 順列の問題です。
名前:ヨッシー    日付:9月23日(火) 23時53分
1,2 の札をA,3,4 の札をB,5,6 の札をC,7,8 の札をDとします。
まず、表裏関係なく、A,B,C,Dのカードを3枚並べると考えます。
すると、4P3=24 通りです。
そのうちの一つの並べ方、たとえば、ABC について、
Aが表の場合と裏の場合、
Bが表の場合と裏の場合、
Cが表の場合と裏の場合、
それぞれ2通りずつ、計2×2×2=8 通りの数が作れます。

実際、135,136,145,146,235,236,245,246 の8つ作れます。

これらは、24通りそれぞれについて8通りずつ存在し、また、同じ数が
二度以上重複することはないので、単に掛け算して、
 24×8=192
です。

樹形図を書くなら、
百の位が1なら、十の位は3〜8の6通り、一の位は、残り4通り で24通り。
つまり、1から24本の枝が分かれます。
他の2〜8を百の位にしても同様なので、
 8×24=192
です。
 
http://yosshy.sansu.org/


10146.Re: 順列の問題です。
名前:ケロ    日付:9月23日(火) 23時58分
樹形図のとき、1の後、3を選んだとしますと、次に選べるのは5,6,7,8です。
2と4は1と3のカードの裏ですから使えません。
たぶん、ここで間違ったのだと思います。
4枚のカードの順列の数から始めています>
カードを先に並べたのでしょう。
並べ方は、樹形図で何通りですか。
並べ方を決めたら、裏表で2通りずつありますね。


10147.Re: 順列の問題です。
名前:あいこ(高1)    日付:9月24日(水) 0時12分
答えは192です!すみません(>_<)みなさんの説明のおかげで理解できました!!!本当に助かりました!ありがとうございます!

10138.コピー機の有効的な使い方について  
名前:田村 正和    日付:9月23日(火) 16時42分
私のバイト先のコピー機は読み込みというボタンがありまして読み込む時間は普通の出力ボタンを押したときの読み込み時間と同じなんです。ただ出力ボタンを押したときは印刷が終わるまで次に押す出力ボタンが押せないのです。しかし読み込みボタンを押した場合は読み込んだ後すぐに次の読み込みボタンが押せるのです。よく先輩に何枚か印刷するときは読み込みを使うといいよと言われるのですがどうも納得できないのです。そこで何枚以上印刷するときには読み込みボタンを押したほうが早いのか教えてほしいのです。ここでは読み込む時間をp、次に印刷するのに必要な手間(本を見開きでコピーするので本を取り上げてページを変えてまた所定の場所に戻すという操作)にかかる時間をq、出力を押して印刷されて次のボタンを押せるまでの時間をr(r>q、r=読み込みを終えたときに最後に印刷される時間の1枚分)とするとどういう条件のとき有効的な使い方ができますか?ちなみに私の考えでは読み込みだけを使う場合(簡単に2枚として)pqpqrrとなり出力だけを使う場合r>qなのでqの時間が節約されprprとなり論理的には出力だけを使ったほうがすべての場合において早いと思うのですがどうでしょうか?ついでにこのコピー機は読み込みだけを使った場合、読み込みを終えた後出力ボタンを押してすべて印刷される仕組みです。



10150.Re: コピー機の有効的な使い方について
名前:花パジャ    日付:9月24日(水) 10時9分
人間がコピー機の前に拘束されている時間が短くていいよ、という事では?
rr...rの時にはその場を離れていてもいい、という所で。
もしくは、r>qにならないようにと、コピー機に追われて仕事をするのがイヤ、とか。


10156.Re: コピー機の有効的な使い方について
名前:田村 正和    日付:9月24日(水) 13時20分
なるほど。そういう見方もありますね。
こんな長い文章を読んでいただいてどうもありがとうございます。

10135.線形空間の問題ですが、、、  
名前:アデレス    日付:9月23日(火) 15時58分
すみませんがこの問題で答えられないでいます、、、。誰かわかる方いないでしょうか?いたら答えてください。お願いします。もうぼくじゃあ答えられません、、、。大学2年です。

K上の線形空間Vとその部分空間Wがあるとする。Vの二元x、yに対し、x−y∈Wのとき、x〜yと定義すれば、〜はVの同値関係である。Vの〜による商集合をV/Wで表わし、Vの元xの含まれる類を[x]で表わす。


イ)V/Wの元[x],[y]に対し、和およびスカラー倍を、[x]+[y]=[x+y],c[x]=[cx]で定義することができること(すなわち、類の代表の取りかたによらないこと)を示し、この演算によってV/WがK上の線形空間になることを示せ。
V/WをVのWによる商空間と言う。


ロ)Wの基底E0=<e1,e2,・・・er>を含むVの基底E=<e1,e2,・・・,er,er+1,・・・,en>を取る。E~=<[er+1],・・・,[en]>はV/Wのの基底であることを示せ。(したがってdimV/W=dimV-dimWがなりたつ。)

ハ)TがVの線形変換で、WがT−不変ならばV/Wの線形変換T~を、T~[x]=[Tx]によって定義できることを示せ。



10136.Re: 線形空間の問題ですが、、、
名前:中川 幸一    日付:9月23日(火) 16時5分
そんなにマルチポストを繰り返しますと信頼を無くしますよ!!
それにこの質問はd3の学習室さんの数学質問室で解決できたのではないですか?

あと HN は極力統一させましょう。

http://www3.ezbbs.net/01/k-nakagawa/


10151.Re: 線形空間の問題ですが、、、
名前:帰ってきた赤猫(旧 : Red cat)    日付:9月24日(水) 10時20分
>マルチポスト
私の個人的な意見に過ぎませんが、
「他サイトで回答をもらって解決したのなら、その旨書き込んで欲しい」
という気持ちはあります。
#私自身は、マルチポスト自体はあまり気にしないことにしています。

回答は他サイトに詳しく書かれているようなので、参考書だけ挙げておきます。

齊藤正彦「線型代数入門」(東大出版会)

大学 2 年のレベルであれば無理なく読みこなせるものとは思いますが、わからないところがあれば遠慮なく。

10130.はじめましてw  
名前:noi    日付:9月23日(火) 11時52分
わからないので教えてくださいw
1〜6の目の出る確率が→1 2 3 4 5 6
p p q p q p
のようになるさいころがある。このさいころを一回投げるときの出る目の期待値が15/4となるようにp,qの値をもとめよ。
よろしくおねがいしますw



10131.Re: はじめましてw
名前:noi    日付:9月23日(火) 11時58分
1 2 3 4 5 6
p p q p q p
ですw


10134.Re: はじめましてw
名前:ヨッシー    日付:9月23日(火) 15時21分
1×p + 2×p + 3×q + 4×p + 5×q + 6×p =15/4
ってことですね。
これと、もう一つ式をたてて、連立方程式を解けば、
 p=1/12 q=1/3
を得ます。
さて、もう一つの式とは?
http://yosshy.sansu.org/


10137.Re: はじめましてw
名前:noi    日付:9月23日(火) 16時36分
ありがとうございました。
すごく助かりました

10124.因数分解  
名前:IGA    日付:9月23日(火) 10時6分
a^2(b-1)+b^2(1-a)+a-bを因数分解しなさい。

a^2(b-1)+b^2(1-a)+a-b=a^2b-ab^2-a^2+b^2+a-b
=ab(a-b)-(a+b)(a-b)+a-b
=abA-A(a+b)+A
=A(ab-a-b+1)
=(a-b)(ab-a-b+1)
と出たのですが・・・
答えと一致しないのです。ご指摘お願いします。



10125.Re: 因数分解
名前:田村 正和    日付:9月23日(火) 10時28分
=(a-b)(ab-a-b+1)
=(a-b)(a-1)(b-1)


10129.Re: 因数分解
名前:IGA    日付:9月23日(火) 11時47分
有り難うございます。m(_ _)m
≧(´▽`)≦アハハハ簡単な問題でした。
感謝します!!

まだ因数分解できましたね。ありがとうございます。

10120.私の答えあってるのでしょうか?  
名前:ぴぽぽ    日付:9月23日(火) 3時43分
ご指導お願いします。

問題:以下の特性を持つRelationを表せ
1:AとBはOne-to-many関係にある
2:AとBはMany-to-one関係にある
3:AとBはOne-to-one関係にある

私の答え
1: For all (a1, b1), (a2, b2) in R, a1=a2 -> b1 not equal b2
2: For all (a1, b1), (a2, b2) in R, a1 not equal a2 -> b1=b2
3: For all (a1, b1), (a2, b2) in R, a1 = a2 -> b1=b2



10152.Re: 私の答えあってるのでしょうか?
名前:帰ってきた赤猫(旧 : Red cat)    日付:9月24日(水) 11時3分
よくよく考えてみると、結構難しいかも?
1. 一対多、と言うことは、A の元に対応する B の元は複数ありますが、それを逆方向に見たとき、つまり B の元に対応する A の元は? と考えると、B の側から見たときには、A の元は一つに決まっていないと駄目です(そうでないと、多対多の対応になってしまいます)。

結論。
(∀b ∈ B , ∃!a ∈ A s.t. (a,b) ∈ R) and (∀a ∈ A , ∃b1 , b2 ∈ B s.t. b1 ≠ b2 and (a,b1) , (a,b2) ∈ R)
#自信はありませんが…。

2. や 3. も同様に考えてみてください。


10177.Re: 私の答えあってるのでしょうか?
名前:我疑う故に存在する我    日付:9月24日(水) 20時30分
>結論。(∀b ∈ B , ∃!a ∈ A s.t. (a,b) ∈ R) and (∀a ∈ A , ∃b1 , b2 ∈ B s.t. b1 ≠ b2 and (a,b1) , (a,b2) ∈ R)

厳密な定義という物はあったかな。前半部は (∀b ∈ B , (a,b) ∈ R なる a は高々1個)、後半は (∃a ∈ A , ∃b1 , b2 ∈ B s.t. b1 ≠ b2 and (a,b1) , (a,b2) ∈ R) と言う感じで捉えていたが。つまり B から A への単射でない写像 f があって、 (a, b) ∈ R ⇔ a = f (b)


10190.Re: 私の答えあってるのでしょうか?
名前:帰ってきた赤猫(旧 : Red cat)    日付:9月25日(木) 16時42分
>厳密な定義
私も良くわからないのですが、いろいろと考えてみたところ、我疑う故に存在する我さんの考え方の方が適切のような気がしてきました。
#部分的に一対一でも、別に関係ないですよね…。

10116.指摘お願いします。  
名前:IGA    日付:9月23日(火) 0時27分
astさん。arcさん。ありがとうございます。お忙しい中・・・すいません。
こういう考えっていいのですか?
(n+2)の場合
nは3〜998 
↓↓↓2を足して
5〜1000で
1000÷5をして
200個
(n+3)
nは2〜997で
↓↓↓3を足して
5〜1000で
1000÷5で200個
200+200=400
こういう考え方は果たして(・∀・)イイ!!でしょうか?
ご指摘お願いします!



10117.Re: 指摘お願いします。
名前:IGA    日付:9月23日(火) 0時28分
すいません!
これはしたのレスです。申し訳ございません。m(_ _)m失敗です。


10118.Re: 指摘お願いします。
名前:arc    日付:9月23日(火) 3時29分
私の式はそれを略して書いただけです。

考え方は間違っていません。


10122.Re: 指摘お願いします。
名前:IGA    日付:9月23日(火) 8時45分
有り難うございます!感謝します。感慨無量です!

10108.お願いいたします!M(_ _)Mペコッ  
名前:IGA    日付:9月22日(月) 22時39分
正の整数nに対して、M=(2n+3)^2+(2n+5)^2+(2n+7)^2-11とするとき次の問に答えなさい。

問い
nが1000以下の正の整数であるとき、Mが60の倍数となるnは何個あるか?その個数を求めなさい。

まったくわかりません。とりあえず式を簡単にしました。
M=12(n+2)(n+3)です。
ここからまったく・・・何からしたらいいのか?
うむ・・・・お願いいたします!



10109.Re: お願いいたします!M(_ _)Mペコッ
名前:ast    日付:9月22日(月) 23時2分
M が 60 の倍数ってことは,

  12(n+2)(n+3) = 60k (k:正の整数)

と書けるってことだから, (n+2)(n+3) が 5 の倍数ってこと.


10110.Re: お願いいたします!M(_ _)Mペコッ
名前:IGA    日付:9月22日(月) 23時27分
(n+2)(n+3)が5の倍数となればよろしいのだから、
(n+2)は3〜998

(n+3)は2〜997ですよね?

しかし・・・これを数えると莫大な時間がかかるので・・・
やはり計算で解く方法があるのでしょうか?
教えてくだされば幸いです。


10112.Re: お願いいたします!M(_ _)Mペコッ
名前:ast    日付:9月22日(月) 23時37分
掛けて 5 の倍数なんだから, どっちかが 5 の倍数なら良くて
たとえば, n+2 が 5 の倍数になるのは, 3 から 1002 まで 5 ごとに
出てくるじゃない.
数えられるでしょ?

n+2 = 5*k とでもおくと
3 ≦ n+2 ≦ 1002 だから, 3/5 < k < 1002/5
になる自然数 k は幾つあるの?

おんなじことを n+3 が 5 の倍数になるときについてもやれば,
n+2 と n+3 は同時に 5 の倍数になることはないんだから
もう全部数えたことになるよね?


10113.Re: お願いいたします!M(_ _)Mペコッ
名前:ast    日付:9月22日(月) 23時39分
もちろん 5 が素数だから
>掛けて 5 の倍数なんだから, どっちかが 5 の倍数なら良くて
その他の場合には 5 の倍数にならないって事は使っているけれど.


10115.Re: お願いいたします!M(_ _)Mペコッ
名前:arc    日付:9月23日(火) 0時9分
(2n+3)^2+(2n+5)^2+(2n+7)^2-11
=(4n^2+12n+9)+(4n^2+20n+25)+(4n^2+28n+49)-11
=12n^2+60n+72
=12(n+2)(n+3)
=M
=60x

12(n+2)(n+3)=60x
(n+2)(n+3)=5x


n=5p-2,5q-3
1≦n≦1000
mod(n,1)=0
mod(p,1)=0
mod(q,1)=0

1≦p≦200
1≦q≦200

200+200=400

n(M)=400


10123.Re: お願いいたします!M(_ _)Mペコッ
名前:IGA    日付:9月23日(火) 8時46分
ありがとうございます。
感慨無量です。
今後ともよろしくお願いします。m(_ _)mペッコッ

10103.確率  
名前:いちご    日付:9月22日(月) 21時35分
A、B、Cの3人がじゃんけんを1回するとき、次の確率を求めよ。

1 Aだけが負ける確率

2 1人だけが勝つ確率


この問題が分からないので教えてください



10107.Re: 確率
名前:arc    日付:9月22日(月) 22時17分
樹形図でいきましょう。



ABC     ABC     ABC 
                    
ぐぐぐ     ちぐぐ★    ぱぐぐ●
ぐぐち     ちぐち●    ぱぐち 
ぐぐぱ●    ちぐぱ     ぱぐぱ 
ぐちぐ     ちちぐ●    ぱちぐ 
ぐちち●    ちちち     ぱちち★
ぐちぱ     ちちぱ     ぱちぱ●
ぐぱぐ●    ちぱぐ     ぱぱぐ 
ぐぱち     ちぱち     ぱぱち●
ぐぱぱ★    ちぱぱ●    ぱぱぱ 

 

※【●は、1人勝ち】【★は、A1人負け】


※全事象33=27(通り)

Aだけが負ける確率 = 3/27 = 1/9
1人だけが勝つ確率 = 9/27 = 1/3



結果を分かりやすくしただけなので、答えを求めるだけなら、

◆Aだけが負ける = Bには負けている + Cには負けている
BがAに勝つ確率=1/3
CがAに勝つ確率=1/3
同時に起こるので1/3*1/3=1/9

◆1人だけが勝つ = Aだけが勝つ×3
AがBに勝つ確率=1/3
AがCに勝つ確率=1/3
同時に起こるので1/3*1/3=1/9
3人のうち誰でもいいので1/9*3=1/3


※【1人が勝つ=1人が負け】【Aが勝つ=Aが負け】【1人が勝つ=Aが勝つ×3(人数)】


10127.確率
名前:いちご    日付:9月23日(火) 11時23分
すごく分かりやすかったです。どうもありがとうございます♪

10094.角の二等分線定理  
名前:味噌汁    日付:9月22日(月) 19時9分
こんにちは。角の二等分線定理で、

△ABCの頂角Aまたはその外角の二等分線は、対辺BCを他の二辺の比に内分または外分する。

というのがありますが、外角の二等分線の場合は、
△ABCの頂角Aの外角の二等分線とBCの延長上の
点をEとした場合、
BE:CE=AB:AC
ということで理解して大丈夫でしょうか?

あと、内角の場合も頂角Aの内角の二等分線とBCの
交点をDとすると、
BD:DC=AB:AC
ということで理解して大丈夫でしょうか?

ちょっと不安なので、教えてください。
よろしくお願いします。



10102.Re: 角の二等分線定理
名前:ケロ    日付:9月22日(月) 21時15分
これは教科書を見たほうがいいと思います。
または検索。「三角形 外角の二等分線」で。例えば、
http://homepage2.nifty.com/tangoh/skikamenu01.html


10104.Re: 角の二等分線定理
名前:味噌汁    日付:9月22日(月) 21時40分
わざわざどうもありがとうございます。
理解できました。

10091.順列の問題です☆  
名前:あいこ(高1)    日付:9月22日(月) 18時11分
男女各4人の8人が男女交互に円卓の回りに座る方法は何通りあるか。
っという問題なのですが、
 男の座り方が (4−1)!
 女の座り方は(4−1)!
っと考えたのですが、解答は 女の座り方は 4!でした;なぜでしょうか?



10092.Re: 順列の問題です☆
名前:ジャグラ 高3    日付:9月22日(月) 18時45分
分かりやすく説明することができないかも知れませんが...
まずポイントと思う所は軸を定める事だと思います。
あいこさんがおっしゃってる考え方は、男の子4人と女の子4人で
男だけで円卓の周りに座る事象と女の子4人で座る事象について考えている
と思われます。
では、どうすれば男女交互に座る通りになるかと言いますと
男子でも女子でも構いませんのでまず、円順列と見て(4−1)!通りで
それに●○●○●○●○● ●が何通りあるかと言いますと、
P(5、4)で4!通りとなります。
よって3!×4!通りかな?
疑問に思ったことがまだありましたら、言ってください^〜^


10093.Re: 順列の問題です☆
名前:ジャグラ 高3    日付:9月22日(月) 19時6分
P(5,4)が4!通りにはなりませんね^^;
失敬。


10096.Re: 順列の問題です☆
名前:我疑う故に存在する我    日付:9月22日(月) 19時23分
質問の直接的解答になっているかどうか分りませんが、次を参考にして下さい。

先ず始点を決めて左回りに男女男女・・・・の座り方が、4!*4!, 女男女男・・・の座り方が、4!*4!. 合わせて 2*4!*4!.
回転した座り方を同一と見なすと、 8 で割って 2*4!*4!/8 = 3!*4!


10097.Re: 順列の問題です☆
名前:ケロ    日付:9月22日(月) 20時10分
みんなでわいわい考えるのって面白いね。
答なんてどうでもいいような気がしてくる。ちょっと言いすぎ。
女の座り方は 4!でした;なぜでしょうか?>
解答の場合は、最初に男の一人をテーブルに固定して、その後、
男の座り方(4-1)!と男の座り方4!を考えています。
女の一人を固定すればその反対ですが答は同じ。(4-1)! 4! だと思います。
固定するのは一人だけです。
男の座り方が (4−1)!
 女の座り方は (4−1)!
だと、二人固定したことになってしまうと思います。


10098.Re: 順列の問題です☆
名前:ヨッシー    日付:9月22日(月) 20時15分
まず、n個を並べる円順列がなぜ、(n−1)!であるかを、理解する必要があります。
円なので、ぐるぐる回ってしまうため、一人を固定してしまって、残りの
n−1人をその左(もしくは右)から並べるように考えたのが(n−1)!です。

最初、男を並べるときは、4人を並べる円順列ですが、そのあと女を並べるときは、
男によって、位置が固定してしまっているので、普通の順列になるのです。
 
http://yosshy.sansu.org/


10099.Re: 順列の問題です☆
名前:あいこ(高1)    日付:9月22日(月) 20時53分
みなさんの説明とてもためになります!!この解答の意味は、ある一人の男を固定して、それに他の男を並べて、間に女を並べるやり方ですね!そもそも(n−1)!の公式の意味をただ単に円のときはこれを使え!!っト言うような程度にしかおもっていなかたのでいけませんね;こんな風ですよね? ●=男 △=女 とする。

  ●(←固定) 固定していない他の3つを一列に並べたと考えて
 ● ●       3! つまりは(4−1)! っとなるんですね!
  ●


   


10100.Re: 順列の問題です☆
名前:あいこ(高1)    日付:9月22日(月) 21時1分

ジャグラさんに質問です!
まさしく答えは 3!×4! です!!しかし、
『それに●○●○●○●○● ●が何通りあるかと言いますと、
P(5、4)で4!通りとなります。』
っというところがいまいち分かりません;丸の数はこの人数の合計である8個ではなく9個なのはなぜですか?そしてP(5、4)っていうのは、5個から4個をとる順列を表しているのですか?




10101.Re: 順列の問題です☆
名前:あいこ(高1)    日付:9月22日(月) 21時8分
順列については本当に苦手なので理解力に欠けている点は目をつむってください;際先ず始点を決めて左回りに男女男女・・・・の座り方が、4!×4!のところなのですが、なぜ求める式が4!×4!になるのかがわかりません(>_<;)教えてください!!


10105.Re: 順列の問題です☆
名前:ジャグラ 高3    日付:9月22日(月) 21時43分
アイコさん>
間違って●の数一つ多かったようです(;;
4!通りで正しいはずです〜。


10106.Re: 順列の問題です☆
名前:arc    日付:9月22日(月) 21時46分
なんだかよく分からないけど、最初の問題の答え。

>男女各4人の8人が男女交互に円卓の回りに座る方法は何通りあるか。

(男1)女1223344(男1

のように男1を固定すると、

1223344

を一列に並べる順列の解と同じことになる。





1223344

女の並び方   44 = 4!
男の並び方   33 = 3!


∴ 4!3! = 24*6 = 144 (通り)


※ちなみに、8人が円卓の回りに座る方法は、7!=5040(通り)
※ランダムに座った場合、男女交互になるのは、(4!3!)/7!=1/35


10121.Re: 順列の問題です☆
名前:あいこ(高1)    日付:9月23日(火) 7時15分
どうもありがとうございました!!!理解いたしました☆★"

10089.数列の問題です。  
名前:トモ(高1年)    日付:9月22日(月) 15時36分
(問題)
数列の和Sn=1+2x+2x^2+・・・+2x^n-1を計算しなさい。

う〜ん、どのように解いていくのでしょうか?解りません!詳しい説明よろしくお願いします。



10090.Re: 数列の問題です。
名前:えいぶ    日付:9月22日(月) 17時39分
Sn=1+2x+2x^2+・・・+2x^n-1が
Sn=1+2x+(2x)^2+・・・+(2x)^(n-1)…(1)
Sn=1+2x+2(x^2)+・・・+2{x^(n-1)}…(2)
のどっちか分からないので両方書いておきます(等比級数ならば(1)ですが。
(1)の場合
Sn=1+2x+(2x)^2+・・・+(2x)^(n-1)…(a)
両辺に2xを掛けると
2xSn=2x+(2x)^2+・・・+(2x)^n…(b)
(b)-(a)=(2x-1)Sn=(2x)^n-1
したがってSn={(2x)^n-1}/(2x-1)

(2)の場合
Sn=1+2x+2(x^2)+・・・+2{x^(n-1)}
Sn-1=2x+2(x^2)+・・・+2{x^(n-1)}
(Sn-1)/2=x+x^2+・・・+{x^(n-1)}
ここでx+x^2+・・・+{x^(n-1)}=k…(A)とおくと
xk=x^2+x^3・・・+x^n…(B)
(B)-(A)より(x-1)k=x^n-x
したがってk=(x^n-x)/(x-1)
(Sn-1)/2=(x^n-x)/(x-1)
……となります(後は自分でもできるでしょう。

10085.絶対値が等しい関係について  
名前:torakiti    日付:9月22日(月) 11時21分
たとえば分母と分子をひっくり返した数の子とを逆数と言いますよね。
では−3と3のように絶対値が等しくて異符号の数どうしのことを
「〜数」という言い方はないでしょうか?



10086.Re: 絶対値が等しい関係について
名前:我疑う故に存在する我    日付:9月22日(月) 13時0分
反数と呼ぶようです。

参考ページ
http://www.info.kochi-tech.ac.jp/sakamoto/lecture/algebra01/oct16.html
http://hosoi05.is.noda.sut.ac.jp/~hosoi/kanzi/html/17.htm
http://hosoi05.is.noda.sut.ac.jp/~hosoi/kanzi/html/17.htm#hansuu


10087.Re: 絶対値が等しい関係について
名前:torakiti    日付:9月22日(月) 13時13分
ありがとうございました。さんこうになりました

10075.弧長の求め方  
名前:RTTR    日付:9月21日(日) 21時24分
弦と高さしかわからない時の弧長の求め方を教えて下さい。



10083.Re: 弧長の求め方
名前:ヨッシー    日付:9月22日(月) 8時5分
図のように、弦の長さを2d、高さをh、中心角を2θとします。

このような扇形において、弧の長さは
 r×2θ=2rθ
です。
一方、太線の直角三角形における三平方の定理より
 r2=d2+(r−d)2
より、
 r=(d2+h2)/2h
三角関数の逆関数を使って、
 θ=Sin-1(d/r)
これらより、弧の長さLは
 L={(d2+h2)/h}Sin-1{2dh/(d2+h2)}
となります。
 
http://yosshy.sansu.org/


10084.Re: 弧長の求め方
名前:ヨッシー    日付:9月22日(月) 10時15分
dは弦の長さではなく、弦の半分の長さであることに注意してください。
 
http://yosshy.sansu.org/


10139.Re: 弧長の求め方
名前:匿名希望    日付:9月23日(火) 18時3分
> 弦と高さしかわからない時の弧長の求め方を教えて下さい。

回答で
>このような扇形において、弧の長さは
> r×2θ=2rθ
>です。
>一方、太線の直角三角形における三平方の定理より
>r2=d2+(r−d)2

の最終行は誤りで、
r2=d2+(r−h)2
でしょう。


10141.Re: 弧長の求め方
名前:ヨッシー    日付:9月23日(火) 22時22分
あらら、そうですね。
打ち誤りです。

その続きからは、hも入っていて、OKのはずです。

P.S. 上の方の同内容の記事は消しておきます。
 
http://yosshy.sansu.org/

10074.お願いします 二問いっきで失礼します  
名前:もう少しで期末    日付:9月21日(日) 21時22分
次の不等式がすべての実行値に対して成り立つかどうか調べよ
x二乗−6x+10>0 

次の不等式をとけ
x3乗−2x≦0
高1です
どうかお願いします



10080.Re: お願いします 二問いっきで失礼します
名前:ケロ    日付:9月22日(月) 0時25分
x二乗−6x>をにらんで、x^2+2ax+a^2 の式を思い出します。
x^2+2ax+a^2=(x+a)^2 なので、 (^2は二乗)
(x-3)^2 を作ります。
(x-3)^2=x^2-6x+9 だから、1足りません。1を足すと、
(x-3)^2+1=x^2-6x+9+1 。結局、
x^2-6x+10=(x-3)^2+1。
(x-3)^2 は実数の2乗だから、0以上。それに1を足したのだから。

x^3-2x=x(x^2-2)=x(x-√2)( x+√2)
これが0になるのはx=-√2), x=0, x=√2 のとき。
その前後と間のプラスとマイナスを調べます。
y= x(x-√2)( x+√2) のグラフを描いて、
x軸とx軸より下になる部分を探してもできます。


10114.Re: お願いします 二問いっきで失礼します
名前:もう少しで期末    日付:9月23日(火) 0時5分
ありがとうございます!!早速やってみます

10069.ベクトル  
名前:味噌汁    日付:9月21日(日) 13時2分
こんにちは。基本的な問題かもしれませんが教えてください。

ベクトル
(8,27)の大きさを求めるにはどうすればよいのですか?

よろしくお願いします。



10070.Re: ベクトル
名前:中川 幸一    日付:9月21日(日) 14時16分
a=(a1, a2) とすると,
|a|=√(a12+a22)
です。

http://www3.ezbbs.net/01/k-nakagawa/


10071.Re: ベクトル
名前:味噌汁    日付:9月21日(日) 15時24分
ありがとうございました。


10073.Re: ベクトル
名前:中川 幸一    日付:9月21日(日) 19時21分
どういたしまして。
http://www3.ezbbs.net/01/k-nakagawa/

10066.確率  
名前:高校1年生    日付:9月21日(日) 10時53分
15本のくじの中に当たりくじが5本ある。このくじを同時に5本引くとき、当たりくじが3本以上含まれる確率を求めよ。

という問題を教えてください。



10068.Re: 確率
名前:あき    日付:9月21日(日) 11時54分
当たりくじを3本、4本、5本ひいたときのそれぞれの確率を求め
足せばいいんじゃないでしょうか?
5C3×10C2/15C5+5C4×10C1/15C5+5C5/15C5だと思いますが・・
間違いがあれば指摘してください

10058.教えてください。お願いしますm(__)m  
名前:たく(中3)    日付:9月20日(土) 23時25分
「親Lとその子l、親Mとその子m、親Nとその子nが川を渡ることになった。6人はみなボートくぉこぐことができるが、定員2名のボートが1そうしかない。また川の両岸でも、ボートの中でも、次の条件も満たすものとする。
(条件):「どの子どもも、ほかの親と一緒のときにはそれぞれ自分の親と一緒にいなければならない」
上の条件の下で、全員川をボートに乗ってわたるにはどうしたらいいか。」



10060.Re: 教えてください。お願いしますm(__)m
名前:えいぶ    日付:9月21日(日) 0時37分
lmnLMN
lLMN→mn
lmLMN←n
LMN→lmn
lLMN←mn
lL→mnMN
lmLM←nN
lm→nLMN
lmn←LMN
l→mnLMN
lm←mLMN
→lmnLMN
もっと効率の良い解答がありそうですが(^^;


10062.Re: 教えてください。お願いしますm(__)m
名前:たく(中3)    日付:9月21日(日) 2時28分
すみません、理解力がなくて、、もう少し詳しく説明していただくとありがたいのですが


10081.Re: 教えてください。お願いしますm(__)m
名前:ヨッシー    日付:9月22日(月) 4時34分
左側を元いた岸、右側を目的の岸とします。
えいぶさんの回答で言うと、
 lmnLMN  ・・・最初6人が左にいる状態
 lLMN→mn ・・・mとnがボートで右に行った状態
 lmLMN←n ・・・mがボートで左に行き、n だけが右に残った状態
を表します。
 
http://yosshy.sansu.org/

10054.三角関数  
名前:味噌汁    日付:9月20日(土) 23時7分
こんばんは。分からないので教えてください。

tan(180°+θ)cos(90°+θ)+sin(180°+θ)tan(180°-θ)
の値を求めよ。という問題です。

よろしくお願いします。



10056.Re: 三角関数
名前:田村 正和    日付:9月20日(土) 23時17分
tan(180°+θ)cos(90°+θ)+sin(180°+θ)tan(180°-θ)
=tanθ−sinθ−sinθ−tanθ
=0


10061.Re: 三角関数
名前:えいぶ    日付:9月21日(日) 0時41分
tan(180°+θ)cos(90°+θ)+sin(180°+θ)tan(180°-θ)
=-tanθsinθ+sinθtanθ
=0
分からない場合は加法定理を使えば導くことが可能です。


10063.Re: 三角関数
名前:田村 正和    日付:9月21日(日) 9時27分
あ、間違えてた。でも答えはあってる(謎)


10064.Re: 三角関数
名前:味噌汁    日付:9月21日(日) 10時6分
どうもありがとうございました。

10052.確率の問題です  
名前:いちご    日付:9月20日(土) 22時47分
男子6人、女子2人がくじ引きで席を決めて円卓に座るとき、女子2人が向かい合う事象の起こる確率を求めなさい。


何回考えても分かりません。詳しい解説もお願いします。



10053.Re: 確率の問題です
名前:いちご    日付:9月20日(土) 22時48分
すみません!高校1年生です


10055.Re: 確率の問題です
名前:田村 正和    日付:9月20日(土) 23時15分
1人を固定して考えると残りの7人の中で女子は一人。
残りの7席のうち特定されてるのは1席なので
答えは1/7。
(別解)
8人の座る事象は公式(n−1)!より7!通り。
そのうち1人が決まってて残り6人の決め方が6!通りあるから
6!/7!=1/7


10065. 確率の問題です
名前:いちご    日付:9月21日(日) 10時41分
どうもありがとうございました!女子を決めて解くと分かりやすいんですね!!

10051.2次曲線(数C)  
名前:みずき    日付:9月20日(土) 21時33分
原点Oを中心とし,半径5の円周上を点Qが回転し,更にQを中心とする
半径1の円周上を点Pが回転する。時刻tのとき,x軸正方向に対し
OQ,QPのなす角はそれぞれt,15tとする。OPがx軸正方向と
なす偏角ωについてtanωをtで表せ。ただし,0≦t≦2πとする。

という問題を教えてください。お願いします。 (高2)



10076.Re: 2次曲線(数C)
名前:ケロ    日付:9月21日(日) 22時23分
線分OPをPの方向へ1延長した点をRとします。
PRをPQまで回転させることを考えます。
∠RPQ=π−(ω-5t)
なので、各点を複素数で表すと、
z_OP+z_PQ=z_OQ
ですから、
OPの長さをrとすると、
z_OP=r(cosω+isinω),
z_PQ=(cosω+isinω)(cos(π−(ω-5t))+isin(π−(ω-5t)))
=cos(π+5t)+isin(π+5t)
=-cos5t-isin5t
z_OQ=cost+isint
r(cosω+isinω) -cos5t-isin5t= cost+isint
rcosω- cos5t- cost+i(rsinω-sin5t-sint)=0
cosω=( cos5t+cost)/r ,sinω=( sin5t+sint)/r
tanω=( sin5t+sint)/ ( cos5t+cost) .
こうなったんだけど、
何か見逃していたらごめんなさい。
最初に謝って置こう。


10082.Re: 2次曲線(数C)訂正
名前:ケロ    日付:9月22日(月) 7時52分
z_OQ=cost+isint →z_OQ=5(cost+isint)
tanω=( sin5t+sint)/ ( cos5t+cost) →tanω=( 5sin5t+sint)/ (5 cos5t+cost)
かな。時間。
行ってきまーす。


10088.Re: 2次曲線(数C)
名前:ヨッシー    日付:9月22日(月) 14時44分


座標(x、y)の点の偏角をθとすると、tanθ=y/x (ただしx≠0)
なので、Pの座標を求めることにします。
Qの座標は(5cos t, 5sin t)
QからPの差分は(cos 15t, sin 15t)
よって、Pの座標は(5cos t+cos 15t, 5sin t+sin 15t)
以上より
 tanω=(5sin t+sin 15t)/(5cos t+cos 15t)
これ以上約せるかどうかと、どんなときに分母が0になるかは
吟味していません。
 
http://yosshy.sansu.org/


10095.Re: 2次曲線(数C)
名前:ケロ    日付:9月22日(月) 19時11分
ただいま。
ヨッシー師匠、図、ありがとうございます。ケロも流れだけ。流れがあっていれば、ヨッシー師匠と同じ答だけど、流れ自体はどうなのかな。設定ミス5→15、計算ミスごめんなさい。


10111.Re: 2次曲線(数C)
名前:みずき    日付:9月22日(月) 23時35分
ケロさん、ヨッシーさんありがとうございます^^
わかりました!!数学は楽しいですv


10126.Re: 2次曲線(数C)
名前:ケロ    日付:9月23日(火) 10時37分
PQではなく素直にQPでよかったのにね。
z_OQ=5(cost+isint)、z_OP=r(cosω+isinω)、z_QP =cos5t+isin5t

10040.3次元での点と直線の距離について  
名前:ひろ(3年)    日付:9月20日(土) 12時5分
すいませんが教えてください。

3次元において2点,A(Xa,Ya,Za),B(Xb,Yb,Zb)があり,
その2点ABを通る直線と点C(Xc,Yc,Zc)との距離を求めるには
どうしたらよいのでしょうか?
教えてください。



10045.Re: 3次元での点と直線の距離について
名前:中川 幸一@誕生日    日付:9月20日(土) 14時8分
少し違いますが, 『点と平面との距離』というものがあります。

点(x1, y1, z1) から平面 ax+by+cz+d=0 へ下ろした垂線の長さは

|ax1+by1+cz1+d|/√(a2+b2+c2)

http://www3.ezbbs.net/01/k-nakagawa/


10047.Re: 3次元での点と直線の距離について
名前:ひろ(3年)    日付:9月20日(土) 19時37分
ありがとうございます。
やはり平面と点との距離しかないのでしょうか?


10048.Re: 3次元での点と直線の距離について
名前:中川 幸一@誕生日    日付:9月20日(土) 19時44分
強引にやれば出来ると思いますよ!!
http://www3.ezbbs.net/01/k-nakagawa/


10049.Re: 3次元での点と直線の距離について
名前:ヨッシー    日付:9月20日(土) 20時8分
手順としては、
・Cを通って、ABを法線とする平面の式を作る・・・(1)
・直線ABの式を作る ・・・(2)
・(1)と(2)の交点Dを求める
・CDの距離が求める距離
です。
 
http://yosshy.sansu.org/


10057.Re: 3次元での点と直線の距離について
名前:ころっさす    日付:9月20日(土) 23時23分
A≠Bならば,三角形ABCの面積を2通りに表わして
sqrt( |vec(AB)|^2*|vec(AC)|^2-(vec(AB)・vec(AC))^2 )/|vec(AB)|
外積を御存知なら
|vec(AB)×vec(AC)|/|vec(AB)|


10059.Re: 3次元での点と直線の距離について
名前:田村 正和    日付:9月20日(土) 23時29分
位置ベクトルがa,bである2点を通る直線に点cより引いた垂線の長さは
{|(a-c)×(b-c)|}/|b-a|
#数学オリンピック教室の本に載ってました。


10079.Re: 3次元での点と直線の距離について
名前:ひろ(3年)    日付:9月21日(日) 23時47分
みなさんありがとうございました。

10033.文章問題  
名前:あつこ    日付:9月20日(土) 8時2分
数学が苦手なのですこしずつ勉強しています。
問題は
AからHの8枚のコインがあり、このうち2枚は偽物のコインで、本物のコインとは重量が異なっている。
ただし、本物より想いか軽いかはわかっていない。
そこで、天びんばかりを3回使って、2枚の偽物を発見することにした。
たとえば、天びんばかりの左にA,Bの2枚、右にC,Dの2枚を載せたとき、釣り合えば(AB)=(CD),左が重ければ(AB)>(CD)、右が重ければ(AB)<(CD)と表すことにすると、3回の計測結果は次のようになった。
このとき、確実にいえるのは?
1回目(ABC)=(DEF)
2階目 (ABG)>(CDE)
3回目 (AH)<(DF)

1.偽物はAとFで、本物より重い。
2.偽物はBとEで、ほんものより重い。
3.偽物はBとFで、本物より重い。
4.偽物はCとEで、本物より軽い
5.偽物はDとFで、ほんものより軽い

私の考えでは2番だとおもうですが???


数学が苦手なので、できれば低レベルで教えてくれると嬉しいです



10037.Re: 文章問題
名前:花パジャ    日付:9月20日(土) 10時56分
ai402@hotmail.comさん、既に回答が寄せられてますが


10042.Re: 文章問題
名前:あつこ    日付:9月20日(土) 13時47分
すいません。
ありがとうございます


10043.Re: 文章問題
名前:なつ    日付:9月20日(土) 13時48分
すいません。
ありがとうございます

10025.わかりません、ぜひ教えてください  
名前:yuu(高校三年)    日付:9月19日(金) 22時20分
直線2kx+3y+k−1=0はいかなるkの値に対しても定点を通る。この定点の座標を求めよ。



10026.Re: わかりません、ぜひ教えてください
名前:中川 幸一    日付:9月19日(金) 22時22分
k について整理をしてみましょう。
http://www3.ezbbs.net/01/k-nakagawa/


10031.Re: わかりません、ぜひ教えてください
名前:ヨッシー    日付:9月20日(土) 7時25分
いわゆる「恒等式」ってヤツですが、
とにかく答えが欲しければ、
k=1,2,3 など代入して、グラフを2,3本引けば
見つけることは出来ます。

ちなみに、kについて整理するとは、
 (2x+1)k+(3y−1)=0
ということです。
 
http://yosshy.sansu.org/


10078.まとめ
名前:ケロ    日付:9月21日(日) 22時52分
問題を言い換えると、
kにどんな数を入れても2kx+3y+k−1=0が成り立つときxとyはいくつですか。
好きな数を代入して連立方程式で解けます。また、
(2x+1)k+(3y−1)=0 と書き直すと、
2x+1=0、3y−1=0 なら、kがどんな値でも成り立ちます。

10017.高校1(数学苦手)  
名前:あつこ    日付:9月19日(金) 19時22分
図はURLにて。
問題
図のように、黒の碁石と白の碁石を交互に加えながら、正方形の形に並べていく。
縦と横に12個ずつ、合計144個の碁石を並べたとき、その144個の中で白の碁石は黒の碁石より何個おおいかという問題です。

これは、どのようにもとめるのですか?
お願いします

なにか公式かなんかつかうのですか?
http://www.rak1.jp/one/user/gen69/



10023.答えを出すだけなら
名前:えいぶ    日付:9月19日(金) 21時58分
1*1 黒が1多い
2*2 白が2多い
3*3 黒が3多い
4*4 白が4多い
…何かに気付きませんか?


10032.Re: 高校1(数学苦手)
名前:あつこ    日付:9月20日(土) 8時0分
ありがとうございました。
わかりました
http://www.rak1.jp/one/user/gen69/

10010.数列の問題の計算過程抜粋  
名前:トモ(高1年)    日付:9月19日(金) 16時33分
=1/6n(n+1)(2n+1)−1/2n(n+1)+n
=1/6n{(n+1)(2n+1)−3(n+1)+6}
後は略します。

※なぜ1/6nで括るのですか、最小公倍数の6掛けて
n(n+1)(2n+1)−3n(n+1)+6n
にして計算を解いてダメなんですか?もちろんこのやり方だと答えは間違っています。では何でダメなのか理由を教えて下さい。バカな質問ですみません。よろしくお願いします。



10011.Re: 数列の問題の計算過程抜粋
名前:ヨッシー    日付:9月19日(金) 17時58分
1/2 + 1/3 の計算をするのに、いきなり6を掛けて
 3 + 2 = 5
ではまずいでしょう。

nで括るのはともかくとして、
1/6 を出すのは、いわゆる「通分」と同じ考え方です。
 
http://yosshy.sansu.org/


10013.Re: 数列の問題の計算過程抜粋
名前:K.N.G.    日付:9月19日(金) 18時1分
方程式などで頻繁に行う「"両辺に"ある値を掛ける」行為と混同しているのではないかと思われます.

 □ = △
という式の"両辺"に6を掛けるならば
 6 × □ = 6 × △
と等号がなりたりますが,
□ = △ の △ だけに6を掛けると
 □ = 6 × △
となってしまい,これは誤りです.


10067.Re: 数列の問題の計算過程抜粋
名前:トモ(高1年)    日付:9月21日(日) 11時11分
理解出来ました。ヨッシーさん、K.N.G.さん、ありがとうございました。

10009.(untitled)  
名前:takumi(高3)    日付:9月19日(金) 16時13分
∫(tan^2x+1/tan^2x)dx
という問題が解けません。
tanxをsinx/cosxに直して置換しよう
としたのですが、何をtと置いていいか分かりません。
たぶん三角関数の性質で与式変形すれば
解けるような気がするのですが・・・
どなたかご教示願えると幸いです。



10012.Re: (untitled)
名前:fan    日付:9月19日(金) 17時59分
tan2x+1/tan2x
=sin2x/cos2x+cos2x/sin2x
=(1-cos2x)/cos2x+(1-sin2x)/sin2x
=1/cos2x+1/sin2x-2
=(tanx-1/tanx-2x)'


10050.Re: (untitled)
名前:takumi(高3)    日付:9月20日(土) 20時14分
そうかぁ。全くその通りですね。
ありがとうございました。

10007.5C3  
名前:TATSUMI    日付:9月19日(金) 15時20分
5案から3案に絞り込む場合の組み合わせ5C3は何通りですか?



10008.Re: 5C3
名前:田村 正和    日付:9月19日(金) 15時22分
10通り

10003.教えて  
名前:aoki    日付:9月19日(金) 14時18分
10×10-6って
0.000001???



10006.Re: 教えて
名前:ヨッシー    日付:9月19日(金) 14時45分
惜しい!!
10^-6 が 0.000001 で、10×10^-6 は、その10倍で、
0.00001 です。

私のページの「全国のお父さん向け ヨッシーの数学テキスト」
の、第9回をご覧下さい。
 
http://yosshy.sansu.org/

10002.文章問題  
名前:あつこ    日付:9月19日(金) 12時19分
問題は
A〜Cの3人は、各字が田中、佐藤、鈴木のうちのいずれかであり、出身地が東京、横浜、大阪のうちのいずれかである。
また、3人の苗字と出身地はいずれも異なっている。
いま、A〜Cがそれぞれ次のように発言した。
このとき、確実にいえるのはどれか。
ただし、鈴木と佐藤は2人とも、前半、後半の2つの発言のうち、どちらか1つがうそであり、田中は前半、後半の2つにの発言のどちらもうそである。
A。Bは鈴木です。鈴木は東京出身です。
B.Cは田中でありません。Aの出身は横浜です。
C.私は田中です。私の出身は大阪です。

1.Aは東京出身の佐藤である。
2.Aは横浜出身の田中である。
3.Bは横浜出身の鈴木である。
4.Bは大阪出身の佐藤である。
5.Cは大阪出身の鈴木である。

私は2番かなとおもうのですが、どうでしょうか?
/



10004.Re: 文章問題
名前:ヨッシー    日付:9月19日(金) 14時32分
田中は必ずウソを言うので、田中は「自分を田中とは言わない」
「自分以外の人を田中ではないとは言わない」より、B、Cは
田中でないことがわかります。
すると、Aが田中であり、その発言から、Bは鈴木ではなく佐藤とわかります。
あとは、出身地をうまく決めてやれば出来上がりです。

正解は5番ですね。

B(佐藤)が言っている、「Cは田中でない」は本当なので、「Aは横浜」はウソになり、
2番は正解でなくなります。
 
http://yosshy.sansu.org/


10016.Re: 文章問題
名前:あつこ    日付:9月19日(金) 19時21分
ヨッシー さんありがとうございました。
わかりました。
/

9992.方程式  
名前:T.T.C.高2    日付:9月19日(金) 1時15分
この間はお世話になりました。ありがとうございます。お礼言えなかったのでここでm(_ _)m
1)a、bが素数であって、2次方程式3x2−12ax+ab=0が2つの整数解を持つa、bの値と整数解をもとめよ。
2)(A)方程式x3=1の虚数解の1つをωとする。実数a、bに対して、等式(ω2+aω)(ω+b)=−3が成り立つとき、aの値を求めよ。
(B)a{(√2)+i}4−b{(√2)+i}3−c{(√2)+i}2+d{(√2)+i}+1−√2=0(iは虚数単位)が成り立つa、b、c、dの値を求めよ。
3)pを素数とする。xの2次方程式x2+(p2−7p−2)x+2p2−15p−8=0が整数解をもつとき、pの値と方程式の解w求めよ。
これらの問題がわかりません。助けてください。



9996.Re: 方程式
名前:ヨッシー    日付:9月19日(金) 9時56分
(1) 整数解をm、nとすると、
 3x2−12ax+ab=3(x−m)(x−n)
と書けます。係数比較して、
 m+n=4a、 3mn=ab
a,bは素数なので、3mn=ab より、
 m、nのいずれかが1で、他方は素数。
 a,bのいずれかが3で、他方は、m、nのうち1でない方。
とわかります。m=1 とおいても一般性を失わないので、
 a=3,b=n と a=n,b=3
に絞って、考えます。

(2)(A) x3=1 より、
 (x−1)(x2+x+1)=0
となり、ωは、x2+x+1=0 から得られる解で、
 ω2+ω+1=0
が成り立ちます。よって、ω2=−ω−1
また、当然 ω3=1。これらを踏まえて、
 (ω2+aω)(ω+b)=−3
を変形すると、
 (ab−a−b)ω+(4−a−b)=0
よって、ab−a−b=0 かつ 4−a−b=0

(2)(B) X=(√2)+i および、Y=(√2)−i について、
 X、Yを解に持つ2次方程式は、
 x2−2√2x+3=0 よって、
 X2−2√2X+3=0 が成り立ち、
 X2=2√2X−3
例えば、
 X4=(X2)2=(2√2X−3)2
  =8X2−12√2X+9
  =8(2√2X−3)−12√2X+9=4√2X−15
などのように、次数を落としていけば、Xの1次式にまとめられます。

(3) 考え中
 
http://yosshy.sansu.org/


9997.Re: 方程式
名前:帰ってきた赤猫(旧 : Red cat)    日付:9月19日(金) 11時33分
(3) は難しいですね。
その様な解があるとすれば、それは
2p2 - 15p - 8 = (p - 8)(2p + 1)
の約数になっています。これがヒントになるかどうかはちょっとわからない…。

力不足で申し訳ありません。


9998.Re: 方程式
名前:Sar    日付:9月19日(金) 11時42分
うーん……

pについて整理して、
(x+2)p^2 - (7x+15)p + x^2 - 2x - 8 = 0
で、x≠-2の場合、このpの2解をα,βとして
α+β=(7x+15)/(x+2)={7(x+2)+1}/x+2=7 + 1/(x+2)で、
α+βが整数になる整数xを考えると、x=-1,-3だから、x=-2だった場合も考えてこれらを代入ってのはどうでしょ。


9999.Re: 方程式
名前:帰ってきた赤猫(旧 : Red cat)    日付:9月19日(金) 11時49分
>Sarさん
ウマー。
ちなみに x = - 2 のときは p = 0 になっちゃうので、駄目なようですね。


10001.Re: 方程式
名前:Sar    日付:9月19日(金) 12時3分
>帰ってきた赤猫さん

解法としては自信ないんです(^^;

こうした上でも一応正解は出るんですが、それが正解の全てかどうかはわかんないです。

(x+2)p^2 - (7x+15)p + x^2 - 2x - 8 = 0
の段階で、よく考えたらpの2解のうち、「少なくとも1つが」素数であったら良いんですよね。だからα+βが整数である必要はないんですよねえ。

それに、αβ=(x^2 - 2x - 8)/(x+2)=(x-4)で、α,βが共に素数ならαβ≧4だからx≧8で、これではα+βが整数になりっこないんで、敢えてこれを無視して前述の様なお茶の濁し方をした訳です。高校生の頭ではこれが精一杯でした(^^;


10005.Re: 方程式
名前:ころっさす    日付:9月19日(金) 14時35分
与式は
(x+p^2-7*p-4)*(x+2)=p
と変形できますね.


10020.Re: 方程式
名前:ひょー    日付:9月19日(金) 20時46分
超 cool !
# それに気づけ、という問題なのか?


10034.Re: 方程式
名前:Sar    日付:9月20日(土) 9時27分
成程。さすがに思いつきませんでした。pが素数である、という条件を不自然でなく使えますね。

逆に言えば、pが素数であるという条件がここまで変形するヒントだ、という事なんでしょうかねえ。

9982.four four's  
名前:たく    日付:9月18日(木) 23時57分
質問ですが、4を4つ使って1111を作れる方いますか?



10015.Re: four four's
名前:BWV645    日付:9月19日(金) 18時47分
[ √(√(√(4!)!)) ] + ( [ √(√((4!!)!)) ] − [ √(√4) ] )^√4

= [√(√(√24!))] + ( [ √( √(8!) ) ] − [ √(√4) ] )^2

= [√(√(√620448401733239439360000))] +( [ √( √(40320) ) ] − [ √(√4) ] )^2

= [ 942.08 …… ] +( [ 14.17 …… ] − [ 1.41…… ] )^2

= 942 +( 14 − 1 )^2 = 942 + 169 = 1111.


( [ ] はガウス記号、! は階乗、!! は二重階乗 )


10027.Re: four four's
名前:たく(中3)    日付:9月19日(金) 22時59分
親切にありがとうございます。これ以外に、方法はありますでしょうか。何通りかあるようなのですが。よろしければお教えくださいm(__)m


10077.Re: four four's
名前:ケロ    日付:9月21日(日) 22時33分
なんでもありなら。
二ヶ所で曲がっている4という字を真っ直ぐにする。
4444→1111。

9975.お願いします。  
名前:IGA    日付:9月18日(木) 22時30分
Original Size: 925 x 443, 18KB

次の図のように、直線y=-x+aが直線y=x,x軸と交わる点をそれぞれP、Qとする。△POQの内部および周上にある点のうち、x座標、y座標がともに正の整数である点の個数をnとする。このとき次の問に答えよ。ただし、あは正の整数とする。

問い
30<n<40となるようなaの値をすべて求めよ。
  = =

すいません『問い』のやつがみにくいかもしれません。
言葉で表すとnは30以上40以下です。
この関数の数える系はかなり苦手で・・汗)( ´д)(´д`)(д` )
ゥ゚゚(´□`。)°゚。ワーン!!ゥ゚゚(´□`。)°゚。ワーン!!



9976.Re: お願いします。
名前:IGA    日付:9月18日(木) 22時32分
すいません。
問題文中の
『あ』は→『a』です。
すいません。


9977.Re: お願いします。
名前:中川 幸一    日付:9月18日(木) 22時36分
学年は確か中学3年でしたよね?

a=1 のときは n=?
a=2 のときは n=?
a=3 のときは n=?
a=4 のときは n=?
a=5 のときは n=?
a=6 のときは n=?

というように具体的に求めて, 規則性を見つけてみましょう。

http://www3.ezbbs.net/01/k-nakagawa/


9981.Re: うむ・・規則性見つけてもダメでした。
名前:IGA    日付:9月18日(木) 23時40分
テキストの解答をみると、
a=11,12なんです・・・
うむ・・・わかりません。
とりあえず規則性を見つけだしたいと思います。
それで(間違っている可能性が高いが)
aが1増えるとnが2増えるという規則があるのですが・・
なぜ?
aはふたつ?しかも11と12?うむ・・・
とても難しくて・・
お願いします。!!!!!


9983.Re: お願いします。
名前:中川 幸一    日付:9月19日(金) 0時14分
a= 1 のときは n= 0
a= 2 のときは n= 0+1=1
a= 3 のときは n= 1+1=2
a= 4 のときは n= 2+2=4
a= 5 のときは n= 4+2=6
a= 6 のときは n= 6+3=9
a= 7 のときは n= 9+3=12
a= 8 のときは n=12+4=16
a= 9 のときは n=16+4=20
a=10 のときは n=20+5=25
a=11 のときは n=25+5=30
a=12 のときは n=35+6=41
a=13 のときは n=41+6=47

規則性は見えましたか?

http://www3.ezbbs.net/01/k-nakagawa/


9984.Re: お願いします。
名前:arc    日付:9月19日(金) 0時21分
私はy=0のときも含めていました・・・(滅

すみませんでした。


9985.Re: お願いします。
名前:IGA    日付:9月19日(金) 0時23分
a= 1 のときは n= 0
a= 2 のときは n= 0+1=1
a= 3 のときは n= 1+1=2
a= 4 のときは n= 2+2=4
a= 5 のときは n= 4+2=6
a= 6 のときは n= 6+3=9
a= 7 のときは n= 9+3=12
a= 8 のときは n=12+4=16
a= 9 のときは n=16+4=20
a=10 のときは n=20+5=25
a=11 のときは n=25+5=30
a=12 のときは n=35+6=41
a=13 のときは n=41+6=47
        ↑↑
それなりにみえてきました。しかし式の意味を十分把握してません。
↑でさしたとこがわからないのですが・・
なぜこのような加法が成り立つのでしょうか?
すいません。


9986.Re: お願いします。
名前:IGA    日付:9月19日(金) 0時25分
すいません↑の位置が悪かったです。
n= 6+3=9です。
つまりn=の一連の式の意味が分かりません。
お願いいたします。


9987.Re: お願いします。
名前:arc    日付:9月19日(金) 0時32分
a=6 のときのnは、
【a-1のn】+【[6/2]】です。

a=7 のときのnは、
【a-1のn】+【[7/2]】です。



[a/2]は、a/2の整数部分です。
(実際にはガウス記号=[x]はxを超えない最大の整数。ですが。)

式の意味はこのようになっております。


9988.Re: お願いします。
名前:中川 幸一    日付:9月19日(金) 0時34分
Original Size: 668 x 668, 79KB

参考になる図を作ってみました。

これを見ながらもう一度考えてみてください。

http://www3.ezbbs.net/01/k-nakagawa/



9989.Re: お願いします。
名前:IGA    日付:9月19日(金) 0時36分
【a-1のn】
とは、はて?なんでしょうか?
うむ・・・
すいません。


9990.Re: お願いします。
名前:arc    日付:9月19日(金) 0時42分
中川さんが作成された図を見ていただければ分かりますが、
(例えば)a=4のときのnは、

辺PQ上にあるx,yが整数の座標 + 【今までのn】

ということになります。

初期の値でない(a=6と7)ので、【a-1のn】と書きましたが、
今までのn("a"の1つ前のaの値) と "a"が通るx,yが整数の座標

個分の点が"a"のnになります。





分かりにくくてすみません。
文章で説明するのは難しいですね・・・(汗


10014.Re: お願いします。
名前:IGA    日付:9月19日(金) 18時2分
なるほど!うむ・・・・
ではこの式をもとに規則性を見つけだします。


10021.Re: 失礼します。
名前:IGA    日付:9月19日(金) 21時45分
9983のやつなんですけど
>a=12 のときは n=35+6=41
a=13 のときは n=41+6=47
の計算が間違っているような気がするのですが・・・
これだと問題集の解答とうまくあわないのです。
a=12 のときは n=36なんですが・・
ちなみにa=13 のときは n=42なんです。うむ・・・・
なぜでしょう?
ちなみにa=13 のときは n=42なんです。


10022.Re: お願いします。
名前:IGA    日付:9月19日(金) 21時55分
ああ・・
入力ミスでしょうか?
35は30ですね。ということは理解できます。
すいません。
ということはうむ・・・

規則性はないような・・・気がするのですが・・
やはりある程度見通しをたてて代入していくのでしょうか?
規則性があれば教えていただけるとありがたいです。


10024.Re: お願いします。
名前:中川 幸一    日付:9月19日(金) 22時13分
そうですね。
計算ミスをしていました。
正しくは,

a=12 のときは n=30+6=36
a=13 のときは n=36+6=42

です。

一般項は,
odd(奇数) : [a/2](1+[a/2])
even(偶数) : [a/2](2+[a/2])-[a]

※ [x] とは x 以下の最大の整数

http://www3.ezbbs.net/01/k-nakagawa/


10028.Re: お願いします。
名前:IGA    日付:9月19日(金) 23時6分
ほほ、一般項ですか・・
ふむ、この式に代入するればできるってことですよね?(自分の考えは間違っているような・・)(汗
odd(奇数) : [a/2](1+[a/2])=n
even(偶数) : [a/2](2+[a/2])-[a]=n←解いても解けないので私の解釈が間違っています)
となるのでしょうか?あれ?全く分かりません。
ごめんなさい。本当に馬鹿で・・・ゥ゚゚(´□`。)°゚。ワーン!!
ほんとに私は理解能力がない・・(悲しい
お忙しいところ暇があればでいいので
この式の意味を教えてくださると幸いです。


10029.Re: お願いします。
名前:中川 幸一    日付:9月19日(金) 23時28分
具体例をあげてみます。

odd(奇数) : [a/2](1+[a/2])
even(偶数) : [a/2](2+[a/2])-[a]

a=5
[5/2](1+[5/2])
=[2.5](1+[2.5])
=2(1+2)
=2×3
=6

a=6
[6/2](2+[6/2])-[6]
=3(2+3)-6
=3×5-6
=15-6
=9

といった感じです。

分かりましたか?

http://www3.ezbbs.net/01/k-nakagawa/


10030.Re: お願いします。
名前:IGA    日付:9月20日(土) 0時53分
ほほ〜ありがとうございます。すいません。お忙しいところ・・
しかしodd(奇数) : [a/2](1+[a/2])
even(偶数) : [a/2](2+[a/2])-[a]
とこの複雑な式ですが、確認ですが、nの値を代入したい場合はこの関係から言うとnが偶数の場合、奇数oddの式に代入すればOKですよね?
うむ・・・なるほど!
ではとりあえずこの式の意味について考えますと
とりあえずaが奇数だとbが偶数でありaが奇数であるとbが偶数であるというわけですね。そこのところにめをつけたのですね。
そこのところまではわかるのですが・・・
このような式をたてるまでの過程を教えてください

※暇があったらでいいです。すいません。いくつも次から次へと質問攻めで・・・まったくすいません。( ´д)(´д`)(д` )


10038.Re: お願いします。
名前:arc    日付:9月20日(土) 11時23分
では、私が・・・


まず、【x軸】と【y=x】と【y=-x+a】の交点の三角形内の点についてですが、
問題は、x,yが共に正の整数(=0を含めない)ということなので、

グラフを作り、ある程度の値まで作図により求めます。

すると、
a=1 のとき n=0 =0
a=2 のとき n=1 =0+1
a=3 のとき n=2 =1+1
a=4 のとき n=4 =2+2
a=5 のとき n=6 =4+2
a=6 のとき n=9 =6+3
a=7 のとき n=12 =9+3

青の数値は、【1つ前のaのときの「n」】です。
赤の数値は【y=-x+a】が通る「x,yが共に正の整数」である点の数です。
青+赤=aのときのnです。


ちょっとややこしいですが、ここまでは分かりますね。
(説明が下手なので分からないかも・・・(汗 )




ここから規則性を求めます。

【漸化式】という式が立つのですが、習っていないと思うので、微妙に触れていきます。(何


■青の値
まず、あるaがあるとき、【1つ前のaのときの「n」】を求めるには、その値が決まってなくてはいけません。

決まっていないと、aの青の数値=(a-1の青=(a-2の青=(・・・)+(a-2の赤)))+(a-1の赤)

となって、青が求められません。


なので、青=(a-1)の値=【1の値】 としましょう。(作図から求めるのでなんでもいいですが、少ない値のほうがいいです。)

【a=1のとき】n=0 ですね。a=1 n1=0 ※a=aのときのnをnaと書きます。


■赤の値
a=1 のとき 赤=0
a=2 のとき 赤=1
a=3 のとき 赤=1
a=4 のとき 赤=2
a=5 のとき 赤=2
a=6 のとき 赤=3
a=7 のとき 赤=3

この後も、a=8,9 赤=4 , a=10,11 赤=5 ・・・・・ となります。

この規則性は、a/2のときの整数部分が赤となっています。
何故かは、【x,yが共に正の整数(整数でも同じ)】というところから来ています。

故に、式は、[a/2] ※[x]はガウス記号といい、xを超えない最大の整数xのことになります。
(※ガウス記号を直接使うと、負の数の場合は、ただ整数部分を表す。という意味にはなりません。)
(例:[1]=1 [1.5]=1 [2.7]=2 [-5]=-5 [-4.7]=-5 [-1.6]=-2)
(ガウス記号を使った値の数値は、【 k≦x<k+1 】が成り立つとき、x=k というものです。)


この式を組み合わせると、


na = na-1+[a/2]

となります。
このような式を漸化式といいます。(一般にはanとかと書きますが。)


一旦終了しますw


10072.Re: お願いします。
名前:IGA    日付:9月21日(日) 17時46分
有り難うございます!。
助かります!

9964.2次関数の問題です  
名前:高校1年生    日付:9月18日(木) 21時42分
次の関数に最大値と最小値があればそれを求めよ。

1 y=3(x+1)(x-2) (0≦x<3)

2 y=x^2-2x+2 (-1<x<2)

この問題が分からないので解説つきで教えてください



9966.Re: 2次関数の問題です
名前:中川 幸一    日付:9月18日(木) 21時46分
まずは平方完成をしてみましょう。
http://www3.ezbbs.net/01/k-nakagawa/


9968.Re: 2次関数の問題です
名前:高校1年生    日付:9月18日(木) 21時55分
平方完成はできました

1 y=3(x-1)^2-9

2 (x-1)^2+1


になりました。


9969.Re: 2次関数の問題です
名前:中川 幸一    日付:9月18日(木) 22時8分
1. の平方完成がおかしいです。

あと, 平方完成した式を頼りにグラフを描いてみましょう。

http://www3.ezbbs.net/01/k-nakagawa/

9963.教えて!  
名前:あさ    日付:9月18日(木) 21時37分
xについての方程式x+1/x=tが相異なる2つの正の解をもつための
実数tの条件を求めよ。という問題です。
指針が浮かびません お願いします。
高1です。。



9965.Re: 教えて!
名前:中川 幸一    日付:9月18日(木) 21時45分
y=x+(1/x) のグラフを描いてみましょう。これと, y=t との位置関係で求まると思います。
http://www3.ezbbs.net/01/k-nakagawa/


9970.Re: 教えて!
名前:boundary    日付:9月18日(木) 22時11分
x+1/x=t の両辺にxをかけて、
 ・
左辺に移項して、
 ・
 ・
判別式Dの式を使って、、
 ・
 ・
ほら、できたでしょ?


9972.Re: 教えて!
名前:中川 幸一    日付:9月18日(木) 22時15分
その解法だと, 相異なる2つの解を得る条件だけであって, それが正かどうかまでは判断できません。

この場合は他の条件もいるのですが, それは何でしょう?

このようにいろいろな解法を学ぶのも勉強ですね☆

http://www3.ezbbs.net/01/k-nakagawa/


10019.Re: 教えて!
名前:あさ    日付:9月19日(金) 20時5分
ありがとうございます。わかりました!

9951.お願いします!!  
名前:IGA    日付:9月18日(木) 17時21分
帯分数でaとc/bとかくとa+(c/b)ですが、aとc/bは*を省いたものと考えてa*c/bと見ることもできます。いま5と1/2-4と1/3をX君は{5*(1/2)}−{4*(1/3)}と計算して7/6を答えまたY君は(5+1/2)−(4+1/3)と計算して同じ値7/6を答えにしました。
ところで5と1/m-4と1/n(m,nは2以上の整数)この二つの計算方法で同じ値になることが上の例(m=2 n=3)以外にあるか?
ないときはそのことを証明し、ある場合はm、nを求めなさい。

という問題なんですが・・帯分数が使われているのでパソコンで表すとわかりにくいかもしれません。すいません。『と』帯分数の区切りです。
それで私なりに考えました。値が一緒なのだから方程式で解けますよね?
それで(5*1/m)−(4*1/n)=(5+1/m)−(4+1/n)になり
解いていくと、
4/m-3/n=1になります。おそらく代入しても成り立つのでOKです。
それで地道に代入していけばたしかにできるのですが・・
それだと数学の意味がないような気がするので、
この式からどう発展さしていけばいいか教えてくださるとありがたいです。
お願いいたします!



9952.Re: お願いします!!
名前:IGA    日付:9月18日(木) 17時24分
中三です。
おねがいします。頭の体操みたいな問題ですね。


9953.Re: お願いします!!
名前:ヨッシー    日付:9月18日(木) 18時2分
4/m-3/n=1 の両辺に mn を掛けて整理すると、
 mn+3m-4n=0
因数分解の要領で変形すると、
 (m-4)(n+3)=-12
n+3 は正なので、積が負になるとすれば、m-4 が負になります。
 m-4=-1,-2,-3,-4,-6,-12
の場合が考えられますが、そのうちで m>0 を満たすものを見つけます。
http://yosshy.sansu.org/


9954.Re: お願いします!!
名前:Sar    日付:9月18日(木) 18時24分
では、(4/m) - (3/n) = 1のままで考えてみるというのも。
m>0,n>0より(4/m),(3/n)は共に正なので、上の式から少なくとも
4/m > 1がいえるので、m>0と合わせてm=3,2,1で以下略、と。


9955.Re: お願いします!!
名前:IGA    日付:9月18日(木) 18時33分
うむ・・・
一応できました。m=3、n=9ですね。
うむ・・・ちょっと質問なんですが・・
>mn+3m-4n=0
因数分解の要領で変形すると、
 (m-4)(n+3)=-12
とありますが、この式までにするまでのやり方がよくわかりません。
しかし〜これは乗法公式をつかってませんよね?
やはり頭の中で計算したのでしょうか?
やりかたを詳しくお願いします。

(本当にすいません。初歩的なことで・・・ゥ゚゚(´□`。)°゚。ワーン!!)


9956.Re: お願いします!!
名前:ケロ    日付:9月18日(木) 19時53分
>一応できました。m=3、n=9ですね。
>m,nは2以上の整数
とありますから、もうひとつ。
>これは乗法公式をつかってませんよね?
展開の問題を何度も解いているうちに気づけばいいと思います。
でも、展開でこういうのが最初に出てきたと思います。
(x+a)(y+b)=xy+bx+ay+ab。


9957.Re: お願いします!!
名前:IGA    日付:9月18日(木) 20時19分
ケロさんへ

(゚ロ゚;)エェッ!?まってください。
(m-4)は負の数となるわけですよね?
なおかつmは2以上の整数ですよね?
つまり正の整数であるからこの条件を満たすのは、
3しかないよなきがするのですが・・・
指摘お願いします!
( ´д)(´д`)(д` )(汗


9959.Re: お願いします!!
名前:ケロ    日付:9月18日(木) 20時42分
何か読み間違えてるかな?
m=2、n=3 でダメ?


9960.Re: お願いします!!
名前:ケロ    日付:9月18日(木) 21時21分
ごめん最初に答えでてたね。


9962.Re: お願いします!!
名前:IGA    日付:9月18日(木) 21時27分
ヨッシーさん、Sarさん、ケロさんありがとうございます。
お忙しいところレスをしてくださいまして誠にありがとうございます。
今後ともよろしくお願いします!!<(_ _*)> アリガトォ


9967.Re: お願いします!!
名前:ケロ    日付:9月18日(木) 21時49分
問題の読み間違えをしたので、式のほうについて。
xy+bx+ayを(x+a)(y+b)にして、
abはあとから考えます。
mn+3m-4n を先に強引に (m-4)(n+3) として、
もとにもどるように12は後で足すわけです。
mn+3m-4n=(m-4)(n+3)+12。


9973.Re: お願いします!!
名前:IGA    日付:9月18日(木) 22時17分
ほほ〜なるほど!
文字が二つあるものがもとになっておりますね。(式の中にxが二つyが二つありますね)なるほど〜
まず強引にやって後で調整するのですね。
ふむふむ。|゚ー゚||。_。||゚-゚||。_。|ウンウン
ケロさんご丁寧にありがとうございます。

9947.大学の数学とってます  
名前:みな    日付:9月18日(木) 12時20分
集合の問題なんですが、解けずに困っています。
1:素数を表す集合
2:要素が少なくとも2つある集合
3:要素が2つしかない集合
教えていただけますか?



9948.Re: 大学の数学とってます
名前:みな    日付:9月18日(木) 12時32分
私の途中までの答えを載せてみます

1:素数を表す集合
素数とは己と1以外の数では割り切れないすうじですよね。
{ p : N | (m=1 V n=1)V(m=p V n=p) → p=mn }
というのが私の答えなのですが。。絶対にまちがっている。。。
2:要素が少なくとも2つある集合
{ ヨx v ヨy: N | x v y ∈S}
あってますかしら?
3:要素が2つしかない集合
{ (ヨx:N| x∈S) v (ヨy:N| y∈S) v (∀z: N|。。。。 }
とこれはとちゅうまでしかできませんでした。


9950.Re: 大学の数学とってます
名前:帰ってきた赤猫(旧 : Red cat)    日付:9月18日(木) 12時44分
難しいですね。
1. 整数 a , b について、a が b を割り切る(⇔ b/a が整数)ことを a|b で表すことにします。
#逆だったかな?
このとき
p is a prime number (p が素数)
⇔ {(n ∈ N and n|p) ⇒ (n = 1 or n = p)}
#敢えてきちんとした論理式の形には書いていません。

2. その様な集合を S とするならば、まずなにがしかの S の元 x があり、それとは別に y という S の元が存在します。それをきちんと論理式で表すと?

3. 2. のヒントを元に、同様に考えてみてください。


9994.Re: 大学の数学とってます
名前:我疑う故に存在する我    日付:9月19日(金) 9時36分
素数全体の集合 =

{ x | x は素数 }


10000.Re: 大学の数学とってます
名前:帰ってきた赤猫(旧 : Red cat)    日付:9月19日(金) 11時58分
1 について。
p ≠ 1 が抜けてましたね。
p (∈ N) is a prime number (p が素数)
⇔ p ≠ 1 and {(n ∈ N and n|p) ⇒ (n = 1 or n = p)}

>素数全体の集合 = { x | x は素数 }
それを言ってしまうと身も蓋もないような…。


10119.Re: 大学の数学とってます
名前:みな    日付:9月23日(火) 3時38分
ありがとうございました。

もう一度トライしてみます
(数学にまけないぞ!)

9940.問題  
名前:あき    日付:9月17日(水) 22時12分
高校1年です。
xについての4次方程式x4(四乗)-ax3+(a+6)x2-ax+1=0が相異なる
4つの正の実数解をもつための実数aの条件を求めよ とあるのですが
どのように解けばよいのかわかりません。因数分解を試みましたが
できません。どのようにすればいいのでしょうか・・



9941.Re: 問題
名前:田村 正和    日付:9月17日(水) 22時32分
両辺をx2で割ってみましょう。


9946.Re: 問題
名前:ヨッシー    日付:9月18日(木) 11時14分
9509 番に同様の質問があります。
 
http://yosshy.sansu.org/


9961.Re: 問題
名前:あき    日付:9月18日(木) 21時24分
(x+1/x)でくくるとa+4が余ってしまうのですが・・
間違ってるんでしょうか


9974.Re: 問題
名前:boundary    日付:9月18日(木) 22時19分
では、余らないようにするためには?


10018.Re: 問題
名前:あき    日付:9月19日(金) 19時50分
どうしたらいいんでしょうか?

9935.文章問題  
名前:なつ    日付:9月17日(水) 16時31分
高校1年です。
数学が苦手なのですこしずつ勉強しています。
問題は
AからHの8枚のコインがあり、このうち2枚は偽物のコインで、本物のコインとは重量が異なっている。
ただし、本物より想いか軽いかはわかっていない。
そこで、天びんばかりを3回使って、2枚の偽物を発見することにした。
たとえば、天びんばかりの左にA,Bの2枚、右にC,Dの2枚を載せたとき、釣り合えば(AB)=(CD),左が重ければ(AB)>(CD)、右が重ければ(AB)<(CD)と表すことにすると、3回の計測結果は次のようになった。
このとき、確実にいえるのは?
1回目(ABC)=(DEF)
2階目 (ABG)>(CDE)
3回目 (AH)<(DF)

1.偽物はAとFで、本物より重い。
2.偽物はBとEで、ほんものより重い。
3.偽物はBとFで、本物より重い。
4.偽物はCとEで、本物より軽い
5.偽物はDとFで、ほんものより軽い

私の考えでは2番だとおもうですが???
/



9936.Re: 文章問題
名前:ヨッシー    日付:9月17日(水) 17時42分
これは、消去法でいけばいいでしょう。

もし「1.偽物はAとFで、本物より重い。」が正しいとすれば、
1回目(ABC)=(DEF) と 3回目 (AH)<(DF) は、同じ結果のはずです。

もし「2.偽物はBとEで、ほんものより重い。」とすると、
1回目(ABC)=(DEF) と 2回目 (ABG)>(CDE) は、同じ結果のはずです

もし「4.偽物はCとEで、本物より軽い」とすると、
3回目 (AH)<(DF) は、つり合うはずです。

もし「5.偽物はDとFで、ほんものより軽い」とすると、
3回目 (AH)<(DF) は、 (AH)>(DF) となるはずです。

では、「3.偽物はBとFで、本物より重い。」が確実に言えるかというと、
これは、もう少し調べないといけません。
それと、問題の条件として、「ニセ物はニセ物どうしで重さが等しい」を
加えないと、例えば、
「GとHが偽物で、Gは本物より重く、Hは本物より軽い」でも良いことになります。
 
http://yosshy.sansu.org/


9937.Re: 文章問題〜追加
名前:kura(高校2年)    日付:9月17日(水) 17時57分
「ニセ物はニセ物どうしで重さが等しい」があったとして、
選択肢がなかった場合の解答です

最初に着目するのが「A」「D」です。
A・Dは全ての計測の時に用いられていますが、
不等号の向きに注意してみていくと、本物になります。。

次に着目するのが3番目の計測結果です
A・Dが本物と言うことは、3番目の計測結果はH<Fになります
HかFのどちらかが偽者となります。

「H」が偽物の場合は@で考えればG・Hが偽者になるんですが
Aの式よりG>本物で、Bの式よりH<本物になってしまい、
G=Hではなくなってしまうので、「H」偽物は本物です{これよりGも本物}

「F」が偽物の場合は@で考えればBかCが偽者になります。
Aの式よりB>Cになります。
この場合、BよりH(本物)<F(偽物)を考えているので
偽物は本物よりも重くなくてはなりません。
よってもう一つの偽物は「B」になります。

よって答えは、偽物はBとFで、本物より重い。

#間違えがあれば、ご指摘ください#


9958.Re: 文章問題
名前:ケロ    日付:9月18日(木) 20時40分
本物より重いものと軽いものの二つの重さが二つの本物の重さになる場合があれば面白いな。
と思ったけれど、この条件ではだめみたい。残念。


10041.Re: 文章問題
名前:あつこ    日付:9月20日(土) 13時47分
すいません。
ありがとうございます。

なつからあつこに変更いたします

9925.図形の問題  
名前:ackey(高2)    日付:9月17日(水) 9時19分
僊BCの二辺AB,ACをそれぞれ一辺とする正方形ABDE,ACFGを僊BCの外側に書き、EGの中点をMとする。このとき、MA⊥BCかつ2AM=BCを証明せよ

この問題がわかりません。教えてください。(学年:高2)



9929.Re: 図形の問題
名前:ヨッシー    日付:9月17日(水) 10時6分

とりあえず図だけ。
なぜ合同になるかは、証明が必要です。
いわゆる、二辺挟角ですが。

で、これが、高2的解法かというと、自信ありません。
単元とかは、指定されているのでしょうか?
 
http://yosshy.sansu.org/


9931.Re: 図形の問題
名前:高橋 道広    日付:9月17日(水) 11時7分
さすがヨッシーさんです。このような解法は知りませんでした。

高校では ベクトルか複素数でやるんでしょうね。平面図形は
単元としては存在してますが 教える高校は少ないようです。

複素数を使った解答をかきます。ベクトルでもほとんど同じです
Aを原点としてB(a+bi) C(c+di)とします
E: (a+bi)*(-i)=b-ai
G: (c+di)*i=-d+ci
M: ((b-ai)+(-d+ci))/2=((b-d)+(c-a)i)/2=((c-a)+(d-b)i)*i/2
BCを表す複素数は (c+di)-(a+bi)=(c-a)+(d-b)i
より BCを半分にして90度回転するとAMとなることがわかります。
httphttp://micci.sansu.org/


9934.Re: 図形の問題
名前:ackey(高2)    日付:9月17日(水) 15時46分
ありがとうございます。助かりました。

9922.それほど難しくないと思ったのですが・・・  
名前:みっちゃ    日付:9月17日(水) 3時24分
三角関数なのですが、直角三角形の角度と高さが判っており
その斜辺の長さおよび、底辺の長さを求めるにはどの様にすれば
よいのでしょう?
無い知恵しぼって考えたところ
高さ=sinθ*斜辺だから、斜辺=高さ/sinθと思い浮かんだのですが
間違っていましたか?お知恵を御拝借おねがいします。



9923.Re: それほど難しくないと思ったのですが・・・
名前:中川 幸一    日付:9月17日(水) 3時27分
tan θ=(対辺)/(底辺)
より, (底辺)=(対辺)/tan θ
(斜辺)=√((底辺)2+(対辺)2)
で求まると思います。

http://www3.ezbbs.net/01/k-nakagawa/

9918.高次方程式、方程式  
名前:T.T.C.高2    日付:9月17日(水) 1時40分
1)2ay2+(a3+3a+2)y+a4-a3-a+1を因数分解せよ。
2)四次方程式x4+(b-1)x3+(2b2+3b-1)x+2b+1=0を実数解をちょうど1個持つように実数bを定めよ。
3)次の2つの条件を同時に満たす整数a,bの組(a,b)をすべて求めよ。(A)2次方程式X2+aX+b=0の2つの解がともに2以上の整数である。
(B)不等式3a+2b≦0が成り立つ
これらの問題がわかりません。たすけてください。



9919.Re: 高次方程式、方程式
名前:ジャグラ 高3    日付:9月17日(水) 2時9分
とりあいず1)は一番低い次数の変数でくくり(この場合はY)
たすきがけの形のもっていけばOKです。
但し、f(a)=a4-a3-a+1=0と置いて因数分解しなければなりません。
それができれば、後は適当にすればできます^〜^


9921.Re: 高次方程式、方程式
名前:ast    日付:9月17日(水) 2時56分
2) 方程式の左辺を P(x) とします.
 4 次方程式で実数解が 1 個 というのは次の二つの場合のみです.
 [i] 実数の 4 重根をもつ. [ii] 実数の重解と共役な虚数解
# ここで,
# 「実数係数 n 次方程式が虚数解を持てばそれと共役な虚数も解」
# という事実を用いました.
するといづれの場合にも, (二次式)*(二次式) の形に因数分解できる
はずです.
そこで, このような因数分解が出来るか考えます. 幸いにも

  P(x)=(x^2-2*x+2*b+1)(x^2+(b+1)*x+1)

となるので, これで, P(x)=0 の解は記述できるわけです.
### 上の因数分解は, 力ずくでやったので, 方法は聞かないでください^^;

あとはうまいこと, [i],[ii] のいづれかに該当するように
b を決めていくと良いでしょう.


9949.Re: 高次方程式、方程式
名前:帰ってきた赤猫(旧 : Red cat)    日付:9月18日(木) 12時35分
3)
x^2 + ax + b = 0 の二つの解を α , β とおきます。
このとき、α ≧ 2 , β ≧ 2 ⇔ α - 2 ≧ 0 , β - 2 ≧ 0
であり、α - 2 , β - 2 を解とする二次方程式は
x^2 + (a + 4)x + (a + b + 4) = 0 ... (*)
となります(何故か?は考えてください)。
条件 (A) は、「(*) の解が共に 0 以上の整数である」という条件と同じです。

なお、解が整数であることから、a , b はどちらも整数でなければいけません(逆は成り立たないのでご注意!)。

#最近、早とちりして、礼を失してしまうことが多く、猛省しております。

9910.確率の問題です。お願いします  
名前:一浪です    日付:9月16日(火) 23時57分
40人のクラスで一組も誕生日が同じ人がいない確率を求めよ。ただし簡単のため1年を360日とする
解答は
40人が2人1組作るとき、その組み合わせは
{C(40,2)C(38,2)・・・C(2,2)}/20!=40!/(20!*2) 通りあり、
それぞれについて、誕生日が異なる確率は359/360だから、
(359/360)^{40!/(20!*2)}。
なのですが
僕は
359/360*358/360*・・・・・・・・・・*321/360と解答しました
何度考えても何が違っているのかわかりません
40人の誕生日が違う確率を求めればいいのではと思ってしまうんです。
ご指導お願いします。



9917.Re: 確率の問題です。お願いします
名前:ジャグラ 高3    日付:9月17日(水) 1時39分
こんばんわ、他の皆さんのように的確に指摘できませんが俺と同じ
今年受験する身ですので一緒に考えればいいなぁと思い書き込みさせて
いただきます。

まず
359/360*358/360*・・・・・・・・・・*321/360と解答しました
>おっしゃる事は良く分かります。
俺も自信がないので偉そうな事は言えませんが、AさんとBさんの誕生日
が違う確率はあくまでも359/360 であり、もし重複不可の場合なら
上の考え方は概ね合っていると思われますが、上記のような問題ですと
たまたま40人すべて重なり合わなかった確率にしかすぎないと思う
のです。上手くいえませんが、重複と言うことが鍵を握ってると思います。
詳しくはSarさんが答えてくださると思いますので(笑)


9926.Re: 確率の問題です。お願いします
名前:ヨッシー    日付:9月17日(水) 9時43分
私は、
359/360*358/360*・・・・・・・・・・*321/360
(もう少し整理すると 359!/320!360^39)
で良いと思います。

4人の人が、A,B,C,Dのうち、好きな1文字を紙に書くとき
どの2人も同じ字を書かない確率は?
を考えるとき、すべての場合は、4^4=256 通り。
そのうち4人とも違う字を書くのは、4!=24通り。
確率は、24/256=3/32 で、
(3/4)(2/4)(1/4) と一致します。

一方、
4人が2人1組を作る組合せは、
 A−B C−D
 A−C B−D
 A−D B−C
の3通り。{C(4,2)C(2,2)}/2! = 3 ですね。
それぞれについて、書いた字が異なる確率は 3/4 だから、
 (3/4)^3
って、これはおかしいでしょう?
 
ところで、
>{C(40,2)C(38,2)・・・C(2,2)}/20!=40!/(20!*2) 通りあり
の右辺、間違ってませんか?
 
http://yosshy.sansu.org/


9927.Re: 確率の問題です。お願いします
名前:Sar    日付:9月17日(水) 9時55分
ご指名ですか(笑)

さて、まず題意について確認しておきたいのですが、
「40人のクラスで2人で1つの組を20個作ったとき、どの組についても誕生日が同じ人が居ない確率」でよろしいですか。この元で進めましょう。

回答例の「(359/360)・(358/360)・…・(321/360)」は、ジャグラさんもご指摘の通り、たまたま重なり合わなかった時についてしか考えていません。具体的に言うならば、

(生徒1,生徒2),(生徒3,生徒4),…,(生徒39,生徒40)

とでも組を作った場合、生徒1と生徒3の誕生日が同日であっても題意は満たすんですが、どの2人も誕生日が違う場合しか注目していないためにこの違いが起きたんですね。

捕らえづらければ着想が繋がる程度に問題を簡単にしてみる事です。
4色のサイコロを振って2つずつの組に分けた時にどの組についても番号が同じにならない確率は? とか。

こんな感じでどうでしょう。


9928.Re: 確率の問題です。お願いします
名前:Sar    日付:9月17日(水) 10時2分
出遅れた(笑)

>ヨッシー氏

私も最初、そう解釈して「P(360,40) / 360^4で合ってるんでわ?」と思ったんですが、解答例を見る限りでは9927番の発言の様に題意を解釈すべきかなぁ、と考えたんですよね。

どうなんでしょ?


9930.Re: 確率の問題です。お願いします
名前:ヨッシー    日付:9月17日(水) 10時14分
>「40人のクラスで2人で1つの組を20個作ったとき、どの組についても誕生日が同じ人が居ない確率」
おそらくこうではないかと、思うのですが、どうも、当初の記事の解答に、結びつかなくて。

再考します。
 
http://yosshy.sansu.org/

9902.分数不等式  
名前:formula    日付:9月16日(火) 22時11分
分数不等式の解き方が分かりません…
逐一グラフを書かないと解けないのでしょうか??

x+1>2/(x-1) 例えばこの問題を解く時…
まず(右辺の分母)=(x-1)≠0⇔x≠1⇔x<1,x>1 …@
(x+1)(x-1)-2>0
x^2-3>0
x<-√3 , x>√3 …A
@とAの共通部分をとって・・と考えたのですが、答えが
全然違いました.何故でしょう(泣)



9903.Re: 分数不等式
名前:中川 幸一    日付:9月16日(火) 22時20分
(x+1)>2/(x-1)
iff (x+1)(x-1)2>2(x-1)
iff (x-1)(x2-3)>0
iff -√3<x<1, √3<x

こんな感じでどうでしょうか?

http://www3.ezbbs.net/01/k-nakagawa/


9904.Re: 分数不等式
名前:中川 幸一    日付:9月16日(火) 22時27分
丸文字は機種依存文字なので括弧で書き直します。

(1) と (2) の範囲を考慮して考えても答えはちゃんと出ますよ!!

もう一度見直してみましょう。

http://www3.ezbbs.net/01/k-nakagawa/


9932.Re: 分数不等式
名前:formula    日付:9月17日(水) 11時54分
私の考えでも答えは出るようですが、やはり
分かりません・・・.本当に数学って苦手で(泣)
すみませんが詳細に教えていただけますか?


9933.Re: 分数不等式
名前:ヨッシー    日付:9月17日(水) 14時19分
両辺に x-1 を掛けるわけですが、これが正か負かによって、不等号の向きが
変わりますね。

x<1 のときは、 x−1は負なので、不等号が変わります。
x>1 のときは、 x−1は正なので、不等号はそのままです。
それぞれから得られるのが、-√3<x<1 と √3<x です。
 
http://yosshy.sansu.org/


9942.Re: 分数不等式
名前:formula    日付:9月17日(水) 22時34分
ヨッシーさん、本当にありがとうございました!!
大変良く分かりました!!
中川幸一さんも、違う考え方まで教えていただき、ありがとう
ございました!これから数学を得意科目にするよう励みたいと
思います.これからもよろしくお願いします.

9901.(untitled)  
名前:くりりん    日付:9月16日(火) 21時59分
こんばんは〜初めまして。
次の問題がわからないので教えて下さい。
第3項が10,初項から第3項までの和が21である等差数列について次の問いに答えよ。
(1)100を超えるのは第何項からか。
(2)100と200の間にあるすべての項の和を求めよ。
教えて下さい。



9905.Re: (untitled)
名前:中川 幸一    日付:9月16日(火) 22時29分
まずは, 初項を a, 公差を d とおいて一般項を求めてみましょう。
http://www3.ezbbs.net/01/k-nakagawa/


9939.Re: (untitled)
名前:くりりん    日付:9月17日(水) 22時9分
こんばんは!
a=4,d=3
となりました。


9943.Re: (untitled)
名前:中川 幸一    日付:9月17日(水) 22時57分
それでは,
an=1+3n
というのは導けましたよね?

(1)については教科書や問題集についていると思うので確認してみましょう。
(2)については
100<an, an<200
となる範囲を計算してみましょう。

http://www3.ezbbs.net/01/k-nakagawa/


9978.わかりました!
名前:くりりん    日付:9月18日(木) 22時43分
こんばんは。
(1)n>33,34項から
(2)4983
となりました。どうでしょうか?

9893.三角比を含む方程式の解の個数  
名前:ジャグラ 高3    日付:9月16日(火) 19時35分
こんばんわ、初歩的な質問かと思いますが分からないので
どなたか助けてください;−;

問)方程式2cos^2θ-asinθ-3=0を満たすθが0°≦θ≦180°の範囲に
ちょうど4個あるとき、定数aの値の範囲を求めよ。

cos^2θ=1-sin^2θより,またcosθをtを用いて表すと
f(t)=2t^2+at+1=0 となり 0≦t≦1の範囲でf(t)がt軸と
異なる4点で交わればよいのかと考えたのですが、
解答を拝見してみますと、異なる2点で交わればよいとあります。
0<-a/4<1,-a^2/8+1<0,f(0)=1≧0,f(1)=a+3>0より
−3<a<-2√2が答えです。

どこか根本的な所を忘れている感じです^^;
でも、いくら考えてもt軸と異なる4点で交わらないといけないのでは
ないか,の考えから離れられませんので教えてくださいー><



9894.Re: 三角比を含む方程式の解の個数
名前:花パジャ    日付:9月16日(火) 19時45分
>どこか根本的な所を...
t=sinθ

9892.極値の問題です(・o・)高3  
名前:かおり    日付:9月16日(火) 18時36分
楕円C:3x^2+4y^2=12と、直線l:x+2y=9の最短距離を求めよ。という問題が分かりません(^_^;)
条件付極値を用いるのは分かるのですが・・・
よろしくお願いします☆



9895.Re: 極値の問題です(・o・)高3
名前:花パジャ    日付:9月16日(火) 19時52分
条件付極値、てのがなんだか知りませんが
楕円C上の点(2cosθ,√3sinθ)と直線lとの距離を求めると
距離=|4cos(θ-60°)-9|/√5
となるので、θ=60°のとき最短、て出たりします


9906.Re: 極値の問題です(・o・)高3
名前:帰ってきた赤猫(旧 : Red cat)    日付:9月16日(火) 22時30分
>条件付極値を用いるのは分かる
と言うことは、高 3 は高 3 でも高専の 3 年かな?
条件付極値を求める際の有効手段(ただし高校範囲を超える)に「ラグランジュの(未定)乗数法」というのがあるのですが、聞いたことあります?
まずは学年を教えてください(高校か高専か)


9908.Re: 極値の問題です(・o・)高3
名前:中川 幸一    日付:9月16日(火) 22時39分
問題設定からして, 『Lagrangeの未定乗数法』の問題そのものにもとれますが, 高専はこのあたりの問題まで習うのでしょうか?

あまり高専の学習範囲が分からないもので(;^_^A アセアセ…

http://www3.ezbbs.net/01/k-nakagawa/


9909.Re: 極値の問題です(・o・)高3
名前:帰ってきた赤猫(旧 : Red cat)    日付:9月16日(火) 23時17分
>高専はこのあたりの問題まで習うのでしょうか
伝え聞いた話ですが、高専の 3 年生になると、大学の範囲の数学が顔を出してくるらしいです。

理屈ぬきに解法だけ書いておくと、

点 (x,y) と直線 x + 2y = 9 の距離 L は
L = |x + 2y - 9|/√5
L が最小 ⇔ L2 が最小
なので、L2 = (x + 2y - 9)2/5( = f(x,y) と置く)の、条件
g(x,y) = 3x2 + 4y2 - 12 = 0 のもとでの最小を求めればよい。

F(x,y,λ) = f(x,y) - λg(x,y) と置いて
∂F/∂x = 2(x + 2y - 9)/5 - 6λx = (2/5 - 6λ)x + (4/5)y - 18/5 = 0
∂F/∂y = 4(x + 2y - 9)/5 - 8λy = (4/5)x + (8/5 - 8λ)y - 36/5 = 0
- ∂F/∂λ = g(x,y) = 0
を解き、x , y (,λ) を求めます。後はそのときに実際極小(最小)となっていることを確かめれば O.K.

でも花パジャさんのやり方のほうが簡単な気がする…。


9920.出題者の意図とは違うのでしょうが
名前:    日付:9月17日(水) 2時50分
C上の点Pにおける接線をkとすると、
lと平行となるkのうちの一方と、lの距離が
求めるものです。

9887.代数学  
名前:エース    日付:9月16日(火) 17時13分
T:={x∈C;|x|=1},R>0:={x∈R;x>0}とおく。このとき、
(C^*:X)/(T:X)と(R>0:X)は群として同型であることを示せ。
注)Xは、演算子の積で、R>0は、R>0です。また、
C,Rは太字のC,Rです。

この問題が分からないので、よろしくお願いします。



9888.Re: 代数学
名前:エース    日付:9月16日(火) 17時22分
> T:={x∈C;|x|=1},R>0:={x∈R;x>0}とおく。このとき、
> (C^*:X)/(T:X)と(R>0:X)は群として同型であることを示せ。
> 注)Xは、演算子の積で、R>0は、R0です。また、
> C,Rは太字のC,Rです。
>
> この問題が分からないので、よろしくお願いします。


9889.Re: 代数学
名前:エース    日付:9月16日(火) 17時26分
すいません、問題を間違いました。

T:={x∈C;|x|=1},R>0:={x∈R;x>0}とおく。このとき、
(C^*:X)/(T:X)と(R>0:X)は群として同型であることを示せ。
注)Xは、演算子の積で、R>0は、R>0です。また、
C,Rは太字のC,Rです。

この問題が分からないので、よろしくお願いします。


9891.Re: 代数学
名前:ast    日付:9月16日(火) 18時31分
既に他所でほとんど解答になっている回答を得ていますよね?

回答を得ても, それにほとんど何の反応も示さない人にこれ以上回答を
しようという奇特な方がいるのでしょうか.

同一サイト内で複数のハンドルを名乗る意味も私には判りませんが.


9896.Re: 代数学
名前:帰ってきた赤猫(旧 : Red cat)    日付:9月16日(火) 19時59分
私は敢えて何も言うまいと思っていましたが、「ほぼ」決定的証拠をつかませていただきましたので、敢えて言います。

エースさん。貴方は余所で、ここの管理人さんの HN 「ヨッシー」を語っていらっしゃる。年齢から明らかに別人とわかりますが、失礼にも程があります。

余所で私が回答させていただいたものは、失礼ながら削除させていただきました。


9897.Re: 代数学
名前:帰ってきた赤猫(旧 : Red cat)    日付:9月16日(火) 20時28分
家路につきながら、回答の削除はいささか行き過ぎであったと反省しております。

ただし、複数 HN の件については、私も合点が行きません。

同じ HN で質問を繰り返すことが恥ずかしいとか、そういう理由ではない?


9898.Re: 代数学
名前:中川 幸一    日付:9月16日(火) 20時43分
IPアドレスが同じなので十中八九同一人物でしょう。
画面上にIPアドレスが表示されなくても, ソースを見ればある程度の所は表示されます。

http://www3.ezbbs.net/01/k-nakagawa/


9899.Re: 代数学
名前:我疑う故に存在する我    日付:9月16日(火) 21時15分
>画面上にIPアドレスが表示されなくても, ソースを見ればある程度の所は表示されます。

何だ。バレてたのか!


9900.Re: 代数学
名前:帰ってきた赤猫(旧 : Red cat)    日付:9月16日(火) 21時44分
>IPアドレスが同じなので十中八九同一人物
やはりそうですかねぇ。
#勿論、9896. の発言は「IP アドレスが同一であったこと」(+ α)に基
#づいてさせていただいております。

##なお、言い訳に過ぎませんが、「回答削除」に関しては、私の回答の
##後に返信がなかったことを確認の上、行っております。その際、read
##only の第三者が存在した可能性は否定できず、結果として不適切な
##行為に走ってしまったことは言い逃れるべくもありません。

>バレてた
いや、私にはさっぱり…。


9912.Re: 代数学
名前:ナックラー    日付:9月17日(水) 1時1分
すみませんが、私もこの問題が分からないので、
解答をお願いしたいのですがよろしいでしょうか?
話の内容からすると、みなさんはこの方に解答を
お教えするのを拒んでらっしゃるようなので、
上記のメールアドレスに解答を送っていただけないでしょうか?

ちなみに大学2年です。
なにとぞよろしくお願いします。

もし、私の発言で不愉快な気分になられた方がおられましたら
お許しください。


9913.Re: 代数学
名前:中川 幸一    日付:9月17日(水) 1時10分
もう別の掲示板で Hint をもらっているのでは?

PSもしかしてほっちゃんファン?

http://www3.ezbbs.net/01/k-nakagawa/


9914.Re: 代数学
名前:ast    日付:9月17日(水) 1時14分
さてさて, proxy なのか何なのか. いづれにしても下のやり取りから
すると, もう回答したくないと思うのですよね.

代数学(群、環)  From:ナックラー(大学2年)
03/09/15(Mon) 21:54:16 No. 5216 / 21 [RES]

代数学で分からない問題あるので、教えて下さい。
@T:={x∈C;|x|=1},R>0:={x∈R;x>0}とおく。このとき、
(C^*:X)/(T:X)と(R>0:X)は群として同型であることを示せ。
注)Xは、演算子の積で、R>0は、R>0です。また、
C,Rは太字のC,Rです。

ARを可換環とし、N(R):={a∈R|a:Rの巾零元}とおく。このとき
N(R)はRのイデアルであることを示せ。

BR,R'を環とし、fをRからR'への同型写像とする。このとき
fの逆写像f^-1はR'からRへの同型写像となることを示せ。
です。
よろしくお願いします。

Re1 : 代数学(群、環)  Name:(>_<)
03/09/16(Tue) 01:17:46 No. 5220 / 25

いっこめ。
実際に同型写像をつくってみてはどうですか?C^*の元を極座標表示しておいて、「ながさ」の方を対応させると、R>0への写像ができるはずです。準同型であることは高校生の教科書にかいてあるとおりですので、あとは核がTになることと全射であることをいいます。

にこめ。
まず、N(R)の元をふたつとってきたときに、足したものがベキ零元であることをいえばいいのですが、これは二項定理という定理を書いてみるとわかります。それから、N(R)の元をいっことってきたときに、それにRの元をかけたものがベキ零であることもいうのですが、こっちはもっとかんたんです。

さんこめ。
環準同型が同型であるとは、全単射であること、です。f^-1は全単射なのは集合論的な命題でかんたんなので、f^-1が準同型写像であることを証明することになりますが、これはfが環準同型であることと、全単射であることからわかります。(しめすべき式の両辺をfでうつすとどうなるでしょう?)

とかとか。

Re2 : 代数学(群、環)  Name:ナックラー(大学2年)
03/09/17(Wed) 01:02:45 No. 5242 / 21

返信ありがとうございました。
また分からない所があった時は、
よろしくお願いします。


9915.Re: 代数学
名前:ナックラー    日付:9月17日(水) 1時28分
すいません、確かに他の掲示板で同じ質問をしたのですが、
うまく答が導きだせなかったので、質問をしました。
答えてくださった方には、大変失礼なことをしたと反省して
おります。これから気をつけますので、また質問があったときは
よろしくお願いします。本当にすいませんでした。

中川 幸一 さんへ
おっしゃる通りほっちゃんファンです。


9916.Re: 代数学
名前:中川 幸一    日付:9月17日(水) 1時33分
私もこの回答者と同じ意見で, 教科書を見直して, どこからが理解できないのかを明記した方が良いと思います。

PS 以前何度かYahoo!のチャットルームでほっちゃん祭をやっていたなぁ〜。

あなたのハートにエンジェルビーム‥‥……━★

http://www3.ezbbs.net/01/k-nakagawa/


9924.Re: 代数学
名前:帰ってきた赤猫(旧 : Red cat)    日付:9月17日(水) 9時17分
>proxy なのか何なのか
の真偽のほどはさておき、やはり HN は統一した方がいい。
HN が同一なら、同一人物であろうと推定できるので、「この人はこう
いうところが苦手なんだっけ…。」とか「この人には、前にこういう説
明をしたから…。」ということを考慮しながら回答を書けるので、話が
スムーズに進みやすいと思うのです。
#逆に、他の人と HN が被ってしまったために、全くの別人が「同一人
#物?」と勘違いされることもありますが…。

質問すること自体は、恥でも何でもないので、どんどんすれば良いと思
いますよ。
>他の掲示板で同じ質問をしたのですが、うまく答が導きだせなかった
と仰るのなら、そこで回答を下さった人に、また質問をすれば良いこと
です。


9938.推察
名前:帰ってきた赤猫(旧 : Red cat)    日付:9月17日(水) 18時8分
どうやらナックラーさんは、質問主とは別人のようですね(私の推測ですが)。


9944.Re: 代数学
名前:ナックラー    日付:9月18日(木) 0時31分
帰ってきた赤猫(旧 : Red cat)さんへ

帰ってきた赤猫(旧 : Red cat)さんのおっしゃる通り
質問主は私ではありません。私と同じ問題で悩んでいる方を
見つけたのでラッキーと思い、質問をしたのです。
でも、もう1度このヒントを頼りにやってみます。
それでも分からない時はよろしくお願いします。


9945.Re: 代数学
名前:帰ってきた赤猫(旧 : Red cat)    日付:9月18日(木) 9時32分
■エースさんへ
まだ読んでいらっしゃるかはわかりませんが、私の 9896. の発言は撤回いたします。たまたま余所で使っていた HN が、ここの管理人さんのものと一致していた、というのが実情なのでしょう。
#余所の投稿と、投稿日時などを比較した上での私の推測です。
##はじめからそうしていれば…というツッコミは甘んじて受けます。

ただ、9924. でも書いたとおり、
>HN は統一した方がいい
という考えは変わっていません。やはり統一されていないと、回答者に与える印象も悪くなりかねませんから…。


9980.申し訳ないです。
名前:エース    日付:9月18日(木) 23時28分
この度は本当に申し訳ありませんでした。私のした軽はずみな行動が、たくさんの方に迷惑及び不快感を与えてしまったことに深く反省しています。お察しの通り、ある掲示板に書いた問題の解答をしていただいたのですが、そこで解答していただいたことを元に考えましたが、学校で習ったことと異なるやり方だったので、別の掲示板(ここの掲示板などです)を探し、全く同じ問題を書かせていただきました。べつに遊び半分でかいたのでなく、本当に分からなかったので、また違う解き方を教えて頂くつもりで書かせてもらったのです。誤解を招きかねないことをして申し訳なく思っています。
HNについては、マリオのキャラクター「ヨッシー」でいったのですが、ここの掲示板の管理人さんが「ヨッシー」であることに気づき、慌てて別のHNにしました。勝手に名乗ったと思われてしまったことをして申しわけなく思っています。管理人さん、申し訳ないです。本当に本当に申し訳ないです。深く反省しています。どうかお許しください。
管理人さんに対して、大変申し訳ないことをしたことを反省しています。どうか、今後も数学の問題に悩んでいる方たちを解決していただけるようお願い申し上げるとともに、この問題についてのお許しを願っています。


9991.Re: 代数学
名前:ヨッシー    日付:9月19日(金) 0時51分
許すも何もないのですが、まぁ、いろいろ勉強したと思って、
今後注意されたらいいと思います。
特に、あちこちに書いたならば、一つ残らずフォローして、
ほったらかしが無いように注意しましょう。

#私も任○堂に「コラァ」と言われたら、開き直るしかありませんが。
 
http://yosshy.sansu.org/


9993.Re: 代数学
名前:帰ってきた赤猫(旧 : Red cat)    日付:9月19日(金) 9時33分
■エースさんへ
9945. でも書きましたが、非礼を詫びねばならぬのは私の方であり、そんなに思い詰めることはないと思います。

これからも、私のような者で良ければ、お力になりたいと思っております。

■ヨッシーさんへ
>#私も任○堂に「コラァ」と言われたら、開き直るしかありませんが。
私も○谷プロに「ゴルァ」と言われたら、開き直るしかありませんね(笑)。

9886.2次方程式の問題の別解法  
名前:我疑う故に存在する我    日付:9月16日(火) 17時7分
9392 の記事から。(名前:ゆき 日付:8月27日(水) 23時58分 )

>(問)xの2次方程式 x^2 - (a - 1)*x + (a + 6) = 0 の2つの解が共に2以上であるような定数 a の値の範囲を求めよ。

管理人がグラフを用いた解答を載せていたが、別解法を。以前別の掲示板にも書いたが、少し間違いがあったので訂正もかねて。
(http://homepage.mac.com/dslender/dsma0306.html 記事3026:2次関数のグラフの応用のレス記事3131)
グラフを使わない解法がある。上記は、 (x + 2)^2 - (a - 1)*(x + 2) + (a + 6) = 0 が2つの正の解を持つ事に同値。以下略。グラフを書かずとも、式と論理(判別式、根と係数の関係)のみで厳密に解ける。

9882.9851の問題の解答  
名前:高橋 道広    日付:9月16日(火) 11時19分
数日間でかけていて このHPを見ていなかったので今日古い順に
見ていったら 解答がないのがあって 回答したら ヨッシーさんが
解いていました。私は別な方法(確率)で解答をつくっていたので
解答を作る時間を使った悔しさからここに解答を載せます。
解決したなら そのことを書いておいてくださいね。

円周をn等分します。ある点を基準にとりその点から 時計周りに
順にP1からPnとします。
今点P1と点Pkを結び弦をひきます。この弦に対し 点P(k+1)から
点Pnのうちの任意の点Pmを結んでできる角P1PmPkを
「弦P1Pkに対する円周角」ということにします。するとkがn/2より
大きい整数であるときのみ弦P1Pkに対する円周角が鈍角である
ことがわかります。
このとき弧P1P2を長さ1として円周は 順に長さk,m-k+1,n-mとなって
いるので(k,m-k+1,n-m)とかくことにします。
たとえば n=7のときは k=4 m=5とすると 
円周は(4,1,2)となります。
最初の数がn/2を越える整数なら三角形P1PkPmは鈍角三角形になります。

つまりこの問題はnを3つの正の整数に分けたとき はじめの数がその半分より大きくなる
ような分け方は何通りあるか という問題になります。
n=2s とすると(偶数のとき) (s+1+a,1+b,1+c)とすると 
a+b+c=s-3でありa,b,cが0以上の整数になる場合の数ですから
C(s-1,2)=(s-1)(s-2)/2となります。(記号はコンビネーション)
n=2s+1 とすると(奇数のとき) (s+1+a,1+b,1+c)とすると 
a+b+c=s-2でありa,b,cが0以上の整数になる場合の数ですから
C(s,2)=s(s-1)/2となります。

よって nが偶数のときは s=n/2を代入して (n-2)(n-4)/8
 奇数のときは s=n-1/2を代入して (n-1)(n-3)/8
ただしこれはP1から考えたものですから 
実際はn倍して(P1から Pnまであるので)
nが偶数のときは n(n-2)(n-4)/8 奇数のときは n(n-1)(n-3)/8
となります。
httphttp://micci.sansu.org/

9880.代数学  
名前:ピーチ(大学2年)    日付:9月16日(火) 11時4分
H,Kを群Gの部分群、|H|=m,|K|=nとする。mとnが互いに素のとき、H∩K={e}となることを示せ。

この問題が分かりません。
よろしくお願いします。



9883.Re: 代数学
名前:帰ってきた赤猫(旧 : Red cat)    日付:9月16日(火) 11時44分
x ∈ H ∩ K としましょう。
|H| = m なので、x^m は e になります(何故でしょう?)。
同様に、|K| = n なので、x^n も e になります。

ここで、
m と n が互いに素 ⇔ ms + nt = 1 となる整数 s , t が存在する
を使うと、
x = x^(ms + nt) = {x^(ms)}{x^(nt)}
= {(x^m)^s}{(x^n)^t} = (e^s)(e^t) = e (q.e.d.)


9884.Re: 代数学
名前:我疑う故に存在する我    日付:9月16日(火) 12時13分
別解。H ∩ K は、 H, K の部分群故、その位数は、|H|, |K| の約数。よって 1.

9866.こんにちは。  
名前:味噌汁    日付:9月15日(月) 20時0分
行列の計算で、(2行1列)×(1行2列)の計算は不可能なのでしょうか?



9868.Re: こんにちは。
名前:ast    日付:9月15日(月) 20時16分
その問題も, 先の投稿におけるえいぶさんの御回答に含まれますが.


9869.Re: こんにちは。
名前:味噌汁    日付:9月15日(月) 20時46分
そうでした…。
2行2列でした。
えいぶさん、大変失礼いたしました。どうもありがとうございます。
astさん、教えてくれてどうもありがとうございます。


9870.Re: こんにちは。
名前:味噌汁    日付:9月15日(月) 20時52分
確認したいのですが、
(a)(c d)
(b)
↑   ↑
(2行1列)×(1行2列)

(ac ad)
(bc bd)

(2行2列)
で合っていますか?


9876.Re: こんにちは。
名前:ast    日付:9月15日(月) 22時27分
OK です.


9878.Re: こんにちは。
名前:味噌汁    日付:9月15日(月) 23時51分
どうもありがとうございました。

9861.(untitled)  
名前:formula    日付:9月15日(月) 18時23分
ご説明ありがとうございます.特にえいぶさん、
2度もありがとうございました.数学は本当に駄目で・・
今から頑張って見ようかと思います.

9856.続けてすみません.もう1つ質問です.  
名前:formula    日付:9月15日(月) 18時4分
下では2重投稿を失礼しました.
媒介変数のθの消去の仕方が良く分かりません。
x=1/(1+t^2) , y=t/(1+t^2) (t>0)のtの消去を行いたいのですが・・
x^2+y^2=x となる所までは良いのですが、
その後、解説ではいきなり{x-(1/2)}^2+y^2=1/4 (x>0, y>0)
となっていたのですが・・さっぱり分かりません.どうか
よろしくお願いします.媒介変数を消去するにあたっての
コツがあるのでしょうか?それとも各方程式によって異なるのか
さっぱり分かりません・・



9859.Re: 続けてすみません.もう1つ質問です.
名前:田村 正和    日付:9月15日(月) 18時12分
ただ右辺のxを左辺に持ってきて円の方程式の形にしただけですが


9860.Re: 続けてすみません.もう1つ質問です.
名前:えいぶ    日付:9月15日(月) 18時14分
x=1/(1+t^2),y=t/(1+t^2)
でt>0なのでx,yは0より大きいということを頭に置いておきます。

x^2+y^2=x 
ここまできたらxについて平方完成することを考えます。
平方完成するとどんな移動をしたか分かり易くなります。
右辺のxを移項して
x^2-x+y^2=0
両辺に1/4を足して
x^2-x+1/4+y^2=1/4
で因数分解すると
{x-(1/2)}^2+y^2=1/4
この式が直交座標でグラフを表すならば中心が(1/2,0),半径1/2の円、となります。

9853.微分の連続性について(高2です)  
名前:formula    日付:9月15日(月) 17時57分
f(x)がx=aで微分可能ならば、f(x)はx=aで連続.
しかしこの逆は成り立たないそうです.
参考書ではf(x)=|x|を例に、この関数は(-∞ , +∞)で
連続ではあるがx=0で微分可能ではない.と説明していたのですが、
そこが分かりません.
実際にグラフも書いてみましたし、
lim f(x)=f(0)となると思ったのですが・・グラフの概形からみても
x→0
+方向、−方向のどちらから近づいても0に近づいていると思うのですが.
基礎から分かっていないようなので、どなたか詳細に説明をおねがいします.



9855.Re: 微分の連続性について(高2です)
名前:えいぶ    日付:9月15日(月) 18時3分
y=|x|においてx→0としたとき0に近づくのはf'(x)ではなくf(x)の値ですね。
f'(x)はx<0のとき-1,x>0のとき1となるので右極限と左極限の値が違うので極限が存在しない、ということになります。


9857.Re: 微分の連続性について(高2です)
名前:えいぶ    日付:9月15日(月) 18時5分
日本語おかしいですね(笑
f'(x)はx<0のとき-1,x>0のとき1となるので右極限と左極限の値が違うので極限が存在しない、ということになります。

f'(x)はx<0のとき-1,x>0のとき1となり右極限と左極限の値が違うので極限が存在しない、ということになります。


9858.Re: 微分の連続性について(高2です)
名前:formula    日付:9月15日(月) 18時5分
なるほど!!なんだか目からウロコな感じです・・
大きな勘違いをしていました.本当にありがとうございました!

9851.すいません。改めて質問させていただきます   
名前:ジョニー・デプ子    日付:9月15日(月) 15時42分
「正n角形の頂点から3つ選んでできる鈍角三角形の数は?」という質問です
順列(パーミテーション)もしくは組み合わせ(コンビネーション)を使うんだと思うんですけど。
三角形の総個数は授業でやったんですけど、鈍角三角形に関しては触れてないもので;;;よければ教えてください(-L-;



9862.Re: すいません。改めて質問させていただきます 
名前:えいぶ    日付:9月15日(月) 18時27分
ヒント。
例えば12角形において向かい合う点(時計で言えば12時と6時の方向)を選ぶと残りはどこを選んでも直角三角形になります。
向かい合わない点(例えば12時と4時)を選んだとき鈍角三角形になるのは1時の点、2時の点、3時の点のみ。


9863.Re: すいません。改めて質問させていただきます 
名前:ヨッシー    日付:9月15日(月) 18時56分
実は、順列、組合せではなくて、数列の和の範疇です。

私のページの「ご質問に答えるコーナー」に解答を載せました。
 
http://yosshy.sansu.org/


9875.Re: すいません。改めて質問させていただきます 
名前:ジョニー・デプ子    日付:9月15日(月) 22時16分
ふむふむ・・・(・∂・)φ≠りがとうございました!!

9849.こんにちは。またお願いします…。  
名前:味噌汁    日付:9月15日(月) 14時32分
行列の計算で、(1行2列)×(2行1列)の計算は不可能なのでしょうか?



9850.Re: こんにちは。またお願いします…。
名前:田村 正和    日付:9月15日(月) 15時2分
可能ですよ。


9854.Re: こんにちは。またお願いします…。
名前:えいぶ    日付:9月15日(月) 17時58分
l行m列とm行n列の積は可能でl行n列になります。
ただし順番を変えてはいけない。


9865.Re: こんにちは。またお願いします…。
名前:味噌汁    日付:9月15日(月) 19時59分
ありがとうございました。


9867.Re: こんにちは。またお願いします…。
名前:味噌汁    日付:9月15日(月) 20時1分
すみません…質問を間違えた可能性があるので、同じような質問を上でもう一回…^^;

9848.明日は実力テスト・・・憂鬱だ・・・  
名前:IGA    日付:9月15日(月) 12時10分
確率の問題なんですけど・・なぜか私なぜか『同時に取り出す』を
『連続する』に見立てて計算してしまうのです。しかしそれでも答えは合っているのですが・・・
つまり、例題で考えますと・・
●袋の中に、白球3個と赤玉4個が入っている。この中から2個の玉を同時に取り出すとき、少なくとも1個は白玉である確率を求めよ。

という問題があるとします。そしたら私は『同時に取り出す』を
『連続する』にみたてるのです。
つまり樹形図をかいてあ〜だこ〜だやれば全体の起こりえる場合の数は『42』とでてきます。あとは30通りてでてきて・・
答えは5/7とでてきます。一応あっているのですが・・
『同時に取り出す』を
『連続する』にみたてるのは果たしていいのだろうか?
うむ・・・是非をお願いします。



9864.Re: 明日は実力テスト・・・憂鬱だ・・・
名前:IGA    日付:9月15日(月) 19時54分
全体の起こりえる場合の数は
7*6/1*2
の意味を教えてくださいませんか?
お願いします。m(_ _)m


9871.Re: 明日は実力テスト・・・憂鬱だ・・・
名前:ケロ    日付:9月15日(月) 21時3分
このような問題に限っていえばということにしてもらいます。
連続を問題にする場合は、連続を入れなければなりませんが、連続を問題にしない場合は、どちらでもいいのではないかと思います。連続で考えると場合の数が増えるだけで確率は同じだと思います。
7*6は、連続的に考えて、一つ目を選ぶ方法は7、そのそれぞれで二番目を選ぶ方法は、一つ取ってしまったので6という意味。掛ければ出ます。2*1は連続的に考えたため、同じものを重ねて数えてしまった数。たとえばABとBAは連続でない場合は同じなのに、違うものとして数えてしまっています。この場合は2個なので、連続的に考えると一番目が2種類、二番目が一種類で2*1です。どれも二回ずつ余分に数えてしまったので、2*1で割っています。


9873.Re: 明日は実力テスト・・・憂鬱だ・・・
名前:IGA    日付:9月15日(月) 21時41分
なるほど!ふむ・・・わかりました。
しかし、1*2
の1は必要ないのでは?うむ・・・
あと一度に三つだすとしたら、
式は1*2*3になるのでしょうか?
うむ・・・
この二つの質問にお答えしてくだされば幸いです。


9874.Re: 明日は実力テスト・・・憂鬱だ・・・
名前:IGA    日付:9月15日(月) 21時49分
1*2の意味がわかりました。(文をよく読んでなかったのです。すいません)
うむ。
解決しました!
有り難うございました!!
<(_ _*)> アリガトォ

9826.本当にお世話になります  
名前:もう少しで期末    日付:9月15日(月) 0時25分
なんどもすいません
失礼ですが二問一気に質問させていただきます
問題1、グラフが二点(1、1)(4、4)を通りx軸に接する二次関数を求めよ
解説のほうにはy=a(x−p)二乗とおきa、pを決定する  とあり、はっきり言ってまったく分かりません。
問題2、y=x二乗のグラフとy軸上の点(0、−1)を通る直線とが接するとき、その直線の傾きを求めよ。またそのときの接点の座標を求めよ  雰囲気は分かるのですが(当たり前)・・・



9827.Re: 本当にお世話になります
名前:もう少しで期末    日付:9月15日(月) 0時26分
どうかお願いしますM(___)M


9830.Re: 本当にお世話になります
名前:中川 幸一    日付:9月15日(月) 0時30分
問題1.
y=a(x-p)2 の頂点の座標は (p, 0) です。
これが理由です。

問題2.
まず初めに学年は高2でいいですか?

http://www3.ezbbs.net/01/k-nakagawa/


9831.Re: 本当にお世話になります
名前:もう少しで期末    日付:9月15日(月) 0時31分
あ、すいません
学年は高一です


9832.Re: 本当にお世話になります
名前:中川 幸一    日付:9月15日(月) 0時33分
問題2.
y=x2, y=ax-1 を連立させて考えます。
x2-(ax-1)=0 を考えます。

http://www3.ezbbs.net/01/k-nakagawa/


9834.Re: 本当にお世話になります
名前:もう少しで期末    日付:9月15日(月) 0時38分
少し考えて見ます


9835.Re: 本当にお世話になります
名前:もう少しで期末    日付:9月15日(月) 0時44分
すいません1、2とともによく分かりません
もう少しヒントをいただけないでしょうか?教えていただいているのにこんな事言ってすいません


9836.Re: 本当にお世話になります
名前:中川 幸一    日付:9月15日(月) 0時47分
問題1.
例えば, y=2(x-1)2+3 の頂点の座標は何でしょう?

問題2.
y=f(x), y=g(x) の共通解は, f(x)-g(x)=0 の解というのは分かりますか?

http://www3.ezbbs.net/01/k-nakagawa/


9837.Re: 本当にお世話になります
名前:もう少しで期末    日付:9月15日(月) 0時51分
問1、頂点(1、3)
問2、はい分かります 両方ともyなわけですから同じのを引いて0ということですよね


9838.Re: 本当にお世話になります
名前:中川 幸一    日付:9月15日(月) 0時55分
問題1.
それでは,
y=a(x-p)2+q の頂点の座標は何でしょう?
y=a(x-p)2 の頂点の座標は何でしょう?

問題2.
点(0, -1) を通る一次関数は何でしょう?
傾きは a としてみてください。
f(x)=x2, g(x)=(上で求めた関数)
とするとどうなるでしょう?

http://www3.ezbbs.net/01/k-nakagawa/


9839.Re: 本当にお世話になります
名前:もう少しで期末    日付:9月15日(月) 1時0分
問1、(p、q)
   (p、0)
問2、y=ax−1
   ax二乗−x? ←よく分からないです


9840.Re: 本当にお世話になります
名前:中川 幸一    日付:9月15日(月) 1時4分
問題1.
これで x 軸に接する二次関数は y=a(x-p)2 とおけることは分かりましたね?

問題2.
g(x)=ax-1 であっています。
よって,
f(x)-g(x)=x2-(ax-1)
となるのはもう分かりましたか?

http://www3.ezbbs.net/01/k-nakagawa/


9841.Re: 本当にお世話になります
名前:もう少しで期末    日付:9月15日(月) 1時8分
はい、大体分かりました!!わざわざ教えていただき本当にありがとうございます!!学校より分かりやすかったです
本当にありがとうございましたM(____)M


9842.Re: 本当にお世話になります
名前:中川 幸一    日付:9月15日(月) 1時10分
どういたしまして。

私の説明で理解して貰えて光栄です。

http://www3.ezbbs.net/01/k-nakagawa/


9843.Re: 本当にお世話になります
名前:もう少しで期末    日付:9月15日(月) 1時21分
いえいえ本当に分かりやすかったです
わざわざここまでして頂きありがとうございました
また分からないところがあればお願いします


9844.Re: 本当にお世話になります
名前:中川 幸一    日付:9月15日(月) 1時22分
こちらこそ宜しくお願いします。
http://www3.ezbbs.net/01/k-nakagawa/

9812.またもや失礼します。  
名前:IGA    日付:9月14日(日) 23時53分
x=7+√3,y=7-2√3のとき、√xー√y/√x+√y+√x+√y/√xー√yの値を求めなさい。
というのですが
とりあえず有理化してx−y/x+y+1となったのですが、あっているのでしょうか?(このまま計算を続けると答えと合わないのですが)
というかこの問題は有理化してからやるというやり方でいいのでしょうか?



9814.Re: またもや失礼します。
名前:IGA    日付:9月14日(日) 23時55分
みにくいので括弧をつけます。
(√xー√y/√x+√y)+(√x+√y/√xー√y)


9815.Re: またもや失礼します。
名前:T兄弟    日付:9月14日(日) 23時55分
どこまでが分子でどこまでが分母か()をつかって表さないとわかりにくいです。


9816.Re: またもや失礼します。
名前:T兄弟    日付:9月14日(日) 23時57分
2(x+y)/(x-y)となりましたが、どうでしょう?


9817.Re: またもや失礼します。
名前:中川 幸一    日付:9月14日(日) 23時58分
{(√x-√y)/(√x+√y)}+{(√x+√y)/(√x-√y)}
=2(x+y)/(x-y)

ここまでは分かりますか?

http://www3.ezbbs.net/01/k-nakagawa/


9818.Re: またもや失礼します。
名前:帰ってきた赤猫(旧 : Red cat)    日付:9月14日(日) 23時59分
>√xー√y/√x+√y+√x+√y/√xー√y
(√x - √y)/(√x + √y) + (√x + √y)/(√x - √y)
のことでしょうかね?だとすると、有理化すると
(√x - √y)2/(x - y) + (√x + √y)2/(x - y)
= {(√x - √y)2 + (√x + √y)2}/(x - y)
= 2(x + y)/(x - y)
となります(つまり、有理化する計算の途中に間違いがある、ということです)。

「有理化してから計算する」という考え方自体はあっています。


9823.Re: またもや失礼します。
名前:IGA    日付:9月15日(月) 0時15分
すいません。私は基礎がなっていないせいか・・
(√x - √y)2/(x - y) + (√x + √y)2/(x - y)
まで導くまでがなぜかわかりません。
うむ・・・すいません・・・
お手数をおかけします。


9825.Re: またもや失礼します。
名前:中川 幸一    日付:9月15日(月) 0時22分
{(√x-√y)/(√x+√y)}+{(√x+√y)/(√x-√y)}
{(√x-√y)2+(√x+√y)2}/{(√x-√y)(√x+√y)}
=2(x+y)/(x-y)

http://www3.ezbbs.net/01/k-nakagawa/


9833.Re: またもや失礼します。
名前:IGA    日付:9月15日(月) 0時35分
わかりました!!!
Wow!
それぞれ左から順に・・
√xー√y  √x+√yをかければできますね。
ありがとうございました。
T兄弟さん 中川さん 帰ってきた赤猫(旧 : Red cat)さん
m(_ _)m<(_ _*)> アリガトォ

9806.よろしくお願いします。  
名前:IGA    日付:9月14日(日) 23時39分
√7と5−√7のそれぞれの小数部分をかけあわせるとなにになるか?
という問題です。

まず√7の整数部分は2ですよね。
だから5−√7は5−2で3。

それで私の記憶が確かならばAの小数部分はAーAの整数部分だったようなきがするのですが・・・裏覚えだったら指摘してください。
お願いします!



9807.Re: よろしくお願いします。
名前:IGA    日付:9月14日(日) 23時45分
あ!すいません。間違えに気がつきました。
5−√7の整数部分は2ですね。うむ・・
しかしなぜ5−√7はの整数部分は2になるのでしょうか?
『5ー』とは影響しないのでしょうか?


9810.Re: よろしくお願いします。
名前:T兄弟    日付:9月14日(日) 23時51分
√4<√7<√9
つまり
2<√7<3
なので
5-3<5-√7<5-2
つまり
2<5-√7<3
で5-√7の整数部分は2となります。


9811.Re: よろしくお願いします。
名前:田村 正和    日付:9月14日(日) 23時53分
3>5−√7>2をといてみましょう。


9819.Re: よろしくお願いします。
名前:IGA    日付:9月14日(日) 23時59分
なるほど
みなさん有り難うございます。
すいません。ごく単純な質問で・・・
これからもよろしくお願いします!

9805.お世話になってます  
名前:もう少しで期末    日付:9月14日(日) 23時37分
何度もお世話になってます
次の性質を持つ放物線の方程式を求めよ
二点(−1、2)(1、2)を通り、軸がx=−1のもの

自分でやってみたのは
y=p(x+n)二乗+q
2=p(−1+1)二乗+q・・・@
−2=p(1+1)二乗+q・・・A
@よりq=2
q=2をAを代入し
−2=p(1+1)二乗+2
−4=4p
p=−1
よってy=−(x+1)二乗+2
となりました
答えをなくしてしまったんで合ってるか見ていただけないでしょうか?
もし間違えてたら(高い確率で間違ってます)どこがおかしいかご指摘お願いします



9808.Re: お世話になってます
名前:田村 正和    日付:9月14日(日) 23時47分
グラフを書けばわかると思いますがこのような放物線は存在しません。


9820.Re: お世話になってます
名前:もう少しで期末    日付:9月15日(月) 0時4分
そうなんですか?ではもう少しねばってきます


9822.Re: お世話になってます
名前:T兄弟    日付:9月15日(月) 0時7分
問題が
二点(−1、2)(1、ー2)を通り、軸がx=−1のもの
なら、
y=-(x+1)2+2
(=-x2-2x+1)
で良いでしょう。


9824.Re: お世話になってます
名前:もう少しで期末    日付:9月15日(月) 0時18分
あ・・・問題間違えてました
そのとおりですすいません
ありがとうございました
確かに上記だとありえませんね

9804.アポロニウスの円・・・の問題です。よろしくお願いします。  
名前:Toshi_高1    日付:9月14日(日) 22時54分
2点A,Bからの距離比が1:k(0<k<1)である点Pの軌跡は円になりますが、
その円の中心が線分AB上にないことを
→数式計算
→幾何的解法 
の各方法にて証明するにはいったいどのようにすればいいでしょうか?



9813.Re: アポロニウスの円・・・の問題です。よろしくお願いします。
名前:帰ってきた赤猫(旧 : Red cat)    日付:9月14日(日) 23時54分
A(0,0) , B(a,0) (a > 0) として、条件を満たす軌跡(アポロニウスの円)の方程式を求め、中心の x 座標 x_0 が 0 ≦ x_0 ≦ a と「ならない」ことを示せば良いのではないでしょうか。


9872.Re: アポロニウスの円・・・の問題です。よろしくお願いします。
名前:Toshi_高1    日付:9月15日(月) 21時5分
ありがとうございます!
色々と考えて見ます!

それではこれからもよろしくお願いします。。。


9877.Re: アポロニウスの円・・・の問題です。よろしくお願いします。
名前:ケロ    日付:9月15日(月) 22時34分
ちょっと座標の設定が違いますが、計算してみます。
A(-m,0),B(mk,0)とおくと、円の方程式(k≠0)は、
√{(x+m)^2+y^2}:√{(x-km)^2+y^2=1:k
を整理すると、
(x-km/(1-k))+y^2=(km/(1-k))^2 。
中心の座標は、
(km/(1-k),0)。
km<km/(1-k)
だから、中心は線分AB上にはない。

9792.onegaisimasuNo.2  
名前:IGA    日付:9月14日(日) 17時19分
すいません。今日塾で過去問たくさん解いたのでまたまた質問です。
ごめんなさい。

A地点からC地点までは55qあります。A地点とC地点との間にB地点があり、P君はA地点からB地点までは毎時40q、B地点からC地点までは毎時60qで進んだら、ちょうど1時間かかりました。このとき、次のかく問いに答えなさい。

P君がA地点を出発すると同時に、Q君はC地点をA地点に向かって毎時30qで出発しました。このとき、Q君がC地点を出発してから何分後にP君とQ君は出会いますか?

という問題なんですが・・まずこの二人が出会う位置なのですが、
A〜BとB〜C地点で出会うときはまったく式が異なります、
それで質問なんですがA〜BとB〜Cどっちかで出会う時はどうやって予測するのでしょうか?

ちなみに答えはB〜Cでで会うのですが・・・



9796.Re: onegaisimasuNo.2
名前:Sar    日付:9月14日(日) 19時39分
では、A〜Bで出会う、と仮定してみるとどうなりますか?
「ちょうど1時間かかりました。」の条件に気をつけて実際に答えを出してみましょう。


9798.Re: onegaisimasuNo.2
名前:IGA    日付:9月14日(日) 20時40分
うむ・・・一応やってみました。
式が間違ってたらご指摘願います。
※xは出発してからすれ違うまでの時間。(時間)
40x+30x=55を解くとx=11/14になり分になおして330/7です。
それでだいたい47分ぐらいとなります。
そうするとま〜A〜B=10qなので
B〜Cは45qで
おかしいということですか?
お願いいたします。( ´д)(´д`)(д` )


9799.Re: onegaisimasuNo.2
名前:Sar    日付:9月14日(日) 20時55分
そゆことです。

11/14(時間)後に会うんですから、時速40kmで11/14時間進んだら(分に直すと逆に混乱するのでこのまま)30kmほど進むのでAB=10kmに矛盾するんです。


9801.Re: onegaisimasuNo.2
名前:IGA    日付:9月14日(日) 21時22分
なるほど!そうですね。
ふむ・・・・
有り難うございました!!!!!!!!!!!!!!!!!!!!!!!!!!!!!!!!!!!!!!!!!!!!!!!!!!!!!!!!!!!!!!!!!!!!


9802.Re: onegaisimasuNo.2
名前:IGA    日付:9月14日(日) 21時24分
すいません。
!マークを押しすぎてしまいました。

9790.onegaisimasu  
名前:IGA    日付:9月14日(日) 17時8分
abの公約数はいくつあるかという問題なんですが・・・
ちょっと説明不足かもしれませんが・・・
なぜか指数に1を足すんだそうです。
つまりa^2b^2になるわけです。
そして指数同士を出して四つとなるらしいのです。
じゃ〜『8』という数字で考えますと・・・
8=2^3
というわけですから2^4として
4つということになります。実際計算すると
1、2、4、8、となるわけですからあってますよね。
なぜ指数に1をたし、指数同士をたすのでしょうか?
疑問です。やはりそういうふうだと覚えるしかないのでしょうか?



9794.Re: onegaisimasu
名前:田村 正和    日付:9月14日(日) 19時4分
72で見てみましょう。
72=8×9=23×32
約数は(1+2+22+23)(1+3+32)を展開するとうまいことすべて出てきますね。
ですからa,bが素数のときanbmの公約数は(n+1)(m+1)になります。これはa,b,c・・・になったときも同じです。


9795.Re: onegaisimasu
名前:IGA    日付:9月14日(日) 19時24分
有り難うございました。
感慨無量です。

9787.確率の問題 教えて下さい  
名前:高2    日付:9月14日(日) 16時57分
表に1、裏に2と書いてある硬貨を回投げて、
1回目に出る数をx、2回目に出る数をyとして、
座標平面上の点(x,y)を決める。
この試行を独立に2回繰り返して決まる2点と点(0,0)とで
決まる図形(三角形または線分)について

(1)図形が線分になる確率を求めよ。
(2)図形の面積の期待値を求めよ。但し、線分の面積は0とする。

答えは(1)3/8 (2)9/16 です。
(1)の『図形が線分になる』点の位置は分かったんですが、そこから計算の仕方が分からず、躓いてしまいました。
(2)は全く手がつけられません・・・

どうかよろしくお願いしますm(__)m



9800.Re: 確率の問題 教えて下さい
名前:ヨッシー    日付:9月14日(日) 21時2分
私のページの「ご質問に答えるコーナー」に解答を載せました。
 
http://yosshy.sansu.org/


9845.Re: 確率の問題 教えて下さい
名前:ケロ    日付:9月15日(月) 1時30分
ヨッシー師匠、リンクが張ってありませんよ。ケロケロケロ。


9846.Re: 確率の問題 教えて下さい
名前:ヨッシー    日付:9月15日(月) 1時48分
すみません。
直しました。
 
http://yosshy.sansu.org/

9785.わかりません  
名前:もう少しで期末    日付:9月14日(日) 16時6分
次の性質を持つ放射線の方程式を求めよ
3点(−1、6)(1、0)(3、2)を通るもの

自分なりにやってみたんですが答えが合いません
答えはy=x二乗−3x+2です



9786.Re: わかりません
名前:えいぶ    日付:9月14日(日) 16時55分
放物線の方程式y=ax^2+bx+cのx,yに座標の値を代入します。
(-1,6)(1,0)(3,2)を通りますから
6=a-b+c…(1)
0=a+b+c…(2)
2=9a+3b+c…(3)
の3つの連立方程式が出来ます。
(1)-(2)より
6=-2b
b=-3
(1),(3)に代入して
3=a+c…(4)
11=9a+c…(5)
(5)-(4)より
8=8a
a=1
(4)に代入して
c=2
以上より
y=x^2-3x+2


9791.Re: わかりません
名前:もう少しで期末    日付:9月14日(日) 17時11分
そういうぐあいにやるんですか!!てっきりy=p(x+n)+qにいれてやるものだと思ってむちゃくちゃやってました。
ありがとうございました!!


9793.Re: わかりません
名前:えいぶ    日付:9月14日(日) 17時46分
y=a(x-p)+qですね。これは頂点や最大値、傾きなどが分かっているとき使います。先ほどのy=ax^2+bx+cは3点の座標が分かっているときに使うと便利です。


9828.Re: わかりません
名前:もう少しで期末    日付:9月15日(月) 0時28分
なるほど!!使い分けるんですね!!本当にありがとうございました

9784.はじめまして 高1です。昨日から悩みっぱなしです。教えてください  
名前:ジョニー・デプ子    日付:9月14日(日) 15時28分
正n角形で出来る鈍角三角形の個数は?



9788.Re: はじめまして 高1です。昨日から悩みっぱなしです。教えてください
名前:えいぶ    日付:9月14日(日) 17時0分
条件が足りないような気がするのですが。

例えば
正n角形の対角線を結んだときにできる鈍角三角形の数は?

正n角形の頂点から3つ選んでできる鈍角三角形の数は?
などが考えられます。


9829.Re: はじめまして 高1です。昨日から悩みっぱなしです。教えてください
名前:ジョニー・デプ子    日付:9月15日(月) 0時29分
えーっと・・・。「正n角形の頂点から3つ選んでできる鈍角三角形の数は?」
です。順列(パーミテーション)もしくは組み合わせ(コンビネーション)を使うんだと思うんですけど。
三角形の総個数は授業でやったんですけど、鈍角三角形に関しては触れてないもので;;;よければ教えてください(-L-;


9881.Re: はじめまして 高1です。昨日から悩みっぱなしです。教えてください
名前:高橋 道広    日付:9月16日(火) 11時11分
円周をn等分します。ある点を基準にとりその点から 時計周りに
順にP1からPnとします。
今点P1と点Pkを結び弦をひきます。この弦に対し 点P(k+1)から
点Pnのうちの任意の点Pmを結んでできる角P1PmPkを
「弦P1Pkに対する円周角」ということにします。するとkがn/2より
大きい整数であるときのみ弦P1Pkに対する円周角が鈍角である
ことがわかります。
このとき弧P1P2を長さ1として円周は 順に長さk,m-k+1,n-mとなって
いるので(k,m-k+1,n-m)とかくことにします。
たとえば n=7のときは k=4 m=5とすると 
円周は(4,1,2)となります。
最初の数がn/2を越える整数なら三角形P1PkPmは鈍角三角形になります。

つまりこの問題はnを3つの正の整数に分けたとき はじめの数がその半分より大きくなる
ような分け方は何通りあるか という問題になります。
n=2s とすると(偶数のとき) (s+1+a,1+b,1+c)とすると 
a+b+c=s-3でありa,b,cが0以上の整数になる場合の数ですから
C(s-1,2)=(s-1)(s-2)/2となります。(記号はコンビネーション)
n=2s+1 とすると(奇数のとき) (s+1+a,1+b,1+c)とすると 
a+b+c=s-2でありa,b,cが0以上の整数になる場合の数ですから
C(s,2)=s(s-1)/2となります。

よって nが偶数のときは s=n/2を代入して (n-2)(n-4)/8
 奇数のときは s=n-1/2を代入して (n-1)(n-3)/8
ただしこれはP1から考えたものですから 
実際はn倍して(P1から Pnまであるので)
nが偶数のときは n(n-2)(n-4)/8 奇数のときは n(n-1)(n-3)/8
となります。
httphttp://micci.sansu.org/

9783.またもや苦戦中 (一次方程式) お願いします  
名前:アリス 中1    日付:9月14日(日) 15時9分
ある商品を定価どおりに売れば、一個につき45円の利益がある。これを定価の1割5分引きで8個売るのと、一個につき定価から35円値引きして12個売るのとは、その利益が等しくなるという。この商品の定価を求めよ。



9797.Re: またもや苦戦中 (一次方程式) お願いします
名前:ヨッシー    日付:9月14日(日) 20時18分
一次方程式による文章題の解き方の手順は
私のページの「お父さん向け数学テキスト」にあるとおりです。

「定価を求めよ」なので、定価をx円とします。
すると、原価(仕入れ値)は、x−45円です。
1割5分引きで売った売値は0.85x円、1個あたりの利益は0.85x−(x−45)円です。
8個売ると利益はその8倍です。・・・・(ア)
定価から35円引くと売値はx−35円、1個あたりの利益は(x−35)−(x−45)円です。
12個売ると利益はその12倍です。・・・・(イ)
(ア)と(イ)が等しいという式をつくって、解けばいいです。

答えは200円です。
 
http://yosshy.sansu.org/


9803.Re: またもや苦戦中 (一次方程式) お願いします
名前:アリス 中1    日付:9月14日(日) 22時6分
ありがとうございます。いろんな問題を解いて慣れることが大事だと思い、たくさん問題を解いているのですが、すんなり解けるものもある反面、時々あれ?なんで?と いつまでも引っかかって解けないものが出てきます。勘違いしていたりすることもあるので、ヒントや解説をいただくと とても助かります。頑張りますので、これからもいろいろと教えてください。

9778.ベクトル  
名前:味噌汁    日付:9月14日(日) 12時26分
こんにちは。((OB→)の二乗)は|OB→|^2としていますが、よいのでしょうか?お願いします。



9780.Re: ベクトル
名前:Sar    日付:9月14日(日) 13時34分
ベクトルは数ではないので単純に二乗できるものではないんですが…
もし内積について仰っているのでしたら、今一度内積の定義を教科書などでご覧下さい。いや、|OB→|^2で合っているんですけどね(^^;


9781.Re: ベクトル
名前:味噌汁    日付:9月14日(日) 13時49分
どうもありがとうございました。^^


9782.Re:あぶり出し発見!!
名前:味噌汁    日付:9月14日(日) 13時53分
あぶり出しを発見しました!
ァハハ!!ミヾ(^∇^*)乂(=^∇^)ツ彡アハハ
htmlの知識でしょうか?
もしそうならば、タグをどのように入力すればよいのか教えていただけないでしょうか…?面白いので…


9821.Re: ベクトル
名前:帰ってきた赤猫(旧 : Red cat)    日付:9月15日(月) 0時5分
ソースを見てお勉強…とは言わずに今回だけ。
<font color="white">〜</font>
で出来ますよ。
#"white" のところは、背景の色に合わせればO.K.です。


9847.Re: ベクトル
名前:味噌汁    日付:9月15日(月) 11時18分
どうもありがとうございます。
ども\( ̄ー ̄ ) (  ̄ー ̄)/ども
面白いですね…ヾ(@>▽<@)ノぶぁっはははっははは♪
勉強になりました。(数学の質問でないので場違いですみませんm(__)m)


9879.Re: ベクトル
名前:我疑う故に存在する我    日付:9月16日(火) 10時12分
あぶり出しで失敗。白色の場合は問題ないが、背景が <font color="#FF87A9"> 等で指定されていた場合、文字色も同一にすると、
フルカラーディスプレーでは見えないが、256 色ディスプレーでは、何故か色が微妙に異なり、字が読めてしまった。


9911.Re: ベクトル
名前:味噌汁    日付:9月17日(水) 0時42分
なるほどです…

9776.ベクトル  
名前:かえで    日付:9月14日(日) 11時1分
平面上に点Pと三角形ABCがある。2PA・PB=3PA・PCを満たす点Pの軌跡を求めよ。   
(PA、PB、PCの上には「→」がつきます。) 高1



9789.Re: ベクトル
名前:ケロ    日付:9月14日(日) 17時6分
点Pを基準にしたベクトルで考えてみます。
↑PA=−↑AP、↑PB=↑AB−↑AP、↑PC=↑AC−↑AP。
↑AP=↑a、↑AB=↑b、↑AC=↑cと置きます。代入して整理すると、
↑a・(↑a−(3↑c−2↑b))=0 となります。
3↑c−2↑b=(−2↑b+3↑c)/(3−2)ですから、これは
BCを3対2に外分した点を表しています。
↑aと↑a−(3↑c−2↑b)の内積が0ですから、この二つは垂直。ですから、
Pの軌跡はBCを3対2に外分した点と点Aを直径とした円となります。

9770.一次方程式の問題 教えてください  
名前:アリス 中1    日付:9月14日(日) 3時31分
ある中学校の一年生は、女子が男子より10人多く、25mを泳げる人数の割合は男子では30%、女子では15%で、全体としては22%である。
男子の人数を求めよ。



9771.Re: 一次方程式の問題 教えてください
名前:中川 幸一    日付:9月14日(日) 3時50分
男子の人数を x とすると,
女子の人数は x+10 ですね。
ということは全体では 2x+10 ということになります。

あとは割合に付いての式を立てて考えましょう。

http://www3.ezbbs.net/01/k-nakagawa/


9772.Re: 一次方程式の問題 教えてください
名前:アリス 中1    日付:9月14日(日) 4時12分
ありがとうございました!こんなに早くお返事くださってとっても嬉しいです。またわからないときは よろしくお願いします。


9773.Re: 一次方程式の問題 教えてください
名前:中川 幸一    日付:9月14日(日) 4時30分
こちらこそ。
http://www3.ezbbs.net/01/k-nakagawa/

9765.ちょっとあやふやなので、教えて頂きたいです。  
名前:ジャグラ 高3    日付:9月13日(土) 23時14分
こんばんわ、また問題を解いていく中で解説を読めばなんとか分かるのです
が、完璧に理解できていないので教えていただけたらいいなぁと思い
書きコマさせて頂きます><

問)実数a,bが a^2+b^2+a+b=1を満たしている時、点(a+b,ab)は
どんな図形を書くか図示せよ。

解説を呼んでみたのですが、a^2+b^2+a+b=1を変形して
与式=(a+b)^2-2ab+a+b=1 x=a+b y=abを与式に代入して
y=1/2(x^2+x-1)ー(1)

a,bはt(媒介変数?)についての2次方程式 t^2-(a+b)t+ab=0の2つの
解である。
x=a+b y=ab, a,bはすべて実数であるから、2次方程式t^2-xt+y=0
は実数解を持つ。よって判別式Dを用いるとD=x^2-4y≧0
そのあとも解説が続きグラフの外形に移るのですが・・・そこはいいとして。

少し難しいナァと思ったのは、tとはナニを表しているか?
tの有用性がいまいち分かりにくくて困ってます><

理解に苦しみますが良問のような気がしますので、教えて頂きたいです。



9766.Re: ちょっとあやふやなので、教えて頂きたいです。
名前:ケロ    日付:9月14日(日) 1時0分
tを使ったのは、xをすでに使ってしまったので、混乱しないためだと思います。

補題 a、bが実数のとき、点(a+b,ab)の動く範囲を図示せよ。
横軸をt軸、縦軸をs軸とし、函数
s=t^2-(a+b)t+ab
を考える。
このグラフはt軸とtの値がa、bのところで常に交わる。
ここで、x=a+b、 y=abとおくと、s=t^2-xt+y。
t軸と交わるには、D=x^2-4y≧0。

上の問いはこの補題に条件a^2+b^2+a+b=1を加えたものでしょう。


9767.Re:
名前:ジャグラ 高3    日付:9月14日(日) 1時32分
ケロさん>
返答ありがとうございます。
なるほど、tを用いた理由が分かりホッとしたケロ!^〜^

9762.お願いします  
名前:もう少しで期末    日付:9月13日(土) 19時6分
高1です
次の条件を満足するp、qの値を求めよ
y=px2乗+x+qはx=2で最大値1をとる
どうかやり方を教えてください
お願いします



9764.Re: お願いします
名前:もう少しで期末    日付:9月13日(土) 23時7分
なるほど!!すごくわかりやすくてびっくりしてます!!本当にありがとうございました!!


9769.Re: お願いします
名前:arc    日付:9月14日(日) 2時46分
すみません・・・(汗


何か違うような気がしました。
(p=-1/2,q=1)



確かめてみます・・・・・。


9777.頂点の座標
名前:kura(高校2年)    日付:9月14日(日) 11時8分
(p=-1/2,q=1)だと
y=-1/2x^2+x+1
=-1/2(x^2-2x)+1
=-1/2(x-1)+3/2となってしまって違うと思うのですが…。

自分の解答を載せておきます
間違えがあるかもしれませんがその時はご指摘を!
y=px^2+x+q
=p(x^2+x/p)+q
=p{x+1/(2p)}^2+q-1/(4p)
頂点の座標は{-1/2p,q-1/(4p)}

ここでx=2で最大値1をとるに注目します

まずxの範囲が定まっていないのに
最大値があるということは
グラフが上に凸のグラフとなります
よってp<0となります!
これは考え方として捕らえてください。

上に凸のグラフのグラフとなるので
最大値は頂点の座標となります。。

-1/(2p)=2
4p=-1
p=-1/4

q-1/(4p)=1
q+1=1
q=0

9757.全国のお父さんむけ・数学テキストの 解答について ♪  
名前:サラ    日付:9月13日(土) 3時24分
はじめまして!ヨッシーさん。素晴らしいHPを拝見しカンゲキです。お恥ずかしいのですが、今度、会社でスキル試験があり、数学がでるのです。もう何十年も解いていないので、このサイトを見つけたときは、嬉しくて感動しました。せめて一次方程式までは必死にこなさないと。
そこで質問ですが、ここは問題が1〜19回までありますが・・・(うーーん難しいですぅ 笑) 答えが書いてないのですが・・・載せてはいないのでしょうか?今日は問題1を解きました。毎日これからがんばりますので、どうか今後とも宜しくお願いいたします。
お手数ですが答えがあると 助かります!41歳 会社員です。



9758.Re: 全国のお父さんむけ・数学テキストの 解答について ♪
名前:ヨッシー    日付:9月13日(土) 7時5分
ページを見ていただいてありがとうございます。

実は、解答も載せてありますが、字が白いので見えないだけです。
1.プリンタで印刷すると、薄く印字される。
これが、確実ですが、もっと手軽に
2.マウスでドラッグして、反転表示する。
で、見えると思います。
 
http://yosshy.sansu.org/

9751.お願いいたします。  
名前:IGA    日付:9月13日(土) 0時20分
Original Size: 925 x 443, 17KB

太郎さんは、ある日の放課後、スクールバスが学校前を出発すると同時に、自転車で学校前を出発し、このバスと同じ道路を通って帰宅した。バスは、学校の前を出発し、バス停Bまで行って、学校前に戻る。行きはは、バス停A,Bでそれぞれ一分間停車し、帰りは、同じ道路を学校前まで停車せずに戻るものとする。自転車とバスはそれぞれ常に一定の速さで走り、バスの速さは時速45キロメートルとする。図1を見て、あとの各問いに答えなさい。

問い
太郎さんが、戻ってきたバスとすれ違うのは、学校前を出発してから何分何秒後か?

私なりに考えました。まず学校前を出発してからx分後の学校前から太郎さんまでの距離をyqとして、
y=1/4xとだしました。(この数は前の設問にあります・・・)
そしてバスが戻ってくる式を作り、連立させればいいのですが、
バスが戻ってくる式が作れません。
お願いします。



9754.Re: お願いいたします。
名前:帰ってきた赤猫(旧 : Red cat)    日付:9月13日(土) 0時45分
まずバスの動きを追ってみましょう。
バスは、出発した後、6 km 走ってバス停 A に着きます。
ここまでにかかった時間は 6/45 時間 = 8 分.
A で 1 分停まり、再び出発してさらに 3 km 走ってバス停 B に着きます。 A を出発してから B に着くまで 3/45 時間 = 4 分.
B で 1 分停まって、ここから「用意、ドン」です。

さてここまでに 8 + 1 + 4 + 1 = 14 分経っているわけですが、このとき太郎君はどこまで進んでいるかというと、学校から
1/4 × 14 = 7/2 km
の位置まで進んでいるわけです。

この状態から始めて、バスと太郎君がいつすれ違うか、考えてみると良いでしょう。


9760.Re: お願いいたします。
名前:IGA    日付:9月13日(土) 13時11分
すいません。私なりに試行錯誤したところ・・できません。
うん・・・・
何から手をつけていいか・・・
うん・・すいません。
もう少しヒントをくださるとありがたいです。m(_ _)m


9768.Re: お願いいたします。
名前:ケロ    日付:9月14日(日) 2時8分
帰ってきた赤猫師匠の計算を参考にして、次の方程式を作ってみてください。
y+(x−バスが学校を出発して太郎君の家の前までもどるのに掛かる時間)×バスの速さ=5
これを連立させればよいと思います。


9775.Re: お願いいたします。
名前:IGA    日付:9月14日(日) 9時14分
うむ・・・とりあえず
19分30秒とでましてできました。
本当に有り難うございます!!!!!!!<(_ _*)> アリガトォ
バンザーイ!!∩(´∀`∩) (∩´∀`)∩バンザーイ!!
いや〜感激です。学校の選択数学でやっててわからずにかなり悔いが残ってたんですよ。
いや〜感謝感謝。
感慨無量です。


9779.Re: お願いいたします。
名前:ケロ    日付:9月14日(日) 13時3分
同じ答になりました。
「用意、ドン」から始めると次のような問題になります。
距離が11/2km離れている地点PとQがある。太郎君がP地点からQ地点に向かい分速1/4kmで出発し、バスがQ地点からP地点に向かい分速3/4kmで出発した。何分後に出会うか。


9809.Re: お願いいたします。
名前:帰ってきた赤猫(旧 : Red cat)    日付:9月14日(日) 23時51分
ケロさんフォローありがとうございます。

>「用意、ドン」から始めると次のような問題になります。
>距離が11/2km離れている地点PとQがある。太郎君がP地点からQ地
>点に向かい分速1/4kmで出発し、バスがQ地点からP地点に向かい分
>速3/4kmで出発した。何分後に出会うか。

この問題を元に方程式(連立にあらず)を立てると、x 分後に出会うとして、
x/4 + 3x/4 = 11/2
∴x = 11/2 (分)
つまり、5 分 30 秒後です。これに、「用意、ドン」の状態になるまでにかかった 14 分を足すことで、お分かりの通り 19 分 30 秒となるわけです。

9750.微分の基礎公式の証明  
名前:formula    日付:9月12日(金) 23時57分
はじめまして 高2です
(x^{a})'=ax^{a-1}を証明するのに対数微分法をつかって
y=x^{a}とおき 両辺の対数をとって logy=alogx
    y/(y')=a/x y'=ax^{a}/x=a^x{a-1}
としたのですが これで点は貰えますか?



9756.Re: 微分の基礎公式の証明
名前:帰ってきた赤猫(旧 : Red cat)    日付:9月13日(土) 1時12分
これでは点はもらえないでしょう。

という厳しいことは言わずに…(^^;)A。

どこが間違いかというと、log y = a log x の両辺を微分したときの
>y/(y')=a/x
の部分。正しくは
(y ')/y = a/x
とならないといけません。


9759.Re: 微分の基礎公式の証明
名前:formula    日付:9月13日(土) 8時33分
あ、書き間違いです。ありがとうございました。
このやりかたが間違ってたわけではないんですね。

9745.複素数平面  
名前:味噌汁    日付:9月12日(金) 18時59分
こんにちは。

複素数平面で、

(1)異なる3点をA(α)、B(β)、C(γ)とするとき、
3点A,B,Cが同じ直線上にある⇔(γ-α)/(β-α)は実数。

(2)AB⊥AC⇔(γ-α)/(β-α)は純虚数。

(1)(2)を示して下さい。

お願いします。



9747.Re: 複素数平面
名前:帰ってきた赤猫(旧 : Red cat)    日付:9月12日(金) 21時50分
ちょっと細工(平行移動)すれば、α = 0 のときに証明すれば良いことがわかると思います。


9748.Re: 複素数平面
名前:味噌汁    日付:9月12日(金) 22時23分
すみません…わかりません…^^;
もう少し詳しく教えていただけないでしょうか…
(汗)


9752.Re: 複素数平面
名前:ケロ    日付:9月13日(土) 0時33分
α = 0 のときに>というのは、3点A,B,Cの位置関係がどうなっているかを調べるのだから、点Aが原点になるように、三点を平行移動して、簡単な位置関係O,B′,C′にしたものを考えればよいという意味です。
(1)すると原点から見た点A′と点B′の位置だけが問題で、三点が一直線上にあるためにはβ′を実数倍すればいい。γ′になるようにするのだから、γ′=Rβ′(Rは実数)。γ′/β′=R。
(2)垂直になるようにするには純虚数を掛ければいい。γ′=riβ′(rは実数)。すると、γ′/β′=ri。
平行移動して元に戻しても位置関係は変わりません。


9753.Re: 複素数平面
名前:帰ってきた赤猫(旧 : Red cat)    日付:9月13日(土) 0時34分
A(α) , B(β) , C(γ) とあるときに、A が原点 O(0) に重なるように平行移動すると、A(0) , B(β - α) , C(γ - α) となります。
平行移動しても、
「A , B , C が一直線上にある」
「AB ⊥ AC」
という関係は変わらないので、だったら最初から α = 0 で良いじゃん!というわけ。
#ただし、この方法で証明を行うときは、α = 0 としてよい理由をきちんと書く必要があります。


9774.Re: 複素数平面
名前:味噌汁    日付:9月14日(日) 7時33分
理解できました。
どうもありがとうございました。

9739.二次関数の問題  
名前:COCO(高1年)    日付:9月12日(金) 15時30分
(問題)
x、yに関する2次式x^2−2xy+2y^2−2ay+b(ただし、a,b は定数)の最小値を求めなさい。また、そのときのx、yの値も求めなさい。

解答見ますと・・・
x^2−2xy+2y^2−2ay+b
それぞれ平方完成して
(x−y)^2+(y−a)^2−a^2+b
x−y=0、y−a=0のときに最小値−a^2+bを持つことがわかるからx=y=aのとき、最小値−a^2+b

※どうして( )^2は中身がゼロのときが最小になるのですか?説明よろしくお願いします。



9740.Re: 二次関数の問題
名前:帰ってきた赤猫(旧 : Red cat)    日付:9月12日(金) 15時59分
「何か」は実数とします。
(何か)^2 は必ず 0 または正になります。だから、(何か)^2 が一番小さいのは (何か)^2 = 0 のときです。
ところで、2 乗して 0 になるものは 0 しかありません。だから (何か)^2 = 0 と (何か) = 0 とは同じことです。よって
(何か)^2 が最小 ⇔ (何か) = 0


9743.Re: 二次関数の問題
名前:COCO(高1年)    日付:9月12日(金) 17時43分
すみません!( )^2は中身がゼロのときが最小になるは分かりましたが、なぜ( )^2は中身が最小になると、最小値が−a^2+bに決まるのでしょうか?最大値や最小値はy座標を答えれば良いんですよね、だから( )^2が最小になろうが、なろまいが関係無いんじゃないと思うのですが・・・う〜ん分りません・・・助けてください・・・


9744.Re: 二次関数の問題
名前:Sar    日付:9月12日(金) 17時53分
タイトルが「二次関数の問題」となっているので、ちょっと疑問だったのですが……

例えば、以下の例題を解いてみてください。これから何か掴めるかな?

x,yに関する2次式x^2 + y^2 + 1の最小値を求めよ。
また、そのときのx,yの値はいくらになるか。


9749.Re: 二次関数の問題
名前:ケロ    日付:9月12日(金) 23時18分
じゃあ、図形的に考えてみましょう。
y=(x-1)^2+3 ならグラフが描けて、x=1のときyが最小になります。
z=x^2−2xy+2y^2−2ay+b と置いて、空間で考えます。
xyz空間。
xとyに数を入れると、zの値が決まります。
これが空間での高さみたいなものです。
高さを最も低くするには?
z=(x−y)^2+(y−a)^2−a^2+b 
として、括弧の中が0なら高さが一番低くなります。


9755.順番に考えると良いかモナー。
名前:帰ってきた赤猫(旧 : Red cat)    日付:9月13日(土) 1時0分
★(x - y)^2 + (y - a)^2 - a^2 + b の最小値の求め方
まず、y は動かさずに x だけが動くものと思ってください。つまり、「今のところ」y は定数です。するとこれは x の 2 次関数です。これはすぐに最小値がわかります。
x = y のとき最小値 (y - a)^2 - a^2 + b
ですね。

ここで、初めて y を動かします。つまり、y を決めるごとに求まった「最小値たち」(y - a)^2 - a^2 + b の中で、一番小さいものは何か?と考えるわけです。そのようなものがあれば、それは「最小の中の最小」、言ってみれば king of kings です(笑)。
(y - a)^2 - a^2 + b の最小値、これもすぐにわかりますね。
y = a のとき最小値 - a^2 + b
これが「最小の中の最小」です。それはいつのときか?と言えば、x = y で y = a のときですから、x = y = a のときですね。

Sar さんの例
>x,yに関する2次式x^2 + y^2 + 1の最小値を求めよ。
も、全く同じ考えで出来ます。

ちなみに
>最大値や最小値はy座標を答えれば良いんですよね
というのは、y = (x の式) のように、y が x の関数になっているときです。ご質問の問題は、y も x と関係なく動くことが出来るので、理屈として合わないのです。


9761.Re: 二次関数の問題
名前:COCO(高1年)    日付:9月13日(土) 13時55分
帰ってきた赤猫サン、Sarさん、ケロさん理解できました。ありがとうございました!!

9738.2次関数  
名前:otoma高1    日付:9月12日(金) 15時23分
2次関数 f(x)=x^2+kx+3 (ただし、k は定数)について
(1) すべての x に対して、k≦f(x) であるための k の値の範囲を求めよ。
(2) -2≦x≦2 であるすべての x に対して、k≦f(x) であるための
  k の値の範囲を求めよ。

(1)は -6≦k≦2 と解けましたが、(2)の方が分かりません。
よろしくお願いします。



9741.Re: 2次関数
名前:帰ってきた赤猫(旧 : Red cat)    日付:9月12日(金) 16時16分
>-2≦x≦2 であるすべての x に対して、k≦f(x)
これは g(x) = f(x) - k = x^2 + kx + (3 - k) とおくと、
- 2 ≦ x ≦ 2 であるすべての x に対して、g(x) ≧ 0
ということと同じです。- 2 ≦ x ≦ 2 における
g(x) = x^2 + kx + (3 - k) = (x + k/2)^2 + {- (k^2)/4 + 3 - k}
のグラフの様子によって、何パターンかに場合分けが必要ですが、私にはグラフが描けないので、うまく説明出来ないかも知れませんが、その当たりはご勘弁を…。

(i) k > 4 (⇔ - k/2 < - 2) のとき
このとき、g(x) は x = - 2 のとき最小になり、その値は
g(- 2) = 4 - 2k + (3 - k) = 7 - 3k
g(- 2) ≧ 0 ⇔ k ≦ 7/3
なので、考えている範囲と共通部分がない。
(ii) - 4 ≦ k ≦ 4 (⇔ - 2 ≦ - k/2 ≦ 2) のとき
このとき g(x) は x = - k/2 のとき最小になり、その値は
g(- k/2) = - (k^2)/4 + 3 - k
g(- k/2) ≧ 0 ⇔ (k^2)/4 + k - 3 ≦ 0 ⇔ - 6 ≦ k ≦ 2
なので、考えている範囲と合わせて - 4 ≦ k ≦ 2 .
(iii) k < - 4 (⇔ - k/2 > 2) のとき
このとき g(x) は x = 2 のとき最小になり、その値は
g(2) = 4 + 2k + (3 - k) = 7 + k
g(2) ≧ 0 ⇔ k ≧ - 7
なので、考えている範囲と合わせて - 7 ≦ k < -4 .

(i) 〜 (iii) をまとめると - 7 ≦ k ≦ 2 ... (答)


9742.Re: 2次関数
名前:otoma高1    日付:9月12日(金) 17時9分
グラフ書いてみました。
おかげで理解することができました。
ありがとうございました!

9735.高次方程式  
名前:T.T.C.高2    日付:9月12日(金) 0時55分
1)3つの実数x、y、zがx+y+z=3、x2+y2+z2=9、x3+y3+z3=21を満たす。ただしx≧y≧zとする時xyzの値とx、y、zの値を求めよ。
2)aを実数とするとき方程式x4+ax2+1が実数を持たないためのaの条件を求めよ。
3)3次方程式x3−(2a+1)x2+(a2+a+3)x−3aが重解と他の解をもつように定数aの値を定めよ。
これらの問題がよくわかりません。助けてください。



9736.Re: 高次方程式
名前:田村 正和    日付:9月12日(金) 2時6分
(1)それぞれの式からx+y+z、xy+yz+zx、xyzが出せます。
at3+bt2t+ct+d=0において
x+y+z=−b/a、xy+yz+zx=c/a、xyz=−d/aだから
この係数をうまく求めてtについてとけばそれぞれがx、y、zの解になります。

(2)x2=Xとおけば与式=X2+aX+1となります。XについてといたときX>0の解があればxの解もあるわけですからxの実数解がない条件は「Xの解がないかまたはX<0の解がる」です

(3)(x+p)2(x−q)を展開して与式の方程式の係数を見比べて3つの連立方程式を立ててa,p,qを求める。


9737.Re: 高次方程式
名前:帰ってきた赤猫(旧 : Red cat)    日付:9月12日(金) 11時6分
3) 天下りですが、x = a とすると
a3 - (2a + 1)a2 + (a2 + a + 3)a - 3a
= a3 - 2a3 - a2 + a3 + a2 + 3a - 3a
= 0
となるので、方程式
x3 - (2a + 1)x2 + (a2 + a + 3)x - 3a = 0
は x = a を解に持ちます。このとき方程式は
(x - a){x2 - (a + 1)x + 3} = 0
となるので、
(i) x2 - (a + 1)x + 3 = 0 が、a でない重解を持つ
(ii) x2 - (a + 1)x + 3 = 0 が、a と a でない解を持つ
の二通りが考えられます。


9746.Re: 高次方程式
名前:T.T.C.高2    日付:9月12日(金) 19時50分
ありがとうございます。できました。

9726.お願いします  
名前:IGA    日付:9月11日(木) 18時24分
1から50までのすべての自然数の積が7^nで割り切れるような、もっとも大きな自然数nの値を求めよ。
さっぱりわかりません。
とりあえず解答を見ました。
そしたら7の倍数は7個:7^2(=49)の倍数は、1個あるから、答えは、n=7+1=8
とあるのですが・・
さっぱりわかりません。どこが分からないかといいますと
7個:7^2(=49)の倍数は、1個あるから、答えは、n=7+1=8はなにをいってるかです。
多忙の中暇がありましたらお願いします。



9727.Re: お願いします
名前:Sar    日付:9月11日(木) 19時9分
文字数節約のために掛け算の記号"×"をxと表記します。
また、視認性の点からちょっと横長になってます。

(1から50までの自然数の積)
=1x2x3x…x6x7x8x…x13x14x15x…x20x21x22x…x27x28x29x…x34x35x36x…x41x42x43x…x48x49x50
=7x7x7x7x7x7x7x7x1x2x3x…x6x1x8x…x13x2x15x…x20x3x22x…x27x4x29x…x34x5x36x…x41x6x43x…x48x1x50


なので、この数は7^8で割り切れますね。そして7は素数なので下線部は7では割り切れません。
こんな所でいかがでしょうか?


9728.Re: お願いします
名前:IGA    日付:9月11日(木) 19時36分
うむ・・・・とりあえずなんとなくわかってきたのですが・・
本文中の『この数』とはなんのとことですか?
すいません。お願いします。


9730.Re: お願いします
名前:ヨッシー    日付:9月11日(木) 20時8分
『この数』はもちろん、1から50まで掛けた数です。

 1×2×3×4×5×6
は、2で何回割れるでしょう?
素因数分解したときに、2が何回掛けられているかというのを調べればいいですが、
2の倍数が、2,4,6なので、2は3個掛けられているので、2で3回割れる。
これでいいですか?
ちがいますね。4は2×2ですから、さらにもう1回2が掛けられています。
よって、 3+1=4 4回が答えです。
 
http://yosshy.sansu.org/


9733.Re: お願いします
名前:IGA    日付:9月11日(木) 22時40分
なるほど!
Sar さん丁寧なご説明有り難うございました。
ヨッシーさん例題をだしてくれたおかげでわかりました。
<(_ _*)> アリガトォ

9720.ベクトル  
名前:マメ太郎    日付:9月10日(水) 19時45分
平面上に三角形ABCと点Pがあり、2PAベクトル+3PBベクトル+4PCベクトル=0ベクトルが成り立っている。
(1)APベクトルをABベクトルとACベクトルを用いて表せ。また、2直線AP,BCの交点をQとすると、AQベクトルをAPベクトルをもちいてあらわせ。
(2)三角形PAB,三角形PBC,三角形PCAの面積を、それぞれS₁,S₂,S₃とすると、S₁:S₂:S₃=(ア):(イ):3である。ア、イを求めよ。
(1)のとちゅうまではAPベクトル=(3ABベクトル+4ACベクトル)/2と、解けたのですが、その次が解けません。教えてください。



9721.Re: ベクトル
名前:帰ってきた赤猫(旧 : Red cat)    日付:9月10日(水) 20時43分
>(1)のとちゅうまではAPベクトル=(3ABベクトル+4ACベクトル)/2
ここがおかしいような気がします。
2→PA + 3→PB + 4→PC = →0

- 2→AP + 3(→AB - →AP) + 4(→AC - →AP) = →0
と変形すると、→AP が求まります。
→AQ については、Q が AP の延長上にあることと、BC 上にあることを
使えば求まると思います。
(2)はそれからですね。


9729.Re: ベクトル
名前:マメ太郎    日付:9月11日(木) 20時0分
APベクトル=(3ABベクトル+4ACベクトル)/9になりました。あってますか。でも、その次から解けません。教えてください。


9731.Re: ベクトル
名前:ケロ    日付:9月11日(木) 21時34分
↑AP=(3AB↑+4AC↑)/9={(3AB↑+4AC↑)/7}*(7/9)
と変形してみます。中括弧の中身は何を表しているでしょう。


9734.どっちが考えやすいか?
名前:帰ってきた赤猫(旧 : Red cat)    日付:9月11日(木) 22時40分
→AP = (3→AB + 4→AC)/9 は正解です。ここで、Q は AP の延長上にあるので
→AQ = k→AP = (3k→AB + 4k→AC)/9
と置けます。さらに、Q が BC 上にあることから
3k/9 + 4k/9 = 1
が成り立ちます。ここから k を求めると?

9714.一晩考えましたが限界です。。。  
名前:三角形の相似?    日付:9月10日(水) 15時24分
こんな問題ですみません。よろしくお願いします。
三角形OABがある。OBを3等分した点をそれぞれS,Tとする。S,TをAと結び、線分ASを2:1
に分けた点をXとする。線分OXを延長し、線分AT、ABとぶつかる点をY、Zとする。
OX:OY:OZを求めよ。



9715.Re: 一晩考えましたが限界です。。。
名前:帰ってきた赤猫(旧 : Red cat)    日付:9月10日(水) 16時34分
学年がわからないので的確なヒントになっているかは疑問ですが、
→OX , →OY , →OZ を →OA と →OB で表してみると良いと思われ
ます。
#矢印をつけたのは、ベクトルのつもりで…。


9718.Re: 一晩考えましたが限界です。。。
名前:ヨッシー    日付:9月10日(水) 17時18分
Red cat 改め、帰ってきた赤猫さん。お帰りなさい。
(ウルトラマンみたい)


Xが△AOTの重心であることは、割とすぐわかります。
すると、OX:XYも、わかります。
あとは、メネラウスの定理を2回使います。
つまり、
(AX/XS)(SO/OB)(BZ/ZA)=1 または (AY/YT)(TO/OB)(BZ/ZA)=1 から、
AZ:ZB を求め、
(OX/XZ)(ZA/AB)(BS/SO)=1 または (OY/YZ)(ZA/AB)(BT/TO)=1 から、
OX:XZ または OY:YZ を求めます。

もちろん、最初の「Xが△AOTの重心」に気付かなくても
メネラウスの定理を使えば、OX:XY や AY:YT も求められます。

答えは、10:15:18 です。
 
http://yosshy.sansu.org/


9719.Re: 一晩考えましたが限界です。。。
名前:帰ってきた赤猫(旧 : Red cat)    日付:9月10日(水) 17時59分
>ウルトラマンみたい
ちょっと狙ってました(苦笑)。

メネラウスとは思いもよらず。
ベクトルに頼ってしまう私はまだまだかな?

参考までに、ベクトルを使うとすると
→OS = (1/3)→OB で、
AX : XS = 2 : 1 だから →AX = 2→XS
∴→OX - →OA = 2(→OS - →OX)
まとめると →OX = (1/3)→OA + (2/3)→OS = (1/3)→OA + (2/9)→OB.
→OY については
→OY = s→OX = (s/3)→OA + (2s/9)→OB (O , X , Y は一直線上)
→OY = t→OA + (1 - t)→OT = t→OA + {2(1 - t)/3}→OB
から s/3 = t , 2s/9 = 2(1 - t)/3 ∴ s = 3/2 , t = 1/2.
つまり →OY = (3/2)→OX.
→OZ も
→OZ = s '→OX = (s '/3)→OA + (2s '/9)→OB (O , X , Z は一直線上)
→OZ = t '→OA + (1 - t ')→OB
から s '/3 = t ' , 2s '/9 = 1 - t ' ∴ s ' = 9/5 , t ' = 3/5.
つまり →OZ = (9/5)→OX.
よって
OX : OY : OZ = 1 : 3/2 : 9/5 = 10 : 15 : 18.
(高校生向けの模範解答?)

9711.挑戦状(解答を教えてくださいませ)  
名前:批点無き理論の渦巻き    日付:9月10日(水) 2時15分
2つの自然数列 {An} {Bn} について、以下の条件がある。

1. {An} {Bn} は、重なりなくすべての自然数を覆い尽くす

2. An = Bn + 3

この条件下で、A(100)を求めよ。



9712.Re: 挑戦状(解答を教えてくださいませ)
名前:ヨッシー    日付:9月10日(水) 6時59分
An<An+1
という条件を加えるなら、
 B1=1 A1=4
 B2=2 A2=5
 B3=3 A3=6
この連続した6つの自然数を1単位として、7〜12,13〜18・・・
を作ることが出来ます。
 B4=7 A4=10
 B5=8 A5=11
 B6=9 A6=12
・・・・・・・・・・・・・・・
 B97=193 A97=196
 B98=194 A98=197
 B99=195 A99=198
という具合です。
 
http://yosshy.sansu.org/

9703.多角形の内角の和  
名前:crane(大学生)    日付:9月9日(火) 15時45分
こんにちは。
小学5年生と多角形の内角の和を求める問題を解いていてふとぶち当たった
疑問です。

多角形(n角)の内角の和を求めるために、三角形がいくつ入っているか
を求めるのですが、三角形の数がn-2個になることをうまく説明できません。

5角形を例に取ると1つの頂点から引ける対角線の数は5-3本なので、その
対角線が交わらないように図形を分けると対角線の数より一つ多く分けら
れるから「−2」をするんだよ、と図を書いて説明しましたが、本人は「???」

皆さんなら小学生を相手にどう説明しますか?!



9704.はじめまして。
名前:えいぶ    日付:9月9日(火) 16時26分
n角形の1つの頂点aから他頂点に対角線を引くことを考えます。
このときa自身からaには勿論引けずさらにaと隣り合った頂点も引けることが出来ません。したがって対角線は交わらずに最高n-3本引くことが出来ます。
ここまでで平面をn-3本の線分で分割しました。したがって分けられた平面は合わせてn-2あるはずです。(これは例えば長方形などに線を書き込ませるなどすれば理解できると思います。長方形に何もしなければ1つだけ囲まれた形があり、1本線を引くと2つになり…あとは規則性に気付くはずです。


9707.Re: 多角形の内角の和
名前:ヨッシー    日付:9月9日(火) 17時38分
三角形 → 三角形1つ
四角形 → 三角形2つ
五角形 → 三角形3つ
六角形 → 三角形4つ
七角形 → 三角形5つ
八角形 → 三角形6つ
という、規則性では納得しませんか?
 
http://yosshy.sansu.org/


9885.Re: 多角形の内角の和
名前:crane(大学生)    日付:9月16日(火) 14時50分
とりあえず納得した様子でした。
かんがえてくださったみなさん、ありがとうございました。

9700.分数のn進法表記、あるいはn進法の除法  
名前:ほるもん(大卒・・一般市民?)    日付:9月9日(火) 13時36分
問題は
「14/3を2進数で表示せよ。」
というものです。
多分
(14/3)_10 = (4)_10 + (1/3)_10
= (100)_2 + (0.01)_2 ・・・最後の01は循環
= (100.01)_2      ・・・最後の01は循環
で合っていると思うのですが、これを
(14/3)_10 = (1110/11)_2
として割り算を実行して
= (100.10)_2      ・・・最後の10は循環
としたら違う答えになるのか???
小数をふくむ割り算は2進法ではできないのでしょうか?
整数での割り算は自分の持っている本に載ってましたが。。
よろしくお願いします。



9702.Re: 分数のn進法表記、あるいはn進法の除法
名前:ヨッシー    日付:9月9日(火) 14時25分
>(14/3)_10 = (4)_10 + (1/3)_10
ここの所が、正しくは、
 (14/3)_10 = (4)_10 + (2/3)_10
ですね。
この部分で、答えがずれてきていると思われます。
 
http://yosshy.sansu.org/


9709.Re: 分数のn進法表記、あるいはn進法の除法
名前:ほるもん(大卒・・一般市民?)    日付:9月9日(火) 20時12分
ほんとだ・・・。
失礼しましたー。
ありがとうございました。m(_ _)m

9693.This question is difficult!  
名前:IGA    日付:9月8日(月) 23時11分
Original Size: 925 x 443, 20KB

次の図で4点A,B,C,Dは円Oの円周上にあり、点A、B,C,を頂点とする△ABCはAB=ACの二等辺三角形である。また、ACは角DABの二等分線で、線分AC、BDの交点をEとする。角ABC=72°であるとき、次の問に答えなさい。

問い
弧DCの長さは、円周の長さの何倍か求めなさい。

まったくわかりません。
おそらく角度について目をつけると思うのですが・・
なにからてをつけていいかさっぱり・・
お願いします。

図が見にくい場合は拡大できます。
図が汚くてすいません。



9695.Re: This question is difficult!
名前:ケロ    日付:9月9日(火) 0時1分
△ABCで考えると、∠BACが出ます。
∠BAC=∠CAD。
すると、円周角と中心角を考えて、∠CODは?
∠CODが解れば、弧DCは円周の何倍(何分の何)が出ると思います。


9696.Re: This question is difficult!
名前:ヨッシー    日付:9月9日(火) 0時2分
つまりは、弧DCと、中心Oとで出来る扇形の中心角がわかればいいわけです。
で、∠DACは、弧DCに立つ円周角ですから、求める中心角の半分です。
しかも、∠DACは、ある角と等しくて、そのある角は、二等辺三角形の
等辺を挟む角だったりします。
 
http://yosshy.sansu.org/


9705.Re: This question is difficult!
名前:IGA    日付:9月9日(火) 16時46分
なるほど!
っということは・・
2/5倍っということですか?
つまり中心角が144°になり
144/360ってことで・・
2/5倍ですか?
お願いします


9706.Re: This question is difficult!
名前:ヨッシー    日付:9月9日(火) 17時35分
む...
ちゃいます。

知りたいのは、∠CODですよね?
だったら、調べるべき円周角は、∠CAD です。
 
http://yosshy.sansu.org/


9708.Re: This question is difficult! 私って本当にダメですね
名前:IGA    日付:9月9日(火) 18時25分
(゚ロ゚;)エェッ!?すいません。
本当にわたしはダメですね・・・はあ・・(落胆)
図に書き込んでいた数値がまちがっていました。
答えは1/5倍ですね。
おそらく・・・
多忙の中、暇があったらでいいのですが確認をお願いします。


9732.Re: This question is difficult!
名前:ケロ    日付:9月11日(木) 21時36分
同じく。

9692.すみません…もう一度…^^;汗  
名前:味噌汁    日付:9月8日(月) 22時32分
すみません。ややこしくなってきてしまったので、もう一度ちゃんと質問させていただきたいと思います。

問題は、
放物線y=ax^2上(a>0)にx座標が-2の点A、x座標が6の点Bが与えられている。
(1)点Aにおける接線が直線ABと垂直に交わるときのaの値を求めよ。
(2)点Aと点Bのあいだの放物線上に点Pをとる。直線APと放物線で囲まれる図形の面積をS1、直線PBと放物線で囲まれる図形の面積をS2をする。S1+S2の最小値とその時のS1の値を求めよ。

という問題なのですが、
(1)の答えはa=1/4です。
(2)の解答は…
曲線y=ax^2と線分ABで囲まれる面積は一定であるから(★)、S1+S2が最小になるときは△APBの面積が最大になるときである。△APBは底辺をABとすると、Pにおけるy-ax^2の接線が直線ABと平行になるとき高さが最大となるから、面積も最大になる。
よって、Pのx座標をpとすると、(1)より、
2ap=4a
p=2
このとき、直線APの方程式は、y=4aであるから、
S1=∫(-2〜2)(4a-ax^2)dx=32a/3

とあります。
(1)のAにおける接線が直線ABと垂直に交わる、と言う条件は(2)では残っているのでしょうか?残っていないような気がするのですが…

要するに、分からない所は、上の★の所の

「曲線y=ax^2と線分ABで囲まれる面積は一定であるから」

という文章です。aが定数ならば、一定だと思いますが、定数であるという断りは書いてありません。だとするなら、一定ではないと思うのですが…。もし、(1)の問題の続きであるならば、a=1/4と出ているので、定数ですが、(1)とは独立した問題ですよね?だとするなら、aは定数とはいえないので、aは変数??だと思うのです。だとすると、「曲線y=ax^2と線分ABで囲まれる面積は一定であるから」という、「一定であるから」という意味が判りません。しかし、解答にはこのように書いてあるのです。


お願いします。



9694.Re: すみません…もう一度…^^;汗
名前:ケロ    日付:9月8日(月) 23時18分
一言入れればいいかなあ。
曲線y=ax^2と線分ABで囲まれる面積は「aの値が決まれば」一定であるから、S1+S2が最小になるときは△APBの面積が最大になるときである。と。
aがどんな値を取っても(a≠0)、このように考えられると言っているのかな。
aの値によって放物線の形は変わるけど、どの放物線でも同じ方法でよい、ということだと思います。


9697.ありがとうございます。^^
名前:味噌汁    日付:9月9日(火) 0時6分
ありがとうございます。
なるほど…そういわけですか。納得できました。^^
何度もお手数をおかけしてしまい、すみませんでした。
ケロさん、それから前スレの田村 正和さん、
どうもありがとうございました。^^

9688.ベクトル  
名前:千夏(高3)    日付:9月8日(月) 21時54分
数学のワークをやっていて、そこに導くまでのやり方が
わからないので、教えてください<(_ _)>

aベクトル=(−1、−3、0)
bベクトル=(3、x、6)のとき、|bベクトル|=√7|aベクトル|
となるように、xの値を求めよ。

答えは、X=+−5


aベクトル=(−2、0、1)、bベクトル=(3,5、x)
のとき、|bベクトル|=√10|aベクトル|となるように、
xの値を求めよ。

答えは、+−4

教えてください。<(_ _)>
※ベクトルの記号がかけなかったので、文字にしました。<(_ _)>



9690.Re: ベクトル
名前:ヨッシー    日付:9月8日(月) 22時23分
|というのは、 の大きさのことで、
=(x、y、z)の大きさは
 ||=√(x2+y2+z2)
で求められます。
これに当てはめてみてください。
 
http://yosshy.sansu.org/


9691.Re: ベクトル
名前:ジャグラ 高3    日付:9月8日(月) 22時24分
成分化されたベクトルを大きさの概念を用いて表す時は、
(a,b,c)なら大きさは√a^2+b^2+c^2となるので、
aベクトルの大きさ=√(-1)^2+(-3)^2
bベクトルの大きさ=√3^2+x^2+6^2

よって√45+x^2=√70 両辺を2乗し 45+x^2=70
x^2=25で x=+ー5です。

同様にすればもう一つも解くことができますよ^〜^

9676.指数方程式なのですが。。。  
名前:ほるもん(大卒・・一般市民?)    日付:9月8日(月) 12時38分
6^x + 2^(2x+1) = 3^(2x) を解け、という問題で詰まっています。
2^x = s、3^x = tとして
st + 2s^2 - t^2 = 0
に直し、log_2(s) = log_3(t)
と連立させて解こうと思ったのですが????解けないです。
方針がまずいのでしょうか?
よろしくお願いします。



9677.Re: 指数方程式なのですが。。。
名前:fan    日付:9月8日(月) 13時31分
t/s=(3/2)x=uとおけば
u2-u-2=0となりますね。


9679.Re: 指数方程式なのですが。。。
名前:ヨッシー    日付:9月8日(月) 13時40分
fan さんの記事を見ての結果論なのですが、
u の式がきれいに因数分解できる。
 →s と t のままでも、因数分解できるはず。
というわけで、
  st + 2s^2 - t^2 = (2s-t)(s+t)
と因数分解しても、解くことができます。
 
http://yosshy.sansu.org/


9681.Re: 指数方程式なのですが。。。
名前:ほるもん(大卒・・一般市民?)    日付:9月8日(月) 14時43分
レスありがとうございます。
その後なのですが、s,t > 0であることから、
2s = t
これをlog_2(s) = log_3(t)
に代入して・・・とやっていったのですが、
複雑になるばかりで??


9683.Re: 指数方程式なのですが。。。
名前:ヨッシー    日付:9月8日(月) 20時22分
>これをlog_2(s) = log_3(t) に代入
これは、良い方法ではありません。
 2s=t
はそのまま、
 2・2x=3x
両辺 2x で割るなりして、まとめます。
 
ちなみに、
>これをlog_2(s) = log_3(t) に代入
を続けると、
 t=2s
を代入して、
 log2s=log32s
公式 logab=logb/loga (底は底となりうる任意の数)より、
 log2s=log22s/log2
 log2s=(log22+log2s)/log2
などとして解けます。
 
http://yosshy.sansu.org/


9685.Re: 指数方程式なのですが。。。
名前:ほるもん(大卒・・一般市民?)    日付:9月8日(月) 20時44分
ありがとうございました。
どちらの方法でも、
x = 1/{log_2(3) - 1}
が出てきました!
なんか、複雑そうに見えてましたが簡単にすることが
できるんですねえ。

9671.頼みますゥ゚゚(´□`。)°゚。ワーン!!解けません。(___´Д`)  
名前:IGA    日付:9月8日(月) 10時50分
(x+3)(x+2y+3)+2y-1を因数分解しなさい。
という問題なんですが、私なりにやってみたところ
まず
(x+3){(x+3)+2y}+2y-1にしてx+3をラージAとおき
A^2+2yA+2y-1までやったのですが・・・
このあと何から手をつけていいのかさっぱり・・・
お願いいたします!



9672.Re: 頼みますゥ゚゚(´□`。)°゚。ワーン!!解けません。(___´Д`)
名前:ヨッシー    日付:9月8日(月) 11時20分
公式
 x2+(a+b)x+ab=(x+a)(x+b)
を使って、例えば、
 x2+6x+8=(x+4)(x+2)
とするときの考え方は、
「掛けて8、足して6となる2数を見つける」ですね?
では、 A^2+2yA+2y-1 を解くには、
「掛けて 2y-1、足して 2y になるような2数を見つける」

もうひと息。
 
http://yosshy.sansu.org/


9675.Re: 頼みますゥ゚゚(´□`。)°゚。ワーン!!解けません。(___´Д`)
名前:IGA    日付:9月8日(月) 12時33分
なるへそ!
ということは(A+2y-1)(A+1)
になって
(x+3+2y-1)(x+3+1)
答えは上の奴ですか?


9678.Re: 頼みますゥ゚゚(´□`。)°゚。ワーン!!解けません。(___´Д`)
名前:ヨッシー    日付:9月8日(月) 13時34分
もうちょい。
99点。
 
http://yosshy.sansu.org/


9680.Re: 頼みますゥ゚゚(´□`。)°゚。ワーン!!解けません。(___´Д`)
名前:IGA    日付:9月8日(月) 14時6分
(x+3+2y-1)(x+3+1)
じゃないですね。
(x+2y+2)(x+4)
ですかね?


9684.Re: 頼みますゥ゚゚(´□`。)°゚。ワーン!!解けません。(___´Д`)
名前:ヨッシー    日付:9月8日(月) 20時23分
はい。正解。
 ◎
 
http://yosshy.sansu.org/


9686.Re: 頼みますゥ゚゚(´□`。)°゚。ワーン!!解けません。(___´Д`)
名前:IGA    日付:9月8日(月) 21時36分
Thank you.
Yosshy,thank you very much.

9658.お願いします〜  この問題を解いていただきたいのですが。  
名前:あや    日付:9月7日(日) 19時8分
2.3log(R/r)

R....100
r....37.26
宜しくお願いします



9668.Re: お願いします〜  この問題を解いていただきたいのですが。
名前:花パジャ    日付:9月8日(月) 10時8分
計算するだけなんだけど...

他の人にも役に立つかもしれないのを。
google ( http://www.google.com/intl/ja/ )で
 2.3*log(100/37.26)
と入れて検索すると計算してくれます。


9673.Re: お願いします〜  この問題を解いていただきたいのですが。
名前:ヨッシー    日付:9月8日(月) 11時38分
これは便利>>Google

注意しないといけないのは、もとの問題の log が
常用対数(底が10)か、自然対数(底がe)のどちらを言っているかと
言うことです。

Google では、
 2.3*log(100/37.26) ・・・常用対数
 2.3*ln(100/37.26) ・・・自然対数
Excel など、パソコン系のルールはおおむねこのようです。
 
http://yosshy.sansu.org/


9674.Re: お願いします〜  この問題を解いていただきたいのですが。
名前:あや    日付:9月8日(月) 12時16分
どうもありがとうございました。

log10になっていたことでちょっと迷ってしまいました。

また質問させていただくことがあると思いますが宜しくお願いします

9655.This question is difficult for me.No.2  
名前:IGA    日付:9月7日(日) 18時56分
Original Size: 925 x 443, 14KB

一辺の長さが1pの正方形を、次の図のように1段、2段、3段・・・とすきまなく、重ならないように階段状に並べていく。次の問に答えなさい。

問い
n段並べたとき、面積は何平方センチメートルになるか。nを使った式で表しなさい。



9656.Re: This question is difficult for me.No.2
名前:IGA    日付:9月7日(日) 18時57分
お願いします。
お願いします。
こういう問題のコツ見たいのを教えてください。
頼みます。


9657.Re: This question is difficult for me.No.2
名前:IGA    日付:9月7日(日) 18時59分
すいません。中三です。


9659.Re: This question is difficult for me.No.2
名前:Sar    日付:9月7日(日) 19時29分
以下、個数は1cm×1cmの正方形の個数について書いています。

1段並べたときは1個。
2段並べたときは、1段並べた時の右に2段追加したものなので、1+2=3個。
3段並べたときは、2段並べた時の右に3段追加したものなので、1+2+3=6個。

というのは分かりますね。

だから、n段並べた時の個数は、1+2+3+…+n=n(1+n)/2個なんですが、これをどう説明しましょうかねえ。
高校の知識さえあればすぐなんですが、とりあえず下の説明を見て納得頂けるかな?

--------------

■■■■■■□
■■■■■□□
■■■■□□□
■■■□□□□
■■□□□□□
■□□□□□□

この図において、□の個数を考えてみましょう。
1+2+3+4+5+6=21個、と数えればすぐですが、これは□が少ないから数える事ができたんですね。多いと数え損なうかもしれない上に面倒です。
別の考え方をしましょう。□の個数と■の個数は明らかに同じですね。
ですから、□と■の合計を出して2で割ればそれが□の個数と一致します。

もうおわかりですね?


9660.Re: This question is difficult for me.No.2
名前:IGA    日付:9月7日(日) 20時4分
ありがとうございます!
感激です。
分かった瞬間思わず叫んでしまいました。
いや〜まったなにからてをつけていいかどうかわからなかったのですが・・
ありがとうございます。
どうしてこのような考え方がすぐに思いつくのでしょうか?
やはり試行錯誤をするのですか?
それかこのような類の問題は『このような解き方がある』みたいなものがあるのでしょうか?


9662.Re: This question is difficult for me.No.2
名前:arc    日付:9月7日(日) 22時32分
規則性を見つければすぐに分かります。


9665.Re: This question is difficult for me.No.2
名前:IGA    日付:9月7日(日) 23時15分
規則性ですか・・・・
そんな簡単に見つかるものなのでしょうか?


9666.Re: This question is difficult for me.No.2
名前:ヨッシー    日付:9月8日(月) 6時43分
こちら
 1+2+・・・+100
の計算の仕方が、いろいろ載っています。

こういう問題は、高校の「数列」の中の「等差数列」という中でも出てきます。
 
http://yosshy.sansu.org/


9667.Re: This question is difficult for me.No.2
名前:IGA    日付:9月8日(月) 8時48分
なるほど!
こういう問題は解くしかないんですね。
解いて慣れるしかないんですね。
割り切ります!
有り難うございました。

9654.(untitled)  
名前:呆け人    日付:9月7日(日) 17時36分
記号の質問です。
≧と≦が上下に重なってくっついてる符号は
どういう意味を表して入るのでしょうか。



9669.Re: (untitled)
名前:ヨッシー    日付:9月8日(月) 10時15分
前後の文脈や、実際の書かれ方がわからないと、何とも言えませんが、
おそらく、
「 ・・・・、よって
 A(その記号)B
が成り立つ」
という、書かれ方はしていないと思います。
むしろ、
「A,Bについて
 A(その記号)B
を調べると、以下のようになる。
 1)A<B のとき ・・・・・・
 2)A=B のとき ・・・・・・
 3)A>B のとき ・・・・・・」
というような書かれ方ではないかと思われます。
 
http://yosshy.sansu.org/

9650.This question is difficult for me.  
名前:IGA    日付:9月7日(日) 16時5分
関数y=ーx^2について
xがkから4だけ増加するとき、変化の割合はー8となった。このようなkの値を求めなさい。

それでこの類の問題はじゅくでならいまして・・・

y=ax^2の関数について答えよ
塾の場合xがmからnまで増加するとき
a(m+n)=変化の割合
と習ったんです。それで私もそのとおりやってみたところ・・・
-1(k+4)=-8となるわけです。
そしてこの方程式をとくとk=4になります
ところがどっこいこたえではk=2なのです。
いったいどこが違うのでしょうか?



9651.Re: This question is difficult for me.
名前:IGA    日付:9月7日(日) 16時7分
できれば
y=ax^2の関数について答えよ
塾の場合xがmからnまで増加するとき
a(m+n)=変化の割合

↑↑このやり方で教えてくださるとありがたいです。
もしかしたらこのやりかたできなかったりするのですか?
おねがいします。


9652.Re: This question is difficult for me.
名前:ジャグラ 高3    日付:9月7日(日) 17時16分
こんにちわ、α(m+n)=変化の割合の意味が少しあやふやなのでしょう。
凅(xの変化量)はkからk+4へ変化したことになります。
つまり塾で教えられた方法で解きますと、-1(k+k+4)=-8
よってK=2となります。
余計なお世話になるかと思いますが、塾、参考書色々な所から色々な解法
を学ばれるかと思いますが、その際に注意して意味を把握していきましょう
私も不十分ですが、それが一番いいと思いますねー。


9653.Re: This question is difficult for me.
名前:IGA    日付:9月7日(日) 17時26分
ありがとうございます!
日本語をよく読んでなかったようです。
xがkから4だけ増加するときとはxがkから4まで増加するとは全く違う意味ですね。は〜(汗
やばい・・受験生なのにこんなことでは・・
あと数ヶ月しかないっていうのに(汗
ジャグラさんのアドバイスに従います。
ありがとうございます<(_ _*)> アリガトォ

9645.こんにちは  
名前:味噌汁    日付:9月6日(土) 17時53分
放物線y=ax^2上にx座標が-2の点A、x座標が6の点Bが与えられているとき、この放物線と線分ABで囲まれる面積は一定であるのはなぜでしょうか?

お願いします。



9647.Re: こんにちは
名前:田村 正和    日付:9月6日(土) 20時56分
a<0、a>0の2通りありますが結局x軸に対称なのでa>0で考えて見ましょう。
直線ABの方程式はy=4x+4a+8なので
求める面積は∫(−2〜6)4x+4a+8−x2
ここでaが変わると面積も変わりますね。よってaに関して一定ではありません。時刻tに関して一定ではありますが。


9648.すみません・・・もう一度お願いします。
名前:味噌汁    日付:9月6日(土) 23時46分
すみません。戻ってきてしまいました。

問題は、
放物線y=ax^2上にx座標が-2の点A、x座標が6の点Bが与えられている。
(1)点Aにおける接線が直線ABと垂直に交わるときのaの値を求めよ。
(2)点Aと点Bのあいだの放物線上に点Pをとる。直線APと放物線で囲まれる図形の面積をS1、直線PBと放物線で囲まれる図形の面積をS2をする。S1+S2の最小値とその時のS1の値を求めよ。

と言う問題なのですが、
(1)の答えはa=1/4です。
(2)の解答は、曲線y=ax^2と線分ABで囲まれる面積は一定であるから、S1+S2が最小になるときは△APBの面積が最大になるときである。△APBは底辺をABとすると、Pにおけるy-ax^2の接線が直線ABと平行になるとき高さが最大となるから、面積も最大になる。
よって、Pのx座標をpとすると、(1)より、
2ap=4a
p=2
このとき、直線APの方程式は、y=4aであるから、
S1=∫(-2〜2)(4a-ax^2)dx=32a/3

とあります。
(1)のAにおける接線が直線ABと垂直に交わる、と言う条件は(2)では残っているのでしょうか?残っていないような気がするのですが…

お願いします。


9664.Re: こんにちは
名前:ケロ    日付:9月7日(日) 23時13分
最初の問題は自分で作ったのかな。
aが変数となる問題のように作られています。だから、一定ではないと思います。
後の問題(1)ではaに条件がないから、答は±1/4じゃないのかな。
(2)は答のようにやれば(1)とは別の独立した問題を解いたことになるし、(1)の続きならばaの値を代入すればいいみたい。わからないけど、独立した問題ならaを定数としても、変数としても同じ答が出るのかな?


9689.Re: こんにちは
名前:味噌汁    日付:9月8日(月) 22時16分
ありがとうございます。

チョット自分でもややこしくなってきてしまったので、
新たに、再度スレっドを立ててもう一度質問し直したいと思います。
すみません…。

a>0の条件が抜けていたり、質問を簡素にしたつもりが、別の問題になってしまったりしたので…

9641.辺の分ける比  
名前:n    日付:9月5日(金) 21時49分
□ABCDにおいて、△ABD∽△BCDでAB:AC:CD=4:5:1であるという
この時、四角形の対角線の交点をPとするとPは対角線BDをどうのような比に内分するか?



9642.Re: 辺の分ける比
名前:n    日付:9月5日(金) 21時58分
自分でやったこと
1 ABをB側に延長し、DCをC側に延長した点をEとしたら∠CBE=
  ∠CDB=∠ADB
この手はここどまりで先がわかりません

2DをC中心に、BをA中心にAC上に移すとその交点は一致する。
この手もここどまりです

以上角、辺の両方の情報を結びつけるうまい手がみつかりません。
お願いします。


9670.Re: 辺の分ける比
名前:ヨッシー    日付:9月8日(月) 10時19分
学年と、単元を書いてください。
高2以上なら、答えを書く用意があります。
それ以下なら、再考します。
 
http://yosshy.sansu.org/


9687.Re: 辺の分ける比
名前:n    日付:9月8日(月) 21時53分
中一です。遅れてすみませんでした。

9639.二次関数  
名前:yuu    日付:9月5日(金) 19時31分
二次関数の頂点の座標の出し方で

a b^2-4ac
-− -−
2,  4a

という公式のY座標の前にあるマイナスはどこかかっているのでしょうか?-b^2-4acですか?それとも?-b^2+4ac?どっちなのでしょうか
教えてください



9640.Re: 二次関数
名前:ジャグラ 高3    日付:9月5日(金) 20時34分
まず2次関数の一般形y=ax^2+bx+cを平方完成してみましょう。
y=a(x+b/2a)^2-b^2/4a+c より切片座標を美しく書くために通分すると
y=a(x+b/2a)^2-b^2+4ac/4a となることが分かると思います。
つまり、-b^2+4acってことですね。


9644.Re: 二次関数
名前:yuu    日付:9月5日(金) 23時34分
ありがとうございました

9637.またまた積分です・・・・  
名前:line-border    日付:9月5日(金) 15時14分
また積分の問題でつまずきました・・・・。
ちなみに今度は不定積分です。
問題は、

2x^3−4x−1/x^4−2x^3−x+2

なのですが、これといっていい方法が思いつきません。
取りあえずわたしは分母を因数分解して、方法を探ってみましたが・・・・・無理でした。
良い方法を教えてください。
お願いします



9643.Re: またまた積分です・・・・
名前:K.N.G.    日付:9月5日(金) 23時3分
被積分関数の分母は
 (x - 1)*(x - 2)*(x^2 + x + 1)
と因数分解できることより
 (2x^3 - 4x - 1)/(x^4 - 2x^3 - x + 2)
= A/(x - 1) + B/(x - 2) + (Cx + D)/(x^2 + x + 1)
と置いて,部分分数に分解してみてはどうでしょうか.

9623.(untitled)  
名前:ジャグラ 高3    日付:9月4日(木) 23時0分
返答ありがとございます。
ヨッシーさんにヒントをいただいたおかげで問題が解けました^〜^
ようは、2次式、1次式、0次式と分けて定数がKであることから
たすき掛けの因数分解によりKの値が導き出されると言うことですよね?
ここで疑問が生まれるのですが、もし定数k^2+kの値を求めよと言う問題
ならたすき掛けによる連立方程式を作りk+1とKをどちらに掛ければよい
と言うこと(ここでは(x-2x)と(x+2x)のことです)はあらかじめ分かる
ものなのでしょうか?
それとも、試行錯誤を行う中で導き出すということなのでしょうか?



9624.Re: (untitled)
名前:ジャグラ 高3    日付:9月4日(木) 23時1分
間違えて別スレを立ててしまいました。
申し訳ありません(m__m)


9626.Re: (untitled)
名前:ast    日付:9月4日(木) 23時7分
何が解けたのでしょう・・・?

(ax+by+c)(dx+ey+f)=0 が 2 直線を表すということは理解された
のでしょうか・・・?


9628.Re: (untitled)
名前:ジャグラ 高3    日付:9月4日(木) 23時14分
astさん>
はい、ヨッシーさんのヒントにより,y^2-4y-4x^2+4x+k=(y+2x)(y-2x)-
4(y-x)+k=0より たすき掛けを行いK=3と言う答えが導かれました。

(ax+by+c)(dx+ey+f)=0 が2直線を表すことは納得致しました^〜^


9630.Re: (untitled)
名前:ast    日付:9月4日(木) 23時24分
何で普通に
   y^2-4y-4x^2+4x+k = (ax+by+c)(dx+ey+f)
と因数分解できるときを考えればよいから, 右辺を展開して
係数比較をしてやればよい. という風に解かないのですか?


9632.Re: (untitled)
名前:ast    日付:9月4日(木) 23時35分
あ, つまり, テクニックが必要な因数分解より, 展開して係数比較
というほうが単純で考えやすいでしょう? ということです.


9633.Re: (untitled)
名前:ジャグラ 高3    日付:9月4日(木) 23時51分
もちろん係数比較も考えましたが、それより早い方法はないかと思考した
所このケースは因数分解を行えると判断したため踏みきっただけで、
特に深い意味はないです^^;


9663.Re: (untitled)
名前:ケロ    日付:9月7日(日) 22時37分
因数分解した式を展開してみてください。もとにもどる?。
yとxを別々に平方完成したらどうかな。
(y-2)^2-(2x-1)^2=-k+3 
となるから、右辺を0にすれば、左辺が因数分解できます


9701.Re: (untitled)
名前:ジャグラ 高3    日付:9月9日(火) 14時22分
ケロさん>
なるほど、その方法でも因数分解できますね〜(^^
私が試した方法もできます。
このような問題(必ず因数分解できる形)はケロさんがおっしゃる方法が
確実かもしれませんね。
あと、判別の補足ありがとう!
高校なのでナニを表しているのかが少し難点ですが、自力で考えて
見ますね。できれば教えて欲しいかも(笑)


9710.Re: (untitled)
名前:ケロ    日付:9月9日(火) 21時57分
ケロにもまったく解りません。実用として使えると思って写し取っただけでござんす。ケロはカエルなので、人間のしていることはなかなかわかりません。Red Catさんならわかるかも。ケロケロ。


9713.Re: (untitled)
名前:帰ってきた赤猫(旧 : Red cat)    日付:9月10日(水) 14時18分
>Red Catさんならわかるかも
ここで我が名を出されたとあっては、復活しないわけにいかんです。

というわけで、あれからわずか 1 週間ですが、皆様お久しぶりです。

問題の判別式ですが、わかるか、といわれると、私もすぐにはわからん
のです(泣)。
#専門用語を持ち出して、「二次形式の〜」とか言っちゃうと、それま
#でですが(ぉ

自宅に戻れば参考書もあるので、何か参考になることが書けるかも知れ
ません。


9722.調べてみました
名前:帰ってきた赤猫(旧 : Red cat)    日付:9月10日(水) 22時5分
調べてみましたが、どうやら行列の階数や符号と密接な関係にあるようです。この辺のことは、大学に入ると習うことの出来る「線型代数」の知識を必要とするので、当面は「実用的な方法」というおさえ方でよいかも知れません。
なお、高校程度の知識で理解できる 2 次曲線の分類方法は
http://www.dslender.com/math/2jibunrui.pdf
が詳しいでしょう。
#私もお世話になっている D Slender さんのサイトより。


9723.おお!ありがとうございます^〜^
名前:ジャグラ 高3    日付:9月10日(水) 22時28分
Red catさん>
わざわざ調べていただき大感謝です。
マイペースに理解して行きたいと思ってます(^^
ケロさんがおっしゃるようにRedcatさんすごいですな!
カエルより猫のが上と(メモメモw  (゜Д゜)


9724.Re: (untitled)
名前:帰ってきた赤猫(旧 : Red cat)    日付:9月10日(水) 23時3分
>カエルより猫のが上と
猫は気まぐれなので、カエル以下です(ぉ


9725.Re: (untitled)
名前:ケロ    日付:9月11日(木) 0時45分
Red cat師匠、ありがとうございます。
この判別式、前々(3百年ぐらい前かな?)から気になっていたのですが、どうせわからないだろうとあきらめていたのでほうっておきました。資料をいただいたので、がんばって勉強してみます。でも、理解するのに5百年ほどは掛かるかなあ。

9617.ヒィッィ><  
名前:ジャグラ 高3    日付:9月4日(木) 22時28分
こんばんわ、また問題を解いていく中で分からない問題に出くわしました。
今回の問題は恥ずかしながら検討の余地がありません;;

問)方程式 y^2-4y-4x^2+4x+k=0が2直線を表すような定数Kの
値を求めよ。

2直線・・・ハテw
感覚的に2つの方向から見るのだと思うのですが・・・
どなたか助けてください。



9619.Re: ヒィッィ><
名前:ヨッシー    日付:9月4日(木) 22時31分
もしこの式が
 (ax+by+c)(dx+ey+f)=0
の形に変形できたならば、
 ax+by+c=0 または dx+ey+f=0
なので、この式を満たす点は、2直線 ax+by+c=0、dx+ey+f=0 のいずれか
の上にあることになります。
 
http://yosshy.sansu.org/


9627.Re: ヒィッィ><
名前:ヨッシー    日付:9月4日(木) 23時10分
たしか、2次曲線の標準化とかいう単元で(高校ではない)
形状の判別(楕円か、双曲線か、放物線か、直線か、等)をする
方法があったと思います。
行列を使ったりするヤツです。
 
http://yosshy.sansu.org/


9629.Re: ヒィッィ><
名前:ジャグラ 高3    日付:9月4日(木) 23時20分
なるほど、高校の範囲内で考える時は思考錯誤により導き出すのですね。
また分からない点が出てくると思いますが、その時はヒントをいただき
いたです・・・。


9661.二次曲線の判別式
名前:ケロ    日付:9月7日(日) 21時59分
検索で見つからなかったので。写し下記。検索係。
http://www.geocities.co.jp/Bookend-Soseki/7526/rakugaki2.htm

9611.最大・最小の入試過去問なんですが。。。  
名前:T.T.C.高2    日付:9月4日(木) 21時53分
1)実数x、yがy≧x2-1、y≦-x/2+2を同時に満たすとき、x+yのとりうる値の範囲を求めよ。
2)実数x、yが方程式x2+y2−2(x+y)−6=0を満たすとき。
a)x+yのとりうる値の範囲を求めよ。
b)x+y-2xyの最大値を求めよ。
わからないのでお願いします。助けてください。



9616.Re: 最大・最小の入試過去問なんですが。。。
名前:ast    日付:9月4日(木) 22時23分
グラフにして視覚的に考えるのが良策でしょう.

1) 二次不等式 y ≥ x^2 - 1, 一次不等式 y ≤ -x/2 + 2
の示す領域をまず書いてみましょう.
両方の領域が重なる部分が条件に合う領域です.
このとき, x+y の取る値が k のとき, 直線 x + y = k が出来上がります.
つまり, k は直線 y = -x + k の y 切片の値になっています.

2) も a) は 1) とおなじですね. b) は考え中.


9621.Re: 最大・最小の入試過去問なんですが。。。
名前:ast    日付:9月4日(木) 22時32分
言い忘れです.
>このとき, x+y の取る値が k のとき, 直線 x + y = k が出来上がります.
>つまり, k は直線 y = -x + k の y 切片の値になっています.
のあとに, 条件の領域に直線 y = -1 + k が交わるときというのは
そのような k を実現する x, y が存在するときということですので,
このような状況で k が最大になるときを考えれば答えがでてきます.
というのを追加します.


9635.(2) 別解
名前:    日付:9月5日(金) 9時18分
a) x2+y2-2(x+y)-6=0 ⇔ (x-1)2+(y-1)2=8
∴16 = (12+12)((x-1)2+(y-1)2)≧(1*(x-1)+1*(y-1))2 (∵シュワルツ)
∴-4≦x+y-2≦4

b) x+y-2xy = x+y-2xy-(x2+y2-2(x+y)-6)
= -(x+y-3/2)2+33/4≦33/4
(x, y)=(3(1+√7)/4, 3(1-√7)/4) のとき等号成立


9638.Re: 最大・最小の入試過去問なんですが。。。
名前:T.T.C.高2    日付:9月5日(金) 18時47分
ありがとうございます。できました。

9610.漸化式  
名前:ゆんりん高3    日付:9月4日(木) 21時46分
学校でまだ漸化式をやってないので、ほとんどわかりません。
できるだけ、丁寧に教えてください!!
(1)a_1=1,a_n+1−a_n=3n (n=1,2,3,……)
   で定められる{a_n}がある。このとき、a_15を求めよ。
(2)数列{a_n}がa_1=0,a_n+1=a_n+n(n+1) (n≧1)
で定義されているとき、一般項a_n およびΣa_kを、求めよ。
よろしくおねがいします(^-^)



9613.Re: 漸化式
名前:ast    日付:9月4日(木) 22時10分
>学校でまだ漸化式をやってないので、ほとんどわかりません。
では習ってからにしましょうw(とか言ってみたい.
>できるだけ、丁寧に教えてください!!
質問者が丁寧さを要求するのはお門違いかと.

閑話休題.
まず, a_n+1 とは a_(n+1) (数列 a_n の第 n+1 項) のことで
良いのですよね? a_n と 1 の和というのと紛らわしいので
表記には気を付けましょう.

(1) {a_n} の階差数列 {b_n := a_(n+1) - a_n} が丁度
等差数列 {3*n} だということです.
階差数列というのは, 数列 {a_n} で a_n が a_(n+1) になるときに
増える量が b_n だという風にして新たな数列にしたものでした.
だから, a_2 = a_1 + b_1, a_3 = a_2 + b_2 = a_1 + b_1 + b_2,
..., とやっていくと, a_15 = a_1 + b_1 +・・・+ b_14 なので・・・.

(2) も同じで, b_n := a_(a+1) - a_n = n*(n+1) になっているので,
a_n = a_1 + Σ_[k=1,...,n-1] b_k となることを使ってやるとできます.


9614.Re: 漸化式
名前:ast    日付:9月4日(木) 22時14分
訂正.
>(2) も同じで, b_n := a_(a+1) - a_n = n*(n+1) になっているので,
b_n = a_(n+1) - a_n のうち間違いです.

#ちなみに, 漸化式の問題は大きく分けて, 「階差数列を計算する問題」と
#「等比数列に帰着する問題」の二種類に分類できます.


9631.Re: 漸化式
名前:ゆんりん高3    日付:9月4日(木) 23時33分
教えて頂いているのに、失礼な言い方をしてしまって
すみませんでした。以後気をつけます。
解説とてもわかりやすかったです(^^)
ありがとうございました!!

9609.質問です。  
名前:タブレット    日付:9月4日(木) 21時44分
[問題]
a,b,cが整数でa^2+b^2=c^2を満たしている。
このとき,cが7の倍数なら,aもbも7の倍数であることを証明せよ。

[自分の解答]
k,lを整数とし、a=7k,b=7lとする。

a^2+b^2=(a+b)^2-2ab=(7k+7l)^2-2*7^2kl=49(k+l)^2-49kl
=49{(k+l)^2-kl}

ここで、c=49{(k+l)^2-kl}とすると、これは7の倍数である。

よって示された。

自分の解答合っているでしょうか?教えてください。

ちなみに、私の年齢は14歳です。



9612.Re: 質問です。
名前:ast    日付:9月4日(木) 22時0分
合っていません.
あなたが証明したのは
「a,b が 7 の倍数ならば c は 7 の倍数である」
ということであり, 仮定と結論が逆です.

本来の証明は, おそらく, a, b をそれぞれ 7 で割った余り
で分類して, c が 7 の倍数になる場合に適合する a,b の条件を
探すという方向でやることになると思います.
#まだ確かめていません.


9618.Re: 質問です。
名前:タブレット    日付:9月4日(木) 22時29分
本来の解答のやり方がいまいちわかりませんので、もう一度考えてみました。よく塾の先生がやっているのですが、逆から書いて証明する方法では正解になるでしょうか?つまり、

k,lを整数とし、c=49{(k+l)^2-kl}とする。(cは7の倍数)

49{(k+l)^2-kl}=(7k+7l)^2-2×7^2kl

ここで、7k=a,7l=bとすると、
(7k+7l)^2-2×7^2kl=(a+b)^2-2×ab=a^2+b^2となる。

よって、cが7の倍数なら、a,bも7の倍数である。


9620.Re: 質問です。
名前:タブレット    日付:9月4日(木) 22時31分
訂正です。

k,lを整数とし、c=7({(k+l)^2-kl})^1/2とする。(cは7の倍数)

c^2=49{(k+l)^2-kl}=(7k+7l)^2-2×7^2kl

ここで、7k=a,7l=bとすると、
(7k+7l)^2-2×7^2kl=(a+b)^2-2×ab=a^2+b^2となる。

よって、cが7の倍数なら、a,bも7の倍数である。


9622.Re: 質問です。
名前:ast    日付:9月4日(木) 22時38分
>逆から〜〜
>k,lを整数とし、c=49{(k+l)^2-kl}とする。(cは7の倍数)
もし, 49{(k+l)^2-kl} という式で, 7 の倍数が「全部」であるなら
さらに, a^2+b^2=c^2 となる a,b が c によって一通りに決まるなら
「逆から書いて証明する方法」は正しい論法でしょう.

では, その二つはきちんと証明できますか? そちらの方が難しいでしょう.
#そもそも正しいかどうかも判らない.


9625.Re: 質問です。
名前:ast    日付:9月4日(木) 23時4分
おそらく, 次のような証明が普通でしょう.

実際に, 自然数を 7 で割った余りで分類して, それぞれを自乗してみると
わかりますが, 自乗して 7 で割った余りは, 0, 1, 2, 4 の 4 通りしか
出て来ません.

また, a^2+b^2=c^2 だというのですから, c^2 を 7 で割った余りと
a^2+b^2 を 7 で割った余りというのは同じ意味です.
ここで, c が 7 の倍数だから, c^2 を 7 で割ったらあまりが 0 です.
だから, a^2+b^2 を 7 で割った余りも 0 なわけです.

a,b がともに 7 の倍数であれば, c^2 は 7 の倍数です.
では, そうでないときはどうなるでしょう?

自乗して加えた後の余りとして考えられる全ての組み合わせは,
  1+1, 1+2, 1+4, 2+2, 2+4, 4+4
となります.
どの組み合わせでもあまりが 0 になることは出来ない.
というわけです.


9634.Re: 質問です。
名前:ヨッシー    日付:9月5日(金) 6時14分
つけたし。

ほぼ自明なのですが、
aが7の倍数で、bがそうでない場合(またはその逆)も、
2 は7の倍数になりません。
 
http://yosshy.sansu.org/


9649.Re: 質問です。
名前:我疑う故に存在する我    日付:9月7日(日) 13時50分
付け足しの付け足し。

a, b, c が a^2 + b^2 = c^2 を満たす整数で、 p が、 p ≡ 3 (mod. 4) を満たす素数である時、c が p で割り切れるならば、 a, b も p で割り切れる。有名な事実です。証明略。この方程式を満たす数(1個の数ではないがピタゴラスの数という)については多くの事実が知られており、いろいろな書物に解説されています。

9607.数学に関係ないかもしれない。_。  
名前:ヒデ(中三)    日付:9月4日(木) 21時19分
<三平方の定理>
古代エジプトの人々は、三平方の定理が発見される以前から、土地を長方形に区分けするために直角を利用したといわれている。
どんな方法で直角をつくっていたか。



9608.Re: 数学に関係ないかもしれない。_。
名前:hippo    日付:9月4日(木) 21時39分
以前テレビの番組でエジプト考古学の吉村作治教授が以下のような話しをしていました。
「古代エジプトの人たちは5:4:3が直角三角形になることを知っていた。紐を用意し等間隔で結び目を作ると簡単に5:4:3の直角三角形が得られる」
この説を立証できる遺物が発見されているか否かは知りませんが、多分このような方法が用いられたものと思います。
http://www5d.biglobe.ne.jp/~tokkii/


9615.Re: 数学に関係ないかもしれない。_。
名前:ヨッシー    日付:9月4日(木) 22時19分
直径の円周角も使えるかも。
 
http://yosshy.sansu.org/

9592.(untitled)  
名前:あいこ(高1)    日付:9月3日(水) 23時19分
A={X|-2≦X≦2}、B={X|X≦1または5≦X}とするとき、つぎの集合を求めよ。ただし全体集合は実数全体とする。
   _    
(1)A∪B

答えは={X|-2≦X<5}でした。なぜ不等号がそれぞれ≦や<になるのですか?



9593.(untitled)
名前:あいこ(高1)    日付:9月3日(水) 23時20分
ちなみに A∪BのBのうえに横棒がつきます。


9595.Re: (untitled)
名前:Bob    日付:9月3日(水) 23時32分
Aのバ−={X|X<−2,2<X}
Bのバ−={X|1<xかつx<5}
   ={X|1<x<5}

A∪(Bのバー)={X|-2≦X≦2}∪{X|1<x<5}
       ∪は和集合なので2つの範囲をあわせると
       ={X||-2≦X<5}

くわしくは数直線上に記入しながら<や>は含まないという意味なので
○にし
≦は●にしていきます。
        
http://homepage3.nifty.com/sumida-3/


9599.Re:
名前:あいこ(高1)    日付:9月4日(木) 0時12分
わかりましたぁ!!!そのようにして導くんですね!!ありがとうございました☆

9591.線分の和の最小  
名前:ジャグラ 高3    日付:9月3日(水) 22時51分
こんばんわ、ある問題がいまいち分からないので、私なりに考えた式の
どこが間違っているかを指摘していただけると嬉しいです(><
また、上記の考え方より早い解答に結びつくアルゴリズムがあれば
教えていただきたいです・・・。

問)2点A(1、4),B(12、−3)と直線L:X+Y−1=0がある。
直線L上に点Pをとり、AP+BPを最小にするとき、点P座標を求めよ。

自答)点P座標を(x,y)とおくと、AP=√(1-x)^2+(4-y)^2
BP=√(12-x)^2+(-3-y)^2
題意より点Pは直線L上に存在することから、x+y-1=0となるx,yが存在し
x=-y+1ー(1)  (1)よりAP、BPを書き直すと、
AP=√y^2+(4-y)^2 BP=√(11+y)^2+(-3-y)^2
さらに進めて AP+BP=√2(y-2)^2+√2(y+7)^2-33

AP+BPが最小になりうるyの値は(y-2)^2+(y+7)^2の最小値のその時の
yの値であり、f(y)=2y^2+10y+53 平方完成して
f(y)=2(y+2/5)^2+K の形になり私はてっきりy=-2/5の時最小となる
値を取るのかなと思ったのですが、答えを拝見して見ますと、
y=-1の時最小となると書かれてあるのです^^;
どこが違うのでしょうかねぇーー;



9596.Re: 線分の和の最小
名前:高橋 道広    日付:9月3日(水) 23時35分
AP=√y^2+(4-y)^2 BP=√(11+y)^2+(-3-y)^2
さらに進めて AP+BP=√2(y-2)^2+√2(y+7)^2-33
がおかしいようですよ
AP=√(2y^2-8y+16)は√{2(y-2)^2}+8にはなりませんから
それにしても全体で-33にはならないか...

AP+BPが最小になりうるyの値は(y-2)^2+(y+7)^2...もおかしいから
√a+√b=√(a+b)みたいな計算をやっているんじゃないかな
これはなりたちませんよ

解法のひんと 「まがってるものはまっすぐに。」
直線Lに関してAと対称な点A'をとると AP+BP=A'P+BP 
これが最小になるときを求めます。このへんでいったん意地悪して
ヒント終わり。模試できなかったらどこまでやれたか書いといて。
きっと他の人たちが答えてくれるから(^_^;)
http://micci.sansu.org


9597.Re: 線分の和の最小
名前:ヨッシー    日付:9月3日(水) 23時37分
式の指摘は、高橋さんがやられたので、こちらは、ヒントの図を。



まぁ、こういうことですね。
 
http://yosshy.sansu.org/


9600.Re: 線分の和の最小
名前:ジャグラ 高3    日付:9月4日(木) 0時29分
返答ありがとうございます。

途中で計算ミスをしてしまいましたーー;
私が提案した解法は正確に計算すると解答に結びつくのでしょうか?
確かに非効率ではあると思いますが...

2人が教えてくださった解法はこの問題では一番適切でしょうね〜。
もし、また分からない点や疑問に思ったことが生じた場合はまた
よろしくです^〜^


9602.Re: 線分の和の最小
名前:ヨッシー    日付:9月4日(木) 8時52分
>私が提案した解法は正確に計算すると解答に結びつくのでしょうか?
おそらくダメでしょう。
高橋さんも指摘されているように
>√a+√b=√(a+b)みたいな計算をやっているんじゃないかな
>これはなりたちませんよ
これはもとより、
ある条件下で a+b を最小にする a,b が √a+√b を最小にする
とは限りません。
 a=1,b=25 のとき a+b=26 √a+√b=6
 a=16,b=9 のとき a+b=25 √a+√b=7
a+b は小さくなっているのに、√a+√b は大きくなっています。
 
http://yosshy.sansu.org/


9606.Re: 線分の和の最小
名前:ジャグラ 高3    日付:9月4日(木) 19時29分
返答ありがとうございます。
なるほど、おかげで納得する事ができました^^
思考した計算式は失敗に終わりましたが、納得できたので
次回に生かせそうです。

9589.(untitled)  
名前:ゆんりん高3    日付:9月3日(水) 22時19分
問題の7/10というのは間違いで、10/7です。
失礼しました!!

9588.群数列教えてください。  
名前:ゆんりん高3    日付:9月3日(水) 22時17分
数列1/1,2/1,1/2,3/1,2/2,1/3,4/1,3/2,2/3,1/4・・・・・
において7/10が出るのは何項目でしょう。
おいう問題なんですけど、どこで群を分けたらいいんですか?
できるだけ詳しい解説おねがいします**



9590.Re: 群数列教えてください。
名前:ヨッシー    日付:9月3日(水) 22時48分
分母+分子が、
2:1/1
3:2/1, 1/2
4:3/1, 2/2, 1/3
 ・・・
16:15/1, 14/2, 13/3, ・・・ 1/15
17:16/1, 15/2, 14/3, 13/4, 12/5, 11/6, 10/7, ・・・

第15群(分母+分子が16)の最後の 1/15 が、
1+2+3+・・・+15 項目
その7つあとが 10/7 です。
 
http://yosshy.sansu.org/


9598.Re: 群数列教えてください。
名前:ゆんりん高3    日付:9月3日(水) 23時55分
なるほど!とてもわかりやすかったです(^^)
ありがとうございました★☆

9582.確率の問題です  
名前:不自由な二択    日付:9月3日(水) 15時45分
高校2年の”不自由な二択”といいます
この問題が分らないので答えと
よければ式を教えていただけませんかませんか?

赤の玉2個.白の玉8個が入った箱から
玉を1個ずつ、1個目の赤球(2個のうちどちらでも良い)が出るまで
非復元抽出で取り出すとき、取り出す玉の期待値を求めよ

・・・ある問題集の問題なのですがはっきり言って訳分りません(^_^;)
非復元抽出とか確率とか調べたけど難しくてよく分りません
自力では不可能と思い書き込みさせていただきました、
どうぞよろしくお願いします



9583.Re: 確率の問題です
名前:ヨッシー    日付:9月3日(水) 16時26分
「非復元抽出」とは、なんとも大層な名前ですが、
「箱から玉を取り出して再び戻す」が復元抽出で、
「取り出した玉は戻さない」が非復元抽出でしょう。おそらく。

取り出す玉の個数が
1個:
 1個目に赤が出る確率は、10個のうちの2個なので、2/10=1/5
2個:○
 1個目に白が出る確率は、10個のうち8個なので、8/10=4/5
 2個目に赤が出る確率は、9個のうち2個なので、2/9
 よって、○ の順に玉が出る確率は、
  4/5 × 2/9 =8/45
3個:○○
 1個目に白が出る確率は、10個のうち8個なので、8/10=4/5
 2個目に白が出る確率は、9個のうち7個なので、7/9
 3個目に赤が出る確率は、8個のうち2個なので、2/8=1/4
 よって、○○ の順に玉が出る確率は
  4/5 × 7/9 × 1/4 =7/45
4個:○○○
 同様に
  4/5 × 7/9 × 3/4 × 2/7 =6/45
以下、
5個:1/9
6個:4/45
7個:1/15
8個:2/45
9個:1/45
それぞれの個数に確率を掛けて合計したものが個数の期待値です。
 1×1/5 + 2×8/45 +3×7/45 + ・・・ +9×1/45=11/3 ・・・答え
 
http://yosshy.sansu.org/


9636.Re: 確率の問題です
名前:不自由な二択    日付:9月5日(金) 10時54分
とても分りやすく教えていただきありがとうございます、
問題が分らず半分諦めかけていたのですが
ヨッシーさんの御教授のおかげで私のようなものでも分るようになりました
重ねて御礼申し上げます
m(_ _ )m ペコ

9580.微分と積分  
名前:ボス    日付:9月3日(水) 14時29分
次の関数を微分せよ。
(1)∫(0→tanx)dt/{1+(tanx)^2}
(2)∫(−x→2x)f(t)dt

次の不等式を示せ。
(1)π/3√5<∫(0→1/2)dx/√(1-x^4)<π/6
(2)log(n+1)<煤ik=1→n)1/k<1+logn (nは整数)

お願いします。



9601.Re: 微分と積分
名前:ボス    日付:9月4日(木) 0時56分
訂正です。(2)のf(t)は連続です。
よろしくお願いします(-.-)


9604.Re: 微分と積分
名前:ヨッシー    日付:9月4日(木) 16時40分
まずは、不等式の(2)です。

曲線は y=1/x のグラフ。
赤の長方形の面積の和が 煤ik=1→n)1/k です(左右とも)。
左の図で、青い部分が曲線を1からn+1まで積分て得られる面積で、log(n+1)
に当たります。
右の図で、赤の長方形に青の部分を加えたものが
一番左の長方形(面積1)+(1からnまでの積分)で、1+logn に当たります。
 
http://yosshy.sansu.org/


9605.Re: 微分と積分
名前:ヨッシー    日付:9月4日(木) 16時47分
微分の方の
(1)は、tanxは、tに関係ないので、単純に
 ∫dt/{1+(tanx)^2}=t/{1+(tanx)^2}
と書けるでしょう。定積分にすると、
 ∫(0→tanx)dt/{1+(tanx)^2}=tanx/{1+(tanx)^2}
これを、xに付いて微分しろと、そういうことでしょうか?
(2)は、f(t) の原始関数の1つを F(t) とおくと、
 ∫(−x→2x)f(t)dt=F(2x)−F(-x)
なので、これをxで微分すると、dF(x)/dx=f(x) なので、
 d(F(2x)−F(-x))/dx=2f(2x)+f(-x)
かな?
 
http://yosshy.sansu.org/


9698.Re: 微分と積分
名前:ボス    日付:9月9日(火) 12時2分
すいません。
(1)の訂正です。
次の関数を微分せよ。
(1)∫(0→tanx)dt/{1+t^2}でした。

よろしくおねがいします。


9699.Re: 微分と積分
名前:ボス    日付:9月9日(火) 12時9分
(2)のd(F(2x)−F(-x))/dx=2f(2x)+f(-x)
の部分がわかりません。なぜdF(x)/dx=2f(2x)で-dF(-x)/dx=f(-x)に
なるのでしょうか?

9579.積分っす・・・  
名前:line-border    日付:9月3日(水) 14時20分
どうも、はじめまして。高校3年の
line-borderといいます。

えっと、√1+t^2(ルート1プラスtの2乗)の
0〜4の積分の問題がわからないんです。

ちなみに答えはたしか・・・
2√17+log(4+√17)だったかなあ?
どうしてこのようになるのかがわかりません。
方法のヒントなどを教えていただけると幸いです。



9581.Re: 積分っす・・・
名前:line-border    日付:9月3日(水) 15時9分
あっと、
0〜4は
0→4のことです。
すみません


9584.Re: 積分っす・・・
名前:中川 幸一    日付:9月3日(水) 16時34分
この問題は置換積分を用いて解きます。

t=sinh x=(ex-e-x)/2 と置いてみましょう。

http://www3.ezbbs.net/01/k-nakagawa/


9594.Re: 積分っす・・・
名前:nabeX    日付:9月3日(水) 23時31分
√(1+x2)=t-xと置いて両辺平方すると
1+x2=t2-2tx+x2
よってx=(t2-1)/(2t) となります。
これでも置換積分が可能です。


9603.Re: 積分っす・・・
名前:line-border    日付:9月4日(木) 11時27分
お二人ともありがとうございました。
早速やってみます。

9574.数学とは関係ないんですが…。  
名前:Red cat    日付:9月3日(水) 8時53分
9298. から続いた一連の発言に置ける、自らの発言に対する責任と戒めの意味を込め、本発言を持ってこの掲示板から身を引くこととします。

途中で出入禁止を言い渡されてもおかしくない状態でした。

管理人様をはじめ、利用者の方々に不快な思いをさせてしまったことを、深くお詫び申し上げます。



9576.Re: 数学とは関係ないんですが…。
名前:中川 幸一    日付:9月3日(水) 9時8分
>>Red cat さんへ

お久しぶりですね。
以前某掲示板で整数問題や色々な問題について話し合ったことが懐かしく思います。

>>この掲示板から身を引くこととします。
えっ?この掲示板から去ってしまうのですか?
的確なアドバイスをなされる Red cat さんがこの掲示板から引退なされると言うことはとても残念です。
(かくいう私もあの某掲示板からは訳あって身を引きましたが…。)

私一個人としての意見は, 戻ってきて欲しいです。


良ければ今度私の方の掲示板にも遊びに来てください。

それでは。

http://www3.ezbbs.net/01/k-nakagawa/


9577.Re: 数学とは関係ないんですが…。
名前:幻想曲とフーガ    日付:9月3日(水) 10時43分
>9298. から続いた一連の発言に置ける、自らの発言に対する責任と戒めの
>意味を込め、本発言を持ってこの掲示板から身を引くこととします。


本気なのですか、Red cat さん。
一度や二度の失言は、我々が人間である以上は誰にでもありえることです。
9298. から続いた一連の発言の中に、失言であるものが含まれているとあなた
自身がお考えになるのであれば、次回からはそのような発言をしないように気
をつければよいだけのことではありませんか。あなたは回答者としては非常に
有能な人であると私は思います。しかしそのあなたが、数学以外のことに気を
とられすぎて右往左往しているのを見るのは残念でなりません。
尤も、「この掲示板から身を引く」というあなたの決意が強固なものであるの
ならば、私はこれ以上、口をはさむつもりはありません。


9578.Re: 数学とは関係ないんですが…。
名前:ヨッシー    日付:9月3日(水) 14時3分
これは、管理人としてお引き止めせねばなりますまい。

投稿された記事は、即座に削除されたものも含めて、すべて管理人宛にメールされます
ので、この一連の記事もすべて目を通していました。
「質問掲示板」としてはどうかとも思いましたが、使用上の注意には、
何でもありと謳っていることと、内容的にも面白そうだったので、そのままに
しておきました。
あと、確固たる根拠と確信がなかったので、コメントはできませんでした。

というわけで、特に不快には思っておりません、というのが私の主観であり、
削除には及ばずというのが私の感性です。それに対して、他の方々が様々な
思いを抱かれたとしても、それらはすべて私の責任です。

そんなわけで、Red cat さんには、これからもお立ち寄りいただいて、回答
その他にわたり助けていただきたいと思います。

今すぐは無理なら、少し間をおいてでも良いので、また戻ってきて下さい。

9574 の記事こそ、削除してしまって、なかったことにしてしまいたいです。
(管理人にはそれができるのですが、とりあえず今はしません ^^; )
気が変わったら、今回例外的にメールアドレスを載せましたので、メールを
下されば、削除します。
もちろん、ご自分で処理なさっても結構です。

それでは、長文失礼しました。
 
http://yosshy.sansu.org/


9646.帰って来て下さい。
名前:我疑う故に存在する我    日付:9月6日(土) 18時41分
私は、質問者の質問に直接回答するより、他人の回答に「補足」するのが性に合っています。Red cat さんが居ないと、書き込みにも張り合いが出ません。是非戻ってきて下さい。


9716.ご心配をお掛けしました。
名前:帰ってきた赤猫(旧 : Red cat)    日付:9月10日(水) 16時46分
一週間は自粛期間としては短いかも知れませんが、気持ちも落ち着き、
復帰することにしました。既に2件ほど書き込みをさせていただいてお
ります。

皆様から温かい励ましのコメントを頂いたことに心より感謝を申し上げ
ます。これからもよろしくお願いします。

9569.うーん…  
名前:味噌汁    日付:9月2日(火) 20時9分
こんにちは。
ネット上で「代数学の基本定理」をgoo検索して、その証明を勉強しようとしましたが、難しくて駄目でした…。読んでもちんぷんかんぷんです…。
そこで、厳密性などどうでもいいので、大体でかまわないので、「代数学の基本定理の証明」はどんな感じで証明をして、どんな感じのことを言っているのか小学生にもわかるような感じで教えていただけないでしょうか…。(アセアセ)

とりあえずは、[厳密性はどうでもいい]ので、日本語で理解したいです…^^;

お願いします。

9561.゚゚(´□`。)°゚。ワーン!!分かりませんbQ 中三  
名前:IGA    日付:9月2日(火) 19時1分
a+b=√5,a-b=2のとき、a^5b^5の値を求めなさい。

という問題なんですが・・・何も思いつかなかったので、問題集の解答を見てたどっていったんですが、疑問点があるので質問します。

(a+b)^2-(a-b)^2=4abであるから
ab=1/4{(√5)^2-2^2}=1/4
与式・・(1/4)^5=1/1024
になるのですが(a+b)^2-(a-b)^2=4abであるから
ab=1/4{(√5)^2-2^2}=1/4
はわかります。
しかし(1/4)^5=1/1024の式がいまいちぴんときません。
なぜなら1/4=abなわけでこれを五乗したらa^5b^5と違うような気がするのですが・・・
教えてください。



9565.Re: ゚゚(´□`。)°゚。ワーン!!分かりませんbQ 中三
名前:Sar    日付:9月2日(火) 19時22分
ううむ、要は

「(a^5)×(b^5)と(ab)^5の値は違うのではないか」という事でしょうか。
aやbに適当な値を代入してみて、(a^5)×(b^5)と(ab)^5の値を比較してみてはいかがでしょうか?
(要するに、5乗して掛けるか掛けて5乗するかの違いですね)


9566.Re: ゚゚(´□`。)°゚。ワーン!!分かりませんbQ 中三
名前:IGA    日付:9月2日(火) 19時58分
Sarさん!
ありがとうございます。
自分で実際代入したらちゃんとできました。
なるほど、いっしょですね。

9559.ゥ゚゚(´□`。)°゚。ワーン!!わかりません。  
名前:IGA    日付:9月2日(火) 18時49分
ax+by-bx-ay-3x+3yを因数分解しなさい。
という問題です。
それでやってみたのですが・・・
-a(-x+y)+b(-x+y)+3(-x+y)
=-aA+bA+3A
=A(-a+b+3)
=(-x+y)(-a+b+3)
になるんですが・・・よさげにみえるのですが、
答えは(a-b-3)(x-y)
なるのです。答えが違うということなので、
私の途中式を指摘してくれませんか?
おねがいします。



9560.Re: ゥ゚゚(´□`。)°゚。ワーン!!わかりません。
名前:ast    日付:9月2日(火) 18時56分
>答えが違うということなので、
違いません, 同じです. 括弧の頭にマイナスがあるのは気持ち悪いので
二つ括り出してあるだけです.


9562.Re: ゥ゚゚(´□`。)°゚。ワーン!!わかりません。
名前:IGA    日付:9月2日(火) 19時3分
あ〜なるほど。
こういう計算はすべて符号を逆にしてもいいのですね?
わかりました。
ありがとうございます。


9567.Re: ゥ゚゚(´□`。)°゚。ワーン!!わかりません。
名前:ast    日付:9月2日(火) 19時59分
>こういう計算はすべて符号を逆にしてもいいのですね?
語弊がありますね. 今の場合はたまたま (-1) を "2 つ" 括りだしたので
単純に符号を逆にしている "ように見える" だけです.
きちんと因数分解の基本を復習しましょう.


9570.Re: ゥ゚゚(´□`。)°゚。ワーン!!わかりません。
名前:IGA    日付:9月2日(火) 21時24分
いや、すいません。
表現の仕方が悪かったです。
ax+by-bx-ay-3x+3yをくくり出す場合
-a(-x+y)+b(-x+y)+3(-x+y)→a(x-y)-b(x-y)-3(x-y)
ってことの違いですね。
そうですよね?


9573.Re: ゥ゚゚(´□`。)°゚。ワーン!!わかりません。
名前:ast    日付:9月3日(水) 2時30分
OK です.


9587.Re: ゥ゚゚(´□`。)°゚。ワーン!!わかりません。
名前:IGA    日付:9月3日(水) 22時9分
astさん!ありがとうございました。
m(_ _)m<(_ _*)> アリガトォ

9552.証明を教えてください。  
名前:道産子中学生    日付:9月2日(火) 14時44分
下の証明問題の方法が全く分かりません。教えていただけますか?

任意の三角形の各頂点の内角3等分線を引いた時、それぞれの内角3等分線が他の内角3等分線と初めて交わる時、内角3等分線を引くのをやめる。その時に新しく出来る点をP、Q、Rとする時、三角形PQRは常に正三角形である事を証明せよ。

です。



9553.Re: 証明を教えてください。
名前:ヨッシー    日付:9月2日(火) 15時19分
モーレーの三角形というものですね。
こちらなどを見てみて下さい。
 
http://yosshy.sansu.org/


9556.Re: 証明を教えてください。
名前:中川 幸一    日付:9月2日(火) 16時58分
この問題は複素数平面を用いても解けます。
http://www3.ezbbs.net/01/k-nakagawa/


9572.Re: 証明を教えてください。
名前:道産子中学生    日付:9月2日(火) 23時43分
どうも、ありがとうございます。

しかし、こんなに複雑なのをスラスラと解ける皆様は凄いです。

では、しつれいします。

9537.そのまんまです  
名前:高橋 道広    日付:9月1日(月) 19時35分
P(a,a^2) Q(b,b+16)とすると
(a+b)/2=1、(a^2+b+16)/2=10から
a+b=2 a^2+b=4
(a,b)=(2,0),(-1,3)
ができてきます。
http://micci.sansu.org/



9538.Re: そのまんまです
名前:高橋 道広    日付:9月1日(月) 19時36分
失敗 失敗
9532に対する返信ですm(__)m
http://micci.sansu.org/


9543.Re: そのまんまです
名前:IGA    日付:9月1日(月) 21時19分
高橋さんありがとうです。<(_ _*)> アリガトォ
多忙の中本当に有り難うございました。
以外とこの問題は簡単でしたか・・・(;´Б(汗
文字をとりあえずおくということを学びました。

9532.お願いします  
名前:IGA    日付:9月1日(月) 18時8分
Original Size: 925 x 443, 17KB

次の図で、点A(ー2、4)、点B(4,16)が関数y=x^2のグラフ上にあり、点Pはそのグラフ上をAからBまで動く。線分ABの中点をMとし、Mに関して点Pと対象な点Qをとって、平行四辺形APBQを作る。このとき、次の問いに答えよ。


問題
点Qが直線y=x+16上にあるとき、点Pはどのような位置にあるか。点Pのx座標を、すべて求めよ。

という問題なんですが、とりあえず点Mの座標を求めました。
そうするとM(1、10)になります。
この交点の座標をうまく使って方程式を作ろうと思うのです。
つまり二点のx座標同士をたし二で割り、二点のy座標同士をたし二で割ると中点の座標が求まるという公式をうまく使って解きたいのですが・・どうすればいいでしょうか?教えてくれませんか?



9548.Re: お願いします
名前:Red cat    日付:9月2日(火) 11時31分
P は放物線 y = x^2 の上の点なので、P(s,s^2) と置けます。
Q が直線 y = x + 16 の上の点だとすれば、Q(t,t + 16) と置けます。
すると、PQ の中点の座標は ((s + t)/2,(s^2 + t + 16)/2) です。
Q が M について P と対称の位置にあると言うことは、PQ の中点が
M(1,10) に一致することと同じです。だから

(s + t)/2 = 1 , (s^2 + t + 16)/2 = 10
一個目の式から t = 2 - s となるので、これを二個目の式に代入する
と s が求まり、s の値に応じて t も求まります。


9563.Re: お願いします
名前:IGA    日付:9月2日(火) 19時5分
ありがとうございました!!<(_ _*)> アリガトォ
感謝します。

9531.勉強方  
名前:数学バカ    日付:9月1日(月) 17時44分
高校数学のよい勉強方はありませんか?学校の先生が言うには「問題を解きまくれ」って言うのですが、もっと具体的にありませんか?問題の質問ではなくてすみません。



9535.Re: 勉強方
名前:あいこ(高1)    日付:9月1日(月) 18時53分
高1なんで参考になるかわかりませんが、問題を解くとき、難しい問題にでくわしたとき、分からなくても少し考えてから答えを見ます。とけなっかた問題は必ず解けるようにします。そして解けなっかた問題の反復も重要です!!!!本当に自分のものになるまで反復は必要です!とにかく反復です!!あと、なんとなく解けた問題やあやふやな箇所は絶対にそのままにしない!!簡単な問題が解けると、発展的な問題を解きたくなります。ぜひ解いてください!わからなくて嫌気が差すときがあるかもしれませんが、がんばってください!!そのぶん分かったときの喜びは大きいです!!参考にならなっかたらすみません(>n<)


9540.Re: 勉強方
名前:数学バカ    日付:9月1日(月) 19時59分
ありがとうございました


9549.Re: 勉強方
名前:我疑う故に存在する我    日付:9月2日(火) 11時51分
自分が解けた、理解できたと思う問題、事柄をまだ解けてない、理解できていないという人に教えてあげてみてください。その過程の中で、自分が本当の理解に到達していなかった事に気づくことが良くあります。そこを考えているうちに第2段、第3段の理解に到達します。又人に物を教える時は、単なる理解だけでなくいろいろな実例を準備しておく(知っておく)方がいいことに気が付くでしょう。他にも色々ありますが、先ずはこの辺りから実践してみては?

9529.いろいろな数列の和  
名前:ゆんりん    日付:9月1日(月) 17時23分
nを自然数とするとき、
(n+1)^2+(n+2)^2+・・・・+(n+n)^2=( )
の( )に入るのはなんですか?
という問題なんですけど、いまいち数列の和がわからなくて
困っています。。お願いします★★



9533.Re: いろいろな数列の和
名前:田村 正和    日付:9月1日(月) 18時13分
与式={煤i1〜2n)k2}−{煤i1〜n)k2


9542.Re: いろいろな数列の和
名前:ゆんりん    日付:9月1日(月) 20時44分
すみません。あまりその式の意味がつかめないんですけど、
どうやったらそういう式が出てきたのか
教えて下さい(>_<)


9550.Re: いろいろな数列の和
名前:ヨッシー    日付:9月2日(火) 12時31分
生徒「11+12+・・・+20 を計算したいのですが」
先生「1+2+・・・・+20 は知ってますか?」
生徒「210です」
先生「1+2+・・・・+10 はわかりますか?」
生徒「55です」
先生「では、210から55を引けばいいでしょう」

ってことです。
 
http://yosshy.sansu.org/


9586.Re: いろいろな数列の和
名前:ゆんりん    日付:9月3日(水) 20時6分
あっあるほど!全体から(n+1)^2までの和をひいたら
答えが出るんですよね??
わかりました**ありがとうございました(^−^)

9527.因数分解  
名前:TJ(中1)    日付:9月1日(月) 16時49分
x^2+4/3x=0←これって因数分解出来ますか?



9528.Re: 因数分解
名前:田村 正和    日付:9月1日(月) 17時2分
xでくくってx(x+4/3)


9530.Re: 因数分解
名前:TJ(中1)    日付:9月1日(月) 17時24分
あっそだった!!本当にアホですわ・・・解答ありがとうございます!!

9520.全統模試の問題で...  
名前:あいこ(高1)    日付:9月1日(月) 0時1分
実数の定数aに対して、I=|a+√2|+1、y=|a-√2|+1とする。a=2のとき、次の式の値を求めよ。
(@)I+y
っという問題なのですが、まず、a=2をI=|a+√2|+1、y=|a-√2|+1 にそれぞれ代入して、
    I=|2+√2|+1、y=|2-√2|+1 
になります。その次に解答には、
     2+√2>0、2-√2>0より 
っとなっているのですが、なぜ絶対値記号の中の数がそれぞれ0よりも大きいと言えるのでしょうか?おしえてください!!



9521.Re: 全統模試の問題で...
名前:中川 幸一    日付:9月1日(月) 0時6分
2 と √2 の大小関係はどうなっているでしょう?
http://www3.ezbbs.net/01/k-nakagawa/


9534.Re: 全統模試の問題で...
名前:あいこ(高1)    日付:9月1日(月) 18時39分
1<√2<2なので、√2<2でしょうか?


9539.Re: 全統模試の問題で...
名前:あいこ(高1)    日付:9月1日(月) 19時58分
2+√2>0、2-√2>0 っと、なる理由わかりました!!しかし1<√2<2はあっているか不安なのですが;


9554.Re: 全統模試の問題で...
名前:Bob    日付:9月2日(火) 15時31分
1<√2<2はあっています

√1<√2<√4 ですからね。
http://homepage3.nifty.com/sumida-3/


9568.Re: 全統模試の問題で...
名前:あいこ(高1)    日付:9月2日(火) 20時6分
ありがとうございました!!


EZBBS.NET produced by Inside Web